Infectious Dz Q's

Lakukan tugas rumah & ujian kamu dengan baik sekarang menggunakan Quizwiz!

A 70-year-old woman presents with cellulitis of her left foot that began yesterday. She does not have diabetes. She thinks she scratched her great toe while walking barefoot in her rock garden at home. Otherwise, she is without complaint. She is afebrile and other vital signs are normal. You note a 8 cm x 4 red area extending from the base of her 1st distal phalanx onto the plantar surface of her foot. There is no drainage or signs of penetrating trauma. She is not allergic to any medications.. She has no history of MRSA and has not been in a hospital setting. Which of the following is the best antibiotic to treat the most likely organism? A) IV ceftriaxone B) PO cephalexin C) IV cefazolin D) PO trimethoprim-sulfamethoxazole E) IV vancomycin

B. PO cephalexin Explanation This patient has cellulitis and should be managed according to the 2014 Infectious Diseases Society of America guidelines for the treatment of skin and soft tissue infections. Cellulitis should be divided into purulent vs. non-purulent, this case representing the latter given the absence of pus. Next, the severity should be determined. Patients who are septic and/or have evidence of penetrating trauma, immunocompromise and/or necrotizing fasciitis are classified as severe and require surgical intervention and empiric vancomycin and piperacillin-tazobactam. Cases with systemic signs but not meeting the criteria for severe disease are classified as moderate severity and should receive IV therapy with antibiotics directed at Streptococcus pyogenes such as penicillin, cefazolin, ceftriaxone, or clindamycin. Patients with no systemic signs and no criteria for severe disease (this patient) are considered mild and should receive oral therapy with an antibiotic predictively active against S. pyogenes such as penicillin, cephalexin, dicloxacillin, or clindamycin. Purulent cellulitis should be drained (sufficient therapy for mild cases), and receive oral therapy directed at methicillin-resistant Staphylococcus aureus (trimethoprim-sulfamethoxazole or doxycycline) if there are systemic signs (moderate severity) or IV therapy (vancomycin, daptomycin, linezolid, televancin, or ceftaroline) if there is sepsis, failure of prior therapy, or immunocompromise (severe disease). Conjugated meningococcal vaccine (Menactra®, Menveo®) is recommended for all adolescents today but particularly for military recruits, college students living in dormitories, and travelers to endemic areas of meningococcal disease.

A 28-year-old man presents with high fever, aches, and pains and says he "feels like a Mack-truck ran over me this morning." You are aware of influenza B in the community and have seen multiple cases in the past week. He became acutely ill this morning about 4 hours ago. Which of the following would be helpful in reducing his symptoms and diminishing his viral shedding? A) Amantadine or rimantadine. B) Oseltamivir. C) Amantadine, rimantadine, or oseltamivir would be helpful. D) Rimantadine is effective, but amantadine is not as effective. E) Acyclovir.

Answer B. Oseltamivir. Explanation Note this is the only agent listed that is effective for influenza B! Amantadine and rimantadine treat only certain strains of influenza A, while oseltamivir or zanamivir get most strains of influenza A and all strains of influenza B. Acyclovir is effective only against herpes viruses.

Which of the following antibiotics should not be used in a pregnant woman? A) Vancomycin B) Oxacillin C) Ciprofloxacin D) Cefepime E) Azithromycin

Answer C. Ciprofloxacin Explanation Remember you shouldn't use any quinolone in pregnancy. Also, tetracyclines are contraindicated as well. The other drugs listed would be okay to use if necessary.

A 50-year-old Hispanic man comes to your office because of a 2-week history of fever, sore throat, shortness of breath, nonproductive cough, and vague pleuritic chest discomfort. He also has had swelling in the neck and night sweats for 3 months. He has not consumed unpasteurized milk products. He has 15 healthy pet cats. PAST MEDICAL HISTORY: Negative SOCIAL HISTORY: Denies IV-drug use Admits to homosexual activity Drinks tequila—a 1/2 pint daily Doesn't smoke PHYSICAL EXAMINATION: Temp 102° F, P 110, RR 28, BP 135/90 mmHg HEENT: PERRLA, EOMI TMs clear Throat: Oral candidiasis Neck/Lymph: 5 x 5-cm firm, rubbery, non-fixed lymph nodes in the right supraclavicular fossa; 1-cm nodes are noted in the anterior and posterior cervical chains, axillae, and inguinal areas Heart: RRR with I/VI flow type murmur Lungs: Bibasilar crackles Abdomen: Bowel sounds present; liver span 15 cm Extremities: No cyanosis, clubbing, or edema LABORATORY: Chest X-Ray: Bilateral lower lobe interstitial infiltrates and right paratracheal and bilateral hilar lymph node enlargement Hemoglobin: 13 g/dL WBC: 4,500/cu mm HIV ELISA: Positive HIV Western blot: Pending CD4 lymphocyte: 45/cu mm Serum bilirubin: 1.8 mg/dL AST: 250 U/L pH: 7.43 PO2: 64 mmHg PCO2: 33 mmHg Which of the following is most likely responsible for his illness? A) Rhodococcus equi B) Pneumocystis jiroveci C) Mycobacterium tuberculosis D) Bartonella henselae E) Coccidioidomycosis

Answer C. Mycobacterium tuberculosis Explanation This man most likely has AIDS. He has scattered lymphadenopathy with lower lobe infiltrates. He does not have severe hypoxia or disseminated diffuse disease on CXR, helping to differentiate TB from Pneumocystis. The hilar adenopathy helps you lean toward tuberculosis also. The other organisms are much less likely. Look for Bartonella with skin lesions; Rhodococcus is possible but much rarer than Mycobacterium tuberculosis. He has no travel history to suggest that coccidioidomycosis should be considered.

A 25-year-old male presents with acute arthritis of his left knee. He has a history of recurrent sore throats over the past year. Sometimes he went to the physician, and other times he ignored the symptoms, and the soreness improved. He has had persistent fever with the onset of the arthritis. When you listen to his heart, you hear a new systolic murmur that was not there 2 years ago. A throat culture for group A streptococcus is negative. Which of the following do you tell this patient about rheumatic fever? A) Prophylactic antibiotic therapy is not necessary for those with rheumatic fever who do not have carditis. B) Group A streptococcal skin infections can be responsible for rheumatic fever. C) Acute rheumatic fever usually occurs within the first week of untreated group A streptococcus infection. D) Diagnosis of acute rheumatic fever in an adult presenting with migratory arthritis and a new heart murmur requires confirmatory evidence of recent group A streptococcal infection.

Answer D. Diagnosis of acute rheumatic fever in an adult presenting with migratory arthritis and a new heart murmur requires confirmatory evidence of recent group A streptococcal infection. Explanation Pharyngeal strains of group A streptococcus (GAS) are responsible for rheumatic fever, not the skin or soft tissue strains. Prophylactic antibiotics are necessary for at least 5 years for all patients with rheumatic fever, regardless of the presence or absence of carditis. Acute rheumatic fever occurs remotely to the pharyngeal infection—which is why you can delay therapy for GAS infection pending throat culture results. Delaying therapy for a few days will not increase the risk of rheumatic fever.

A 19-year-old college student in Arkansas is studying animal husbandry. (For you non-farming folks, this involves learning about animals whose purpose is mainly to provide meat for the table and includes beef cattle, sheep, and meat goats; this is in contrast to dairy animals. Anyway, back to the question at hand.) He presents with a swollen inguinal lymph node of 3 days duration with fever. He likes to hunt and has been in the rural woods of northern Arkansas every weekend for the past 3 months. He has removed numerous ticks from his body. At one tick bite location he has a new ulceration. What is the likely organism responsible? A) Klebsiella B) Ehrlichia C) Yersinia D) Francisella E) Brucella

Answer D. Francisella Explanation He has tularemia. If you see the words "Arkansas," "tick-bite," and "lymphadenopathy," think tularemia due to Francisella tularensis. This combination of words can lead you only to tularemia infection. He certainly doesn't live in a Lyme area nor a plague area (Yersinia). Brucella would cause fever of unknown origin (FUO) and would not result in an ulcerative lesion on his body, although the goats would be a good source for this organism. Klebsiella would not do this.

A 54-year-old female with a long history of diabetes mellitus is admitted with cellulitis of her right foot. Three weeks ago, her podiatrist performed a surgical bunion removal, and on postoperative day 7, she developed erythema, drainage, and pain in the surgical site, which has worsened. She was admitted; the poorly healed wound was débrided, and deep tissue cultures were obtained. Radiographs obtained on postoperative day 21 did not reveal evidence of bone involvement, and blood cultures were negative. She was started on intravenous vancomycin and improved significantly. Cultures from the deep tissue grew Staphylococcus aureus with resistance to methicillin (MRSA) and sensitivity to clindamycin, trimethoprim/sulfamethoxazole, and levofloxacin. The laboratory performed an erythromycin-clindamycin D-zone test, and the results are reported as positive. As the patient improves, arrangements are made for outpatient completion of antibiotic therapy. Which of the following is the most appropriate outpatient antibiotic therapy? A) Oral trimethoprim/sulfamethoxazole B) Oral clindamycin C) Oral dicloxacillin D) Oral amoxicillin-clavulanate E) Oral vancomycin

Answer A. Oral trimethoprim/sulfamethoxazole Explanation Community-acquired methicillin-resistant S. aureus (MRSA) is an emerging challenge for infection control. These strains are often associated with skin and soft tissue infections, but may also cause sepsis, pneumonia, and necrotizing pathology. Clindamycin, trimethoprim/sulfamethoxazole, and quinolones are outpatient options for oral treatment. Unfortunately, community-acquired MRSA has a significant tendency to develop inducible resistance to both clindamycin and the quinolones. The erythromycin-clindamycin D-zone test evaluates a MRSA isolate for its capacity to express inducible resistance genes in the presence of clindamycin. A positive D-zone test indicates that even though an isolate appears susceptible to clindamycin in vitro , the organisms can express their resistance genes after being in the presence of clindamycin after some time. Thus, clindamycin is inappropriate in the setting of a positive D-zone test. Unfortunately, we have no such test for the quinolone drugs. Should you choose to treat a MRSA skin infection with a quinolone, you must be vigilant in observing the clinical response to ensure treatment failure does not occur. Amoxicillin-clavulanate does not cover MRSA. Vancomycin requires intravenous administration and is unnecessary for skin and soft tissue infections only (in the absence of osteomyelitis); oral vancomycin is only used for moderate-to-severe C. difficile infection. MRSA is resistant to dicloxacillin.

Which of the following infections or biological agents would prevent a mother from breastfeeding her baby for a period of time? A) Active, untreated tuberculosis B) Acute rubella infection C) Received rubella vaccine today D) Received mumps vaccine today E) Received measles vaccine today

Answer A. Active, untreated tuberculosis Explanation All of the other items listed are safe for the mother to breast-feed—even acute rubella infection! She is more likely to transmit rubella via other means than breast-feeding. Maternal HIV infection is also a contraindication for breast feeding.

A 26-year-old African-American woman with transfusion-dependent sickle cell disease presents with a 1-week history of fever, malaise, nausea, diarrhea, and cramping abdominal pain. She receives deferoxamine on a daily basis because of severely high ferritin levels. PAST MEDICAL HISTORY: Recurrent transfusions and use of deferoxamine SOCIAL HISTORY: Unremarkable FAMILY HISTORY: Unremarkable PHYSICAL EXAMINATION: BP 95/50, P 130, RR 38, Temp 101.5° F HEENT: Marked scleral icterus PERRLA, EOMI TMs clear Throat clear Neck: Supple Heart: RRR with II/VI systolic murmur at apex Lungs: Bilateral crackles in both bases with occasional faint wheeze Abdomen: Liver is 12 cm in span; no spleen palpated Extremities: No cyanosis, clubbing, or edema Which of the following organisms is the most likely to be growing in her blood cultures? A) Haemophilus influenzae B) Escherichia coli C) Francisella tularensis D) Klebsiella pneumoniae E) Yersinia enterocolitica

Answer E. Yersinia enterocolitica Explanation Certain organisms, such as Yersinia enterocolitica, are iron-loving (siderophoric) and grow better in the presence of high iron concentrations. Thus, iron overload states (from frequent transfusions or hereditary hemochromatosis) promote their virulence. Y. enterocolitica may grow in stored, packed red cells if contaminated by an infected donor. It also can harvest iron bound to deferoxamine. It causes an inflammatory diarrhea with or without the sepsis syndrome. Other sidephoric organisms include Listeria monocytogenes and Vibrio vulnificus. The former typically causes meningitis, and the latter, necrotizing fasciitis. Although E. coli can cause an inflammatory diarrhea, it is not siderophoric. The other organisms listed would not cause an inflammatory diarrhea.

A 32-year-old, otherwise healthy man is brought by ambulance to the emergency department for confusion and hypoxia. His wife called EMS after returning from work and found him febrile and confused, lying in the bed. During the morning, he reported feeling unwell and decided not to go to work, but he was not confused and looked well. At lunch, when his wife called, he was also fairly well but reported having developed a fever. He has no medical history and takes no medications. For the past 3 days, he had been keeping a bedside vigil for his brother who accompanied him on the hunting trip and succumbed to a case of "severe pneumonia" yesterday. He works as a postal worker in New Mexico, and his wife reports that he handles "suspicious packages all the time." He is an avid hunter and returned 4 days ago from a 7-day deer hunt in northern New Mexico, where they stayed in a cabin. He and his brother both killed a deer; his wife does not know whether he wears gloves while field dressing the animals. She reports he sustained several tick bites, and several rodents were noted in the cabin. He does not smoke or use drugs. Physical examination: T 103.8° F, HR 125, RR 32, BP 95/60, Pulse oximetry on room air: 85% He is cyanotic, confused, and somnolent. Nuchal rigidity is absent. Head and neck examination is normal. He is tachypneic with shallow respirations and coughing profusely with blood-tinged sputum. Coarse crackles are audible throughout. Heart rate is tachycardic but regular. Abdomen and skin exam is normal. He moves all his extremities; cranial nerves appear normal. Chest radiograph reveals diffuse, bilateral consolidations. Assuming the patient acquired his infection from his brother, which of the following is the most likely pathogen? A) Bacillus anthrax B) Clostridium botulinum C) Salmonella typhi D) Francisella tularensis E) Yersinia pestis

Answer E. Yersinia pestis Explanation Of the agents listed, only Yersinia pestis (plague) is transferred easily from person-to-person in the "pneumonic" form. Bubonic plague (infection localized to the lymph nodes) is not transmissible person-to-person, but when the organisms spread through bacteremia to the lungs, the infection becomes markedly contagious. In this case, both men visited an area where plague is known to be endemic in the United States and may have acquired the organism through flea bites or aerosol exposure to infected animal fluids. Physicians must maintain a high index of suspicion for plague pneumonia because the organism is resistant to cephalosporins, and the infection is rapidly progressive and fatal. Additionally, because of the contagiousness of the pneumonic form, respiratory isolation must be employed early. Both anthrax and tularemia have pneumonic forms, but neither infection is transmitted person-to-person. Neither botulism nor Salmonella are likely to cause pneumonia. Remember this for the Boards: Pneumonic anthrax and tularemia do not require any special precautions; plague pneumonia definitely requires respiratory isolation!

Vaccination with conjugated meningococcal vaccine is also specifically recommended for patients with which of the following diseases? A) Chronic asthma B) Cystic fibrosis C) Epilepsy D) Sickle cell disease

D. Sickle cell disease Explanation Meningococci are gram-negative diplococci with a polysaccharide capsule. Neisseria meningitides is responsible for several pathological conditions, including meningococcemia and meningococcal meningitis. Close housing conditions, such as military barracks and college dormitories, as well as travel to endemic areas (the African meningitis belt), are risk factors. As with other encapsulated organisms, asplenia is also a risk factor for meningococcal disease. The vaccine is recommended for asplenic patients, including functional asplenia resulting from sickle cell disease. Chronic asthma, cystic fibrosis, and seizure disorder are not risk factors for acquiring meningococcal disease.

A 40-year-old man with negative past medical history presents with a 1-week history of pain in his knees, wrists, and hands. He has been afebrile and has not had a rash. He lives in Massachusetts. Recently, his 12-year-old daughter had an erythematous rash on her face (particularly her cheeks) and arms. Her rash got worse if she took a warm bath or was out in the sun. She also was afebrile. His physical examination: Normal, except for his wrists and hands, which are moderately tender. No effusions of the joints are noted. He has no conjunctivitis or scleral changes on examination. Which of the following is the likely etiology? A) Parvovirus B19 B) Measles C) Neisseria gonorrhoeae D) Human herpesvirus 6 E) Borrelia burgdorferi

A. Parvovirus B19 Explanation His child had the classic "5th disease" with slapped cheeks and a serpiginous rash that worsens with heat or sun exposure. He has the most common adult manifestation, which is a symmetrical arthritis. This virus causes aplastic crises in AIDS patients or patients with sickle cell because it infects the marrow pronormoblasts and arrests red cell maturation. Human herpesvirus 6 causes roseola in kids and is responsible for pneumonia and meningitis in immunocompromised adults. He doesn't have Lyme disease even though you may have taken the bait with him being from Massachusetts. Lyme disease causes a chronic usually mono/oligoarthritis, not such an acute symmetrical arthritis. Disseminated gonorrhea is also an oligoarthritis, often with a pustular rash and uncommon in men.

A 32-year-old Peace Corps volunteer presents to your office 3 weeks after returning from central Africa, complaining of fever, malaise, headache, and mild diarrhea. He spent his last 3 months providing medical care to a small village. He has no previous medical illnesses. He does not drink alcohol and smokes half a pack of cigarettes per day. He received yellow fever and hepatitis A vaccines before the trip. He was given mefloquine for the trip to Africa, which he took weekly during his stay there. He is currently taking only acetaminophen, as needed for the fever and headache. He denies having any sexual contacts during his stay in Africa. On physical exam, he appears ill and fatigued. His blood pressure is 112/70 mmHg, pulse 88 bpm, respirations 14/min, and temperature 102.4° F (39.1° C). There is no jaundice, but scleral icterus is present. ENT exam is normal. There is no lymphadenopathy. Heart is regular with no murmur; lungs are clear; and abdominal exam is normal with no hepatosplenomegaly. There is no peripheral edema. He is lethargic, but his neurologic exam is nonfocal. There is no rash. Laboratory results: WBC: 10.7 cells/mm3 Hgb: 12.2 mg/dL Hct: 37% Platelets: 81,000 cells/mm3 Glucose: 92 mg/dL BUN: 30 mg/dL Creatinine: 1.2 mg/dL AST: 20 U/L ALT: 22 U/L Alk Phos: 100 U/L Total Bili: 3.6 mg/dL Direct Bili: 0.9 mg/dL Several peripheral smears are done and show ring-shaped organisms within erythrocytes, with some of the RBCs infected with multiple organisms. One of the peripheral smears shows the following: Which of the following is the most appropriate initial step in patient care? A) Artemether-lumefantrine B) Doxycycline alone C) Chloroquine followed by two weeks of primaquine D) Exchange transfusion E) High-dose chloroquine alone

Answer A. Artemether-lumefantrine Explanation This patient has P. falciparum malaria. Malaria should be at the top of the diagnostic list for any traveler returning from Africa with fever. The picture is showing a "banana gametocyte," classic for P. falciparum . Other diagnostic possibilities include typhoid fever, dengue fever, tuberculosis, leptospirosis, and helminthic diseases. Chloroquine is not appropriate prophylaxis for travelers to most malaria-endemic areas, because chloroquine resistance is high, including in central Africa (see maps below, taken from CDC). Some chemoprophylaxis regimens require starting medicine prior to the trip and continuing prophylaxis for up to four weeks after returning (e.g., mefloquine). Chloroquine + primaquine is the recommended treatment for P. vivax and P. ovale (primaquine must be used to eradicate the hypnozoites in the liver, which may cause recurrences months after the initial infection). Although P. vivax is the most common form of malaria worldwide, the only species of malaria that causes multiply-infected erythrocytes is P. falciparum . Thus, in this patient with multiply-infected RBCs, the diagnosis is P. falciparum , and first-line treatment is an artemisinin combination therapy, e.g., artemether-lumefantrine (Coartem®). Exchange transfusion may be used for children and for severe malaria (very high parasite load, at least > 10%); but in stable patients, it is not necessary.

A 30-year-old man will be traveling to rural areas of Africa for a year. His medical history is unremarkable. His physical examination is normal. He will be outside in the sun for many hours daily. His trip begins in 2 weeks. Based on this limited information, which of the following agents would be best for him to take for antimalarial chemoprophylaxis? A) Atovaquone/proguanil (250 mg/100 mg): 1 tablet daily beginning 1 day before departure, and continuing during travel until 1 week after return. B) Doxycycline (100 mg): 1 tablet daily beginning 1-2 days before departure, and continuing during travel until 1 week after return. C) Chloroquine 500 mg (300 mg base): 1 tablet weekly beginning 1 week before departure, and continuing during travel until 4 weeks after return. D) Mefloquine (250 mg): 1 tablet weekly beginning 2 weeks before departure, and continuing during travel until 1 week after return.

Answer A. Atovaquone/proguanil (250 mg/100 mg): 1 tablet daily beginning 1 day before departure, and continuing during travel until 1 week after return. Explanation All 4 drugs—chloroquine, mefloquine, atovaquone/proguanil, and doxycycline—are effective for chemoprophylaxis against malaria if dosed correctly and regional resistances are accounted for! Chloroquine, mefloquine, and doxycycline act on blood stages of the malaria parasites following their maturation in the liver. So, they should be continued for 4 weeks after return from an endemic area. There is widespread chloroquine-resistant malaria in West Africa. So, chloroquine should not be used for chemoprophylaxis in that area. Atovaquone/proguanil is active against both blood and tissue stages of the parasite life cycle. So, it interferes with development of actively replicating parasites in the liver and can, therefore, be discontinued 1 week after the end of exposure.

A 19-year-old college student presents with mild fever and cough. He has been coughing for several days and is not that ill. On physical examination, you note crackles in the left mid-lung fields. Which of the following is the most appropriate pharmacotherapy? A) Azithromycin B) Amoxicillin C) Trimethoprim/sulfamethoxazole D) Amoxicillin-clavulanate E) Ceftriaxone

Answer A. Azithromycin Explanation Mycoplasma or Chlamydophila pneumoniae (formerly Chlamydia pneumoniae or TWAR) is the most likely etiology of pneumonia in an adolescent who is not that ill. Of the choices, azithromycin would cover these 2 organisms well. It is possible he has pneumococcus, but that is less likely with his symptoms. Luckily, azithromycin would cover pneumococcus as well. Erythromycin or clarithromycin would also be acceptable. Amoxicillin, amoxicillin-clavulanate, ceftriaxone, and trimethoprim/sulfamethoxazole do not cover Mycoplasma or Chlamydophila .

A 39-year-old woman with a recent history of otitis media is treated with amoxicillin. She presents with a 12-hour history of severe headache, nausea, and vomiting—and is now lethargic. PAST MEDICAL HISTORY: Negative except for recent otitis 2 weeks ago; did not take all of her amoxicillin; stopped after 4 days. SOCIAL HISTORY: Former Miss USA winner Now a talk show host for local TV station Doesn't smoke Drinks a margarita on the weekend FAMILY HISTORY: Noncontributory PHYSICAL EXAMINATION: Temp: 102.6°F, P 110, RR 24, BP 130/65 mmHg HEENT: PERRLA, EOMI Disc sharp TMs clear Throat clear Neck: Mild nuchal rigidity Heart: RRR without murmurs Lungs: Scattered crackles throughout; greatest in left lower lobe Abdomen: Benign Extremities: Normal Neuro: Lethargic No papilledema Cranial nerves tested intact No focal deficits LABORATORY: WBC: 20,000 cu mm; 76% segs; 10% bands Electrolytes: Normal CSF WBC: 3000 WBC/cu mm; 95% neutrophils CSF Glucose: 20 mg/dL (serum glucose 100 mg/dL) CSF Protein: 176 mg/dL CSF Gram stain: Loaded with neutrophils and a few gram-positive, lancet-shaped diplococci Which of the following is/are the best empiric antibiotic choice(s) for her? A) Ceftriaxone and vancomycin B) Ampicillin and ceftazidime C) Penicillin G D) Vancomycin alone E) Ceftriaxone alone

Answer A. Ceftriaxone and vancomycin Explanation Based on the Gram stain, she has pneumococcal meningitis. Realize that in many parts of the United States up to 30% of pneumococci are resistant to penicillin, and up to 10% are resistant to the 3rd generation cephalosporins. Know that vancomycin and ceftriaxone are appropriate for presumed pneumococcal meningitis until the sensitivities are known. Her prior antibiotic usage increases her risk of having a beta-lactam resistant organism. Penicillin alone would be a poor choice because of the high resistance rates. Now, if they tell you the pneumococcus is sensitive to penicillin, then that would be the correct choice!

A 50-year-old nurse whom you have been following while on isoniazid (INH) prophylaxis therapy presents today for routine follow-up. She had a PPD conversion about 3 months ago. Active disease was ruled out, and she was started on INH 300 mg daily. Today, she is returning for a routine checkup and has no complaints. Her physical examination is completely normal. Laboratory testing shows a serum AST of 154 U/L (upper limit normal 40) and a serum ALT of 100 U/L (upper limit normal 56). The rest of her blood tests are normal. Which of the following choices is most appropriate at this time? A) Continue INH; repeat serum aminotransferase measurements in 1 month or sooner if clinically warranted. B) Continue INH; add pyridoxine supplementation. C) Stop INH; start rifampin instead. D) Stop INH; start ethambutol, which has little liver toxicity. E) Discontinue INH; repeat liver tests in 2 weeks; if normal, restart INH

Answer A. Continue INH; repeat serum aminotransferase measurements in 1 month or sooner if clinically warranted. Explanation Otherwise healthy individuals with no history of liver disease should be monitored monthly for symptoms of hepatotoxicity; e.g., nausea, vomiting, right upper quadrant pain, jaundice. Routine monitoring of transaminases is not recommended, and, if performed, should not be used as a reason to discontinue INH if ≤ 5 x normal. This is because approximately 20% of patients started on INH will have such transaminase elevations with no ill consequences, and switching to another regimen unnecessarily promotes resistance and removes INH as a potent treatment of active TB if it occurs. Patients with underlyting liver disease, HIV infection, pregnancy or ≤ 3 months postpartum, drink alcohol regularly, or take other medications that have potential liver toxicity effects should have transaminases monitored monthly. Addition of pyridoxine has no effect on the hepatotoxicity of INH and is indicated only in patients at high risk of peripheral neuropathy; e.g., elderly, malnourished, alcoholic, renal failure, pregnancy, or lactating.

A heterosexual couple presents for advice regarding genital herpes therapy and prevention. The woman was diagnosed with herpes genitalis 6 months earlier. She was treated with acyclovir 400 mg 3x/day for 10 days with complete resolution of the lesions. However, she experienced a recurrence in the past week. The man has never been diagnosed with genital herpes. They're in a new monogamous relationship and are both immunocompetent. Which of the following is the most appropriate next step in management? A) Continuous suppressive therapy with valacyclovir is beneficial in decreasing risk of transmission to the uninfected sexual partner. B) Prescribe valacyclovir 500 mg once daily to the male partner in order to prevent transmission of HSV-2 infection. C) Advise the couple that continuous suppressive therapy with valacyclovir decreases the severity of recurrent episodes, but not their frequency. D) Advise the couple that condom use does not significantly descrease the risk of transmission of genital HSV-2 infection. E) Advise the couple that in the absence of lesions, there is no risk of transmission.

Answer A. Continuous suppressive therapy with valacyclovir is beneficial in decreasing risk of transmission to the uninfected sexual partner. Explanation Most people who acquire genital herpes are in a new sexual relationship. To prevent transmitting the virus, individuals with genital HSV infection should abstain from sexual activity if lesions or prodromal symptoms are present. However, virus shedding continues in infected individuals, even when asymptomatic. It is estimated that up to 70% of the HSV-2 transmissions occur while the infected person is asymptomatic. Condom use decreases the risk of HSV-2 transmission by about 50%. Continued suppressive therapy with valacyclovir of symptomatic patients (500 mg qd) also significantly decreases the risk of transmission to uninfected partners. It also has been shown to decrease the frequency of recurrence and the severity of symptoms. There is no recommendation for prophylactic antiretroviral therapy for the uninfected partner.

You are scheduled to see a 54-year-old female in your clinic as an emergency department follow-up. The patient was seen for a painful, vesicular rash in a dermatomal distribution on her left chest that developed over a 6-hour period. The emergency department physician made a diagnosis of shingles and started valacyclovir. She does give a past history of chicken pox as a child. The patient is currently not working outside of the home, but regularly provides supervision of her grandchildren while their parents are at work. Which of the following is the most appropriate recommendation? A) Counsel that valacyclovir or famciclovir started in the first 48 hours of a zoster flare can reduce the duration of post-herpetic neuralgia. B) Counsel that primary infection with varicella zoster in adults usually restricts itself to a single dermatomal presentation. C) Counsel that a single dose of varicella vaccine is adequate for susceptible adults to induce immunity to a zoster infection. D) Counsel that she should avoid contact with children infected with varicella zoster during the infectious phase to reduce the risk of developing zoster.

Answer A. Counsel that valacyclovir or famciclovir started in the first 48 hours of a zoster flare can reduce the duration of post-herpetic neuralgia. Explanation Varicella zoster is responsible for chicken pox (varicella) in children, herpes zoster in adults, and infrequently, primary varicella infections in adults. Adults who experience a primary infection with varicella tend to have a much more serious course, with increased morbidity and mortality. Primary infection in adults does not present as shingles, but rather, presents as a very serious case of chicken pox with possible pneumonia and/or encephalitis. Children can contract varicella from an adult with shingles, but an adult does not contract shingles from a child with chicken pox. Shingles only results after the virus, through primary infection, establishes itself in the nerve roots, then reactivates at a later date. Dermatomal flares are consistent with reactivation of dormant varicella virus in adults who experienced the initial infection during childhood. Vaccination for susceptible patients older than 12 years of age requires two injections at least one month apart to achieve reasonable levels of immunity. With this vaccine, you are preventing primary infection with chicken pox; and, the reason to vaccinate is to prevent the serious morbidity from adult chicken pox. Valacyclovir or famciclovir started early in the course of a zoster flare can reduce the duration of post-herpetic neuralgia. These 2 agents have been shown to be superior to acyclovir. The grandchildren should be immunized against varicella and if so, they are of no risk to the grandmother. If for some reason they were not immunized, as long as her lesions can be kept covered (by a blouse for truncal lesions), then the risk is very low as well to the unimmunized grandchildren.

You are called to see a 64-year-old female patient admitted during the evening with abdominal pain, fever, and a working diagnosis of diverticulitis. Patient was started on IV fluids and antibiotics. During the night, however, she developed erythematous induration of the skin over the left lower abdomen; the area is now becoming violaceous with blister formation and has extended toward the left flank and left hypochondrial region. She is complaining of significant pain and rigors. You direct the nurse to obtain immediate blood cultures and order appropriate pain medications. Which of the following is the most appropriate next step in management to prevent morbidity? A) Emergent surgical exploration and debridement B) Immediate change of antibiotics to clindamycin, ampicillin-sulbactam, and gentamicin C) STAT contrast tomography of the abdomen D) Leading-edge aspiration or tissue biopsy for Gram stain and culture

Answer A. Emergent surgical exploration and debridement Explanation Necrotizing fasciitis is a rapidly progressive infection that tracts through the subcutaneous tissues and fascial planes. Early recognition and prompt intervention is essential to improve outcome and reduce mortality. Mixed aerobe/anaerobic infections, clostridia, and S. pyogenes are frequently implicated in the process. Clostridia infections are noted for the frequent development of gas gangrene in the affected tissues. Gas gangrene, however, is not universally present. CT scans may delineate the extent of involvement but often only shows nonspecific swelling. The presence of violaceous blisters suggests a very severe underlying deep tissue infection. Leading-edge aspiration and biopsy are often unrevealing. Surgical intervention permits necessary debridement and allows for microbiologic identification of the responsible etiologies. Antibiotics without debridement are inadequate to contain most cases of necrotizing fasciitis.

A 40-year-old man presents with a chief complaint of swelling in his groin for about a week. He lives in Missouri and is an avid hunter and fisherman. He says that he was out hunting raccoons 3 weeks ago and slept with his dog under an old oak tree. He is unsure if he received any tick bites. His appetite is depressed compared to normal, and he has had fever to 102° F (38.9° C) with some chills. On physical examination, you note that he is 5'10" and weighs 280 lbs. The examination is otherwise significant only for a large right inguinal lymph node with a large eschar just proximal to the node (see below): What is the most likely etiology for this illness? A) Francisella tularensis B) Bordetella pertussis C) Yersinia pestis D) Ehrlichia chaffeensis E) Leptospires

Answer A. Francisella tularensis Explanation Combine Missouri, tick exposures, lymphadenopathy, fever, eschar, and a person who frequents the woods, and tularemia is the most likely answer. Yersinia would be found in the desert Southwest. There are 14 different species of leptospires that cause disease; infection with any of them would present with fever, headache, and, (frequently tested on the Boards) conjunctival suffusion and an obstructed jaundice picture. Ehrlichia is a possibility in this geographic area, but it does not cause a single lymph node enlargement, and you would expect the patient to present a CBC showing pancytopenia. Bordetella pertussis causes pertussis, a respiratory disease that has nothing to do with this presentation.

A 38-year-old man with AIDS (CD4 count is 20) presents for evaluation. He resides in Indiana and does not take any anti-HIV medications ("They make me sick"). He presents with fever and chills of several days duration. He notes that he has been weak and cannot walk up a flight of stairs without getting short of breath. Physical examination shows hepatosplenomegaly and a palatal ulcer. Laboratory is significant for pancytopenia with a WBC of 1,500, with hemoglobin of 7.0 mg/dL, and platelet count of 110,000. Which of the following opportunistic infections is most likely the etiology? A) Histoplasmosis B) Coccidioidomycosis C) Cryptococcosis D) Blastomycosis E) Tuberculosis

Answer A. Histoplasmosis Explanation The findings of AIDS with low CD4 count and pancytopenia should make you think of Histoplasma in the Mississippi River valley. Hepatosplenomegaly is common, and a palatal ulcer is classic for Histoplasma! The other fungi less commonly present this way, and the geographic location is not consistent with Coccidioidomycosis (would be seen in the desert Southwest). Disseminated TB can present with all of the features, but a palatal ulcer would be very uncommon.

A 40-year-old woman from Rhode Island presents to you with an isolated Bell's palsy. She awakened yesterday with this and now reports the weakness. She denies fever, headache, or other problems associated with this. She denies history of rash. She frequently goes hiking in the rural neighboring states but denies tick bite or other abnormality. Based on this, which of the following is the most likely etiology for her palsy? A) Lyme disease B) Multiple sclerosis C) Parvovirus B-19 D) Illicit Botox® use E) Group A streptococcus immune response

Answer A. Lyme disease Explanation This is a classic presentation for Lyme with an isolated Bell palsy. Note that most people do not have a history of tick bite or erythema migrans. If on a Board question they had given you the rash of erythema migrans, then it would have been even more straightforward—so think about it especially if they tell you someone has a "roundish" rash, etc. Treatment is with oral doxycycline and does not require intravenous therapy.

A 35-year-old man with Type 1 diabetes presents to the emergency stuporous and is found to have diabetic ketoacidosis. Oral exam reveals a dark black ulcer on his hard palate. Which of the following is the likely diagnosis? A) Mucormycosis B) Aspergillosis C) Blastomycosis D) Sporotrichosis E) Cryptococcosis

Answer A. Mucormycosis Explanation The classic "black eschar" is seen in poorly controlled insulin-dependent diabetics and requires aggressive therapy with antifungal therapy and debridement. Prognosis is very poor.

You are asked to evaluate an 85-year-old female resident of an extended care facility who suffers from marked dementia. Nursing staff reports two nights of marked irritability, poor sleep, and complaints of rectal irritation. She has had no fever, diarrhea, or reported trauma. The patient is alert and is in no apparent distress at the time of examination. Her vitals are within normal limits. The rectal exam is remarkable for significant perirectal irritation with mild erythema and excoriations with thickening of the perirectal skin. The involvement of the skin is limited to the gluteal fold region and is not painful to palpation. No palpable masses or "discharge" is noted. No fissures are noted, and the external vaginal area is unremarkable. A "scotch tape" test is done and shows the following: eggs Which of the following is the most appropriate pharmacotherapy? A) Oral albendazole B) Topical permethrin C) Topical nystatin D) Oral cephalexin E) Topical hydrocortisone

Answer A. Oral albendazole Explanation This presentation is typical for infection with Enterobius vermicularis (pinworms). Pinworms are a parasitic infection, with humans as the only natural host. The females perpetuate the life cycle by laying eggs in the perianal area. Irritation caused by the worms stimulates scratching, which transfers the eggs to the fingers and then to the mouth to continue the GI infestation. The "scotch tape" test involves placing a piece of scotch tape on the anus, then removing it and looking for the eggs stuck on the scotch tape under the microscope. Albendazole is an effective treatment for this disease, but may require repeat administrations to fully eradicate the pinworms. Mebendazole (Vermox®) was an effective therapy, but the drug has stopped being produced by the company. Nystatin is used for topical candidiasis; cephalexin would be used for bacterial cellulitis; and permethrin is used for a scabies or lice infestation. Hydrocortisone may alleviate many of the local symptoms but would not affect the fundamental process.

A 30-year-old woman with negative past medical history presents to you with a 1-week history of pain in her wrists and hands. She has been afebrile and has not had a rash. She lives in a city complex and does not go into wooded areas. She has no pets and has not had any tick bites. Recently, her 8-year-old son had an serpiginous rash on his face and arms. His rash got worse when he took a bath or was out in the sun. He also was afebrile. Everything resolved in 4 days. PHYSICAL EXAMINATION: Essentially normal except for her wrists and hands, which are moderately tender. No effusions of the joints are noted. She has no conjunctivitis or scleral changes on examination. Which of the following is the likely etiology for the symptoms in these two patients? A) Parvovirus B19 B) Neisseria gonorrhoeae C) Borrelia burgdorferi D) Human herpesvirus 6 E) Measles virus

Answer A. Parvovirus B19 Explanation Her child had the classic childhood presentation of parvovirus B19 "fifth disease," with slapped cheeks and a serpiginous rash that worsens with heat or sun exposure. His mother has the classic adult manifestation with arthritis and joint findings. In immunompetent patients, it is a self-limited illness. There are only 3 populations that run into more trouble. Pregnant women have a higher risk of fetal loss and hydrops fetalis. HIV-infected persons develop a chronic red cell aplasia treatable by IVIG, and persons with chronic hemolysis (e.g., sickle cell disease) develop an acute red cell aplasia that, if supported with transfusions, resolves. Human herpesvirus 6 causes roseola in kids and may be responsible for pneumonia and meningitis in immunocompromised adults. Lyme disease starts with erythema migrans in the majority of persons infected with Borrelia burgdorferi, and the late arthritis is usually an asymmetric mono- or oligoarthritis. Disseminated gonorrhea would not be present in both a child and a mother.

A 50-year-old man presents for evaluation. He spent the night camping with a friend in a wooded community of rural New York state. During a shower following his return home, he discovered a brownish lump in the skin that he believed was a tick. He clearly states that the lump was not present in the morning when he was dressing. On physical examination, a tick is noted and removed from the left calf. What is the best treatment option at this time? A) Patient education and observation B) Cephalexin for 14 days C) Amoxicillin for 30 days D) Doxycycline for 14 days E) IV ceftriaxone for 7 days

Answer A. Patient education and observation Explanation Ticks are vectors for the transmission of the spirochete Borrelia burgdorferi , the pathologic agent of Lyme disease. The spirochete lays dormant as cysts in the midgut of the tick and is reactivated at the ingestion of a blood meal by the tick. After reactivation, the spirochete travels through the tick's circulation to the salivary glands where it is transmitted to the new host. This process typically requires 24-48 hours of active feeding by the tick before the spirochete can be transmitted. In this case, there is virtually no risk of infection of the patient. For this reason no treatment is indicated. Doxycycline is the drug of choice for adults and children older than 9 years, but would not be appropriate for an 8-year-old. Second- and third-generation cephalosporins are active in Lyme disease, but first-generation cephalosporins, such as cephalexin, do not provide adequate coverage. Amoxicillin is the drug of choice in younger children, but a 30-day course is unnecessary. IV ceftriaxone is indicated for CNS or cardiac disease, but the duration is 14-28 days.

A 29-year-old woman resides in Brownsville, Texas. She has been noted to have a liver abscess that is felt to likely be due to Entamoeba histolytica. What is the best way to determine if this is the likely etiology? A) Serology testing. B) Stool ova and parasite examination. C) Stool culture. D) String test. E) Aspirate the liver abscess.

Answer A. Serology testing. Explanation For amebic liver abscesses, the best way to diagnose these is by serology. Stool culture would never show ameba, and ova and parasite is likely to be positive only in intestinal disease. Aspiration of the abscess could be dangerous but also is unlikely to show any amoebas and only PMNs. The string test is done for Giardia and today is rarely used because of better stool antigen tests for this organism.

A 26-year-old man presents with a 2-day history of pain in his proximal right arm associated with decreased range of motion and subjective fever. On physical examination, two 3 x 3.5-cm tender, slightly warm, right axillary lymph nodes are identified. Which of the following additional findings is most likely to be identified if his clinical findings are the result of infection caused by Bartonella henselae ? A) Several cutaneous papules B) Multiple small retinal hemorrhages associated with a whitish-pale center C) Painful erythematous raised lesions on the palms and soles D) Thickened yellowed nail plates E) Nontender, small erythematous lesions on the palms or soles

Answer A. Several cutaneous papules Explanation Bartonella henselae , a curved, pleomorphic, gram-negative bacillus, is responsible for cat-scratch disease. This disease is characterized by tender regional lymphadenopathy and erythematous tender papules following a kitten or, less often, a cat scratch. Following an incubation period of 3-14 days, one or several cutaneous papules or pustules develop at the inoculation site. The lesions persist for 1-3 weeks but often go unnoticed. As they resolve, regional lymphadenopathy develops proximal to the inoculation site. Painful and often suppurative lymphadenopathy most often involves the axillary nodes, followed in frequency by the cervical, and then the inguinal lymph nodes. Constitutional symptoms are more often than not mild and may include generalized malaise, low-grade fever, anorexia, nausea, headache, and fatigue. Complications may include encephalitis, osteomyelitis, neuroretinitis, pneumonitis, transverse myelitis, and arthralgia/arthritis. Antibiotics are not required in the majority of cases. Although aspiration of fluctuant nodes will relieve pain, incision and drainage does not hasten recovery and should be avoided because of residual scaring and draining fistulae. Thickened yellow nail plates describes onychomycosis; retinal hemorrhages with whitish-pale center describes Roth spots; nontender erythematous lesions on the palms and soles describes Janeway lesions; and painful lesions on the palms and soles describes Osler nodes. These latter 3 findings are all associated with bacterial endocarditis.

A 20-year-old woman presents to the emergency department with septic shock. She was found by her boyfriend this morning sprawled on the kitchen floor and unresponsive. He reports she was well yesterday. You note that her skin appears red. She has fever and hypotension. Her boyfriend noted that she began having diarrhea last night and that she had started her menses yesterday. Laboratory results reveal pancytopenia, markedly elevated liver enzymes, and a creatinine of 2.3 mg/dL. Which microorganism is most likely responsible for her acute presentation? A) Staphylococcus aureus B) Streptococcus pneumoniae C) E. coli O157:H7 D) Streptococcus agalactiae E) Neisseria meningitidis

Answer A. Staphylococcus aureus Explanation She meets criteria for a diagnosis of toxic shock syndrome due to S. aureus . Note the question mentions "menses." If you see menses mentioned in a question with possible infection, think of two scenarios: 1) toxic shock syndrome and 2) disseminated gonococcal infection. E. coli O157:H7, which can cause a diarrheal illness with rare instances of hemolytic uremic syndrome; but the Boards will give you an adequate exposure history to raw hamburger meat or petting zoos. E. coli O157:H7 infections also usually occur in an outbreak, and several infected patients would be included in the question. S. pneumoniae can cause sepsis, but she has no upper respiratory symptoms; thus, this is much less likely. N. meningitidis is possible, but the "menses" reference infers she was possibly using tampons and, therefore, more at risk for toxic shock syndrome. If, instead, the question presented a freshman college student living in a dormitory, you should then suspect meningococcus because of the known increased risk of invasive meningococcal disease with young people living in close proximity (military or college dormitories). Finally, S. agalactiae , or group B streptococcus, is mainly a problem (especially on Board exams) of newborns, pregnant women, and diabetics.

A 25-year-old man comes to the emergency department complaining of weakness in both legs. He reports that he started to have a tingling sensation in his toes about a day ago along with a slight bilateral foot-drop sensation. On awakening this morning, he had problems grasping objects and noted some weakness in his upper legs. He has had no fever, diplopia, dysphagia, or dyspnea. He had a "cold" 1 week prior to his current symptoms. He removed a tick from his waist about a week and a half ago. He eats homegrown vegetables and fruits that his mother cans for him back in West Virginia. PAST MEDICAL HISTORY: Negative Immunizations up to date SOCIAL HISTORY: Works as a truck driver in the Washington, DC area Smokes 2 packs/day of cigarettes Doesn't drink alcohol FAMILY HISTORY: Negative PHYSICAL EXAMINATION: BP 120/76, RR 18, Temp 97.9° F, P 86 HEENT: PERRLA, EOMI Discs sharp TMs clear Throat clear Neck: Supple Heart: RRR with I/VI systolic flow murmur Lungs: CTA Abdomen: Bowel sounds present; no hepatosplenomegaly Extremities: No cyanosis, clubbing, or edema Neuro: Symmetrical weakness of lower extremities—distal muscles more affected than proximal muscles Bilateral foot drop Weakness of both hands noted Cranial nerves II-XII tested and intact Sensory perception is slightly decreased in the distal lower legs Patellar and Achilles reflexes are absent bilaterally Which of the following is the most likely diagnosis? A) Rabies B) Guillain-Barré syndrome C) Poliomyelitis D) Botulism (foodborne) E) Tick paralysis

Answer B. Guillain-Barré syndrome Explanation Guillain-Barré is the most common acute inflammatory polyneuropathy. It frequently is preceded by an upper respiratory infection and has been associated with Campylobacter jejuni infections—as well as EBV, CMV, HIV, mycoplasma, and Lyme disease. Symptoms include an ascending paralysis and areflexia caused by a segmental demyelination of the peripheral nerves. CSF examination is not necessary to make the diagnosis but may show an elevated protein with a disproportionately smaller increase in CSF WBCs. Nerve conduction studies show slowed conduction due to demyelination. Plasmapheresis is useful if it is begun early in the course. Botulism causes a descending paralysis and begins with cranial nerve weakness manifested by dysphonia, dysphagia, diplopia, and blurred vision; this is followed by the muscle weakness and respiratory insufficiency. Poliomyelitis presents as meningeal symptoms, fever, and asymmetric paralysis. Tick paralysis occurs while the tick is attached and is due to a neurotoxin. It causes an ascending flaccid paralysis and ataxia. Rabies causes an encephalitis characterized by fever, hydrophobia, and agitation progressing to coma and death. It does not have the peripheral nerve findings depicted in this case presentation.

A 45-year-old man lives in Nevada and comes in with a chief complaint of urinary frequency and burning. He has never had these symptoms. He has had fever for 2 days and now has chills. PAST MEDICAL HISTORY: Negative SOCIAL HISTORY: Smokes 2 packs a day Drinks 2 whiskey shots daily FAMILY HISTORY: Mother died of alcoholic liver disease at age 50 Father died of alcoholic liver disease at age 50 Brother died of alcoholic liver disease at age 50 Sister died of alcoholic liver disease at age 50 Cousin died of alcoholic liver disease at age 50 PHYSICAL EXAMINATION: BP 90/70, RR 25, Temp 104° F, P 120 General: Severely ill-appearing man in some distress HEENT: PERRLA, EOMI TMs clear Throat clear Neck: Supple Heart: RRR without murmurs, rubs, or gallops Lungs: CTA Abdomen: Bowel sounds present, liver span 16 cm Extremities: Trace pedal edema GU: No lesions Suprapubic tenderness noted Rectal exam heme-negative; nontender prostate LABORATORY: Blood and urine cultures positive for E. coli; sensitive to ciprofloxacin, amikacin, and piperacillin only You start therapy with intravenous ciprofloxacin, and he responds nicely. It is day 5 of admission, and you are ready to send him home to complete oral antibiotics. Which of the following agents should you not use with his ciprofloxacin? A) Sucralfate B) Cimetidine C) Ranitidine D) Disulfiram E) Loratadine

Answer A. Sucralfate Explanation This agent will substantially interfere with the absorption of the oral drug, resulting in subtherapeutic serum and urine levels. Other agents to avoid that also will do this include antacids that contain magnesium, aluminum, or calcium. Theophylline is another agent to avoid, because concurrent ciprofloxacin will cause increases in theophylline levels and possible theophylline toxicity.

Marla is a 28-year-old woman who steps on a dirty nail in a cow pasture. Her pet iguana licks the wound. Her last tetanus immunization (Td) was 6 years ago. Before that she received the usual immunization regiment as a child. Which of the following does Marla require today? A) Tdap only B) Td and tetanus immune globulin C) Td only D) Tdap and tetanus immune globulin E) DTaP and tetanus immune globulin

Answer A. Tdap only Explanation She definitely has experienced a dirty wound and requires tetanus prophylaxis if she is not up-to-date. Since it has been more than 5 years (for dirty wounds), she requires a booster immunization. Since she has not received a Tdap yet, she should receive one today. If she had received a Tdap before and needed a booster, then a Td would be fine. Today, everyone needing a tetanus booster between the ages of 10 and 64 should receive a Tdap if they have not received one to help protect against pertussis as well. Currently just 1 Tdap is recommended in a lifetime (this may change over time). She does not require immune globulin because she has had more than 3 tetanus immunizations in the past.

A short and to-the-point question: You diagnose C. difficile (non-severe) diarrhea in a patient. You treat the patient with oral metronidazole and the patient's diarrhea resolves. The patient returns 3 weeks later with diarrhea again due to C. difficile. Which of the following is the best next plan? A) Treat with oral metronidazole. B) Do not treat with antibiotics. C) Treat with oral fidaxomicin. D) Treat with oral vancomycin

Answer A. Treat with oral metronidazole. Explanation In 2010, the Infectious Diseases Society of America (IDSA) updated the recommendations for the treatment of C. difficile infection. Mild-to-moderate disease should be treated with metronidazole and severe disease with vancomycin. With either therapy, approximately 25% will relapse. The IDSA recommends an initial relapse that is mild-to-moderate be treated again with metronidazole. Vancomycin does not have a higher cure rate than metronidazole, and a 10-day course is exceedingly more expensive than metronidazole (~ $1,000 vs. ~ $20). Fidaxomicin was FDA-approved in 2011 for the treatment of C. difficile infection. It has a similar cure rate as that of vancomycin but a 50% reduction in relapses. It may be considered in patients with frequent relapses, but it is not indicated for use after only 1 relapse as in this case. It costs ~ $3,000 for a 10-day course.

A 60-year-old nurse presents with new onset of fever and chills. 10 days ago, she developed flu-like symptoms of fever, runny nose, cough, and severe myalgias. An influenza test was performed and was positive. The fever lasted seven days then remitted, and she began improving. However, over the past 24 hours, she has developed a new cough with left-sided pleuritic chest pain. She appears in mild distress. Chest radiograph reveals LLL consolidation. Blood cultures grow the following organism: What is the most appropriate pharmacotherapy? A) Vancomycin B) Piperacillin-tazobactam C) Oral amoxicillin-clavulanic acid (Augmentin®) D) Ceftriaxone E) IV trimethoprim/sulfamethoxazole

Answer A. Vancomycin Explanation With her history of influenza, she is at high-risk for a secondary bacterial infection, specifically pneumonia, from pneumococcus or Staphylococcus aureus . Her blood culture, however, suggests S. aureus given the "grape-clustered" morphology; S. pneumoniae would have diplococcal morphology. Of the options listed, vancomycin is the more directed therapy, especially with her increased risk of MRSA because she is a health care worker (although rates of MRSA in many communities are rising rapidly as well). Piperacillin-tazobactam would also be effective for MSSA, but is much too broad in spectrum. IV ceftriaxone is not appropriate since it lacks coverage for S. aureus . IV trimethoprim/sulfamethoxazole is not a good choice for this infection but would be the drug of choice if Pneumocystis were a concern. Finally, oral amoxicillin-clavulanic acid is not appropriate for bacteremia—it would be fine for a localized skin infection but not a serious life-threatening pneumonia or bacteremia.

The daughter of a divorced 36-year-old woman brings her to your office because she has been "acting strange" for the past week. The family has noted that she has been sleeping less, is "always jittery," has begun to forget things (recent events, names); and today, she placed her boots in the refrigerator. She has no history of chronic medical disease or mental illness. She takes no medications, does not smoke cigarettes, drinks a glass of wine 3-4 times a week, and has never done illicit drugs. She has had 6 sexual partners in the past two years. ROS is significant only for subjective fevers for the past few days. On physical exam, vital signs are normal, and the exam is nonfocal with the exception of the neurologic exam. The patient appears manic; mini-mental status exam score is 24/30. Cranial nerves are intact. Motor and sensory exam are normal. Gait is slightly ataxic; she is unwilling to perform finger-to-nose testing. Reflexes are 2+ and symmetric. Basic laboratory testing is unremarkable, including complete blood counts and metabolic panel. MRI of the brain is performed. There is abnormal signal intensity in the left temporal lobe with slight mass effect. Which of the following is the most likely diagnosis? A) Brain abscess B) HSV encephalitis C) Temporal lobe epilepsy D) Glioblastoma multiforme E) Schizoaffective disorder

Answer B. HSV encephalitis Explanation This patient's symptoms are typical of temporal lobe involvement due to infection with herpes simplex virus (HSV). Symptoms usually are acute in onset, associated with fever, and may include unusual behavior, seizures, dysphasia, amnesia, cranial nerve deficits, and ataxia. Abnormal signal enhancement in the temporal lobe is very suggestive of HSV encephalitis in the proper clinical setting. Temporal lobe epilepsy presents with automatisms, staring episodes, and altered consciousness, but imaging would not show enhancement. Patients with a brain abscess have a more indolent clinical course, complaining of symptoms related to mass effect, and the MRI would show an enhancing focal lesion. Glioblastoma multiforme appears as an irregularly shaped mass with central necrosis and extensive edema—a completely different appearance from HSV encephalitis. Although patients with HSV encephalitis may exhibit signs of hypomania and unusual behavior, this patient does not fit the criteria for schizoaffective disorder.

A 21-year-old male with cystic fibrosis presents to your office with fever to 103° F (39.4° C), shortness of breath, and increased purulent sputum production. He has chills and appears to be quite ill. He has been hospitalized three times in the last 3 years for pneumonia exacerbation. In the past, MRSA and various strains of Pseudomonas were identified on bronchoscopy cultures. Physical exam reveals diffuse coarse breath sounds throughout. Chest radiograph reveals chronic scarring but is unchanged from his last radiograph. Sputum Gram stain reveals gram-positive cocci in clusters and large, gram-negative rods. Cultures are pending. Which of the following is the most appropriate pharmacotherapy? A) Vancomycin, tobramycin, and ceftazidime. B) None, because the organisms on Gram stain likely reflect colonization. C) Ciprofloxacin. D) Vancomycin, ceftriaxone, and tobramycin. E) Ampicillin and tobramycin.

Answer A. Vancomycin, tobramycin, and ceftazidime. Explanation He is quite ill-appearing and definitely has infection (fever, increased purulent sputum production), so "none" is not correct. They tell you that he has had MRSA and various Pseudomonas infections in the past (presence of these organisms from a bronchoscopy specimen implicates them as pathogens as opposed to simply colonizers of the upper airways). With his current probable pneumonia, you have to suspect that he is infected with either or both of these. Therefore, you need to include coverage for both in your empiric regimen. Vancomycin, remember, is the only antibiotic listed effective against serious MRSA, so it is required in the regimen. That leaves us with vancomycin + ceftriaxone + tobramycin and vancomycin + tobramycin + ceftazidime. Ceftriaxone is ineffective against Pseudomonas ; therefore, the only choice here is vancomycin, tobramycin, and ceftazidime.

A 38-year-old-man from rural Missouri presents with a four-day history of fever, mild sore throat, and fatigue. He enjoys hiking in the Ozark Mountains of southern Missouri and denies knowledge of tick bites but has seen a few ticks on his dog. He enjoys swimming with his dog in a local pond. He denies lymphadenopathy and other symptoms. He reports no rash. Physical examination is unremarkable except for mild hepatosplenomegaly. Laboratory is done and shows a WBC of 1,500 with 60% lymphs, 10% bands, and 20% neutrophils. He is anemic with a hemoglobin of 10 mg/dL, and his platelet count is 55,000. Which of the following is the most appropriate study to diagnose the infection causing his illness? A) Skin test B) Acute and convalescent antibody titers C) Urine culture D) Lymph node biopsy E) Blood culture

Answer B. Acute and convalescent antibody titers Explanation He has ehrlichiosis. This is an outdoor person in Missouri with fever, fatigue, and pancytopenia without significant lymphadenopathy. He does not have jaundice. Lymph node biopsy would be useful only in the case of Yersinia, which he does not have any risk factors for (desert Southwest, rats, etc.). Skin testing is good only for tuberculosis. Blood culture would be good for the bacteremic phase of leptospirosis (the swimming in the pond with his dog history), but the pancytopenia and lack of jaundice go against this. Urine culture would be useful for the immune phase of leptospirosis, but again, lack of jaundice is helpful in ruling this out. There is really not anything else that will give you the combination of: Missouri, pancytopenia, and possible tick exposure, except for ehrlichiosis.

A 40-year-old man with acute myeloid leukemia is admitted with fever. His absolute neutrophil count is 200. He is begun on cefepime but remains febrile on day 7 with no localizing signs of infection. Which of the following changes in the antibiotic therapy would you do? A) Add vancomycin. B) Add caspofungin. C) Stop cefepime and observe off antibiotics. D) Add ciprofloxacin. E) No changes, continue the cefepime.

Answer B. Add caspofungin. Explanation He has febrile neutropenia and has been on an appropriate antibacterial agent for more than 5 days and continues to have fever. The latest IDSA guidelines recommend adding caspofungin for antifungal coverage after 5-7 days of fever and neutropenia. A resistant bacterial infection would likely have caused more clinical deterioration so adding an additional antibacterial is not indicated. Stopping antibiotics runs the risk of overwhelming sepsis from a suppressed bacterial infection.

An 18-year-old man lives in rural Arkansas. He is not a hunter. He works on a farm and has numerous animals, including rabbits, chicks, ducks, chinchillas, and sheep. He presents with a 2-week history of fever, night sweats, and malaise. He has noted a painful swelling in his right axilla for the past week. He had some "leftover" dicloxacillin and started taking it about 3 days ago but has not improved. PAST MEDICAL HISTORY: Negative; 1st visit to the doctor since he was 12 for a broken collarbone Due for Tdap now SOCIAL HISTORY: As above Lives on the farm with his 2 brothers Doesn't drink or smoke Chews tobacco PHYSICAL EXAMINATION: BP 110/70, P 110, RR 18, Temp 103° F HEENT: PERRLA, EOMI TMs clear Throat clear Neck: Supple Heart: RRR without murmurs, rubs, or gallops Lungs: CTA Abdomen: No hepatosplenomegaly Extremities: No cyanosis, clubbing, or edema 4 x 3-cm left axillary lymph node; tender No skin lesions LABORATORY: WBC: 17,000 with left shift To confirm your diagnosis, which of the following is the best course? A) Febrile agglutinins B) Assay for acute and convalescent titers C) Biopsy of lymph node D) Culture of lymph node aspirate E) Gram stain of lymph node aspirate

Answer B. Assay for acute and convalescent titers Explanation This patient has tularemia. To diagnose tularemia, which is due to Francisella tularensis, you usually will do acute and convalescent serum titers. You do not want to biopsy or aspirate a lymph node with tularemia. You are putting yourself and the lab at risk for aerosolization of the organism. Febrile agglutinins are worthless. Treatment of this man would be with IM streptomycin, IV gentamicin, or oral doxycycline. He doesn't appear to be that ill, so oral doxycycline might be worth a trial. There is a higher risk of relapse with oral doxycycline, though.

A 50-year-old IV drug abuser presents with new-onset murmur and fever. Blood cultures grow Staphylococcus aureus in 4 out of 4 bottles. What is the most common cause of cardiac death in people with her diagnosis? A) Arrhythmia B) Heart failure C) Thromboembolism to the lung D) Thromboembolism to the brain E) Myocardial infarction

Answer B. Heart failure Explanation Although all of these are potential complications of infective endocarditis, heart failure due to progressive valvular destruction is the most common cause of cardiac death in this condition. Embolic phenomena to the brain (in left-sided disease) or lung (in right-sided disease) may occur but are less commonly fatal. Erosion of infection into the conduction system is a late complication that can cause heart block, but is a rare cause of cardiac death. Also rare are emboli from left-sided disease directly into the coronary arteries causing myocardial infarction.

A 23-year-old woman who is in her 2nd month of pregnancy is referred to you by her obstetrician for evaluation of the following laboratory values. She feels well and has never had jaundice that she knows of. LABORATORY: AST 19 U/L ALT 25 U/L Serum alkaline phosphatase 110 U/L Serum total bilirubin 0.5 mg/dL Hepatitis B surface antigen (HBsAg): Positive Antibody to hepatitis B surface antigen (anti-HBs): Negative IgM antibody to hepatitis B core antigen (IgM anti-HBc): Negative Hepatitis B e antigen (HBeAg): Negative IgG antibody to hepatitis B core antigen (IgG anti-HBc): Positive The newborn of this woman should receive which of the following? A) Hepatitis B immunoglobulin at birth; then hepatitis B vaccine at 2 months of age B) Both hepatitis B vaccine and hepatitis B immunoglobulin at birth C) Alpha-interferon and ribavirin D) No vaccine until 2 months of age E) Hepatitis B vaccine only

Answer B. Both hepatitis B vaccine and hepatitis B immunoglobulin at birth Explanation The mother is most likely a carrier of hepatitis B infection. She will pass this to her newborn infant at delivery with 90% efficiency if there is no intervention; therefore, the child should receive both hepatitis B immunoglobulin as well as vaccine within the first 12 hours of birth! They should be given in 2 separate sites because it is useful for any instance in which you are giving an "immunoglobulin" type product and the vaccine concurrently. Otherwise, the immunoglobulin will "bind up" the vaccine and prevent an immunologic response from occurring in the recipient of the vaccine. Note that this scenario would also work for someone who was exposed to a person with hepatitis B—as with a sexual exposure or a needlestick exposure. Note also that if the exposed person has received vaccine and is known to be immune, you don't have to do anything because that is the purpose of the vaccine—it will protect the exposed person.

A 45-year-old woman with acute myelogenous leukemia is undergoing chemotherapy. She is severely neutropenic and has now spiked a fever to 102° F. Physical exam reveals no focus of infection. Which of the following antibiotics is the best choice for her? A) Ceftriaxone B) Cefepime C) Vancomycin and cefepime D) Vancomycin and metronidazole E) Vancomycin and ceftriaxone

Answer B. Cefepime Explanation The current IDSA guidelines for the treatment of febrile neutropenia recommend monotherapy with a broad spectrum beta-lactam such as imipenem, cefepime, or piperacillin/tazobactam. This is to treat for gram-negative aerobic organisms, specifically Pseudomonas aeruginosa. Ceftriaxone does not cover this organism. Empiric vancomycin is only indicated if there is suspected line sepsis, soft tissue infection, prior colonization with MRSA, or resistant pneumococci or severe sepsis. Anaerobes are rare causes of febrile neutropenia.

A 19-year-old chicken farmer comes to your office with a 2-week history of non-productive cough with low-grade fever. Additionally, he has had sore throat and hoarseness associated with these symptoms. He has not had diarrhea, rigors, sweats, or chills. He owns a pet cockatoo and several parakeets. He is sexually active and uses condoms with every episode of intercourse. He denies IV-drug use or other risk factors for HIV. He does not smoke. PAST MEDICAL HISTORY: Negative SOCIAL HISTORY: On further questioning, he admits to smoking marijuana in the past—but says he never inhaled. Lives alone on his farm. PHYSICAL EXAMINATION: Vitals are normal except for a temperature of 99.8° F HEENT: PERRLA, EOMI Sclera anicteric Throat: Red and inflamed; no exudates Neck: Supple Heart: RRR without murmurs, rubs, or gallops Lungs: Fine crackles heard at the right lung base Abdomen: Bowel sounds present No hepatosplenomegaly Extremities: No cyanosis, clubbing, or edema Skin: No rash LABORATORY: Leukocyte count: 12,000/cu mm; 60% neutrophils, 25% lymphocytes, 10% monocytes, 2% eosinophils ESR: 57 mm/hr TB Skin test: 0 mm CXR: Patchy infiltrate in the right lower lobe Which of the following organisms is most likely to be causing his illness? A) Legionella pneumoniae B) Chlamydophila pneumoniae C) Streptococcus pneumoniae D) Chlamydophila psittaci E) Coxiella burnetii

Answer B. Chlamydophila pneumoniae Explanation The constellation of symptoms—chronic non-productive cough, low-grade fever, hyperemic sore throat, hoarseness—fit this infection. C. psittaci is a concern, but he has no hepatosplenomegaly; plus, this most commonly presents as fever of abrupt onset, pronounced headache, and dry cough in a patient with a recent history of bird exposure. Systemic manifestations are prominent with fever in all patients, rigors occurring in a majority, with sweats and myalgias in most patients. A combination of pneumonia with splenomegaly: Think C. psittaci or Coxiella burnetii! Also think about Coxiella if the case notes he delivered cows or some livestock. Recently, cats have been implicated with this infection—but there usually has to be contact with placentas for this. S. pneumoniae is unlikely to produce this prolonged gradual infection, and the patchy infiltrate goes against it somewhat. In addition, the case kept saying he had no chills—if you see "chills" and "rusty sputum," think of S. pneumoniae.

A 28-year-old woman has a history of AIDS for the past 3 years. She presents to you with a 1-week history of "floaters" in her right eye. She came today because she noted blurred vision in that eye since awakening this morning. PAST MEDICAL HISTORY: Diagnosed with HIV 3 years ago with PCP presentation; currently on zidovudine, lamivudine, efavirenz, and trimethoprim/sulfamethoxazole prophylaxis; has not had any problems since her diagnosis 3 years ago SOCIAL HISTORY: Works as a cafeteria worker at the local elementary school Former boyfriend was an IV-drug abuser PHYSICAL EXAMINATION: BP 110/70, P 80, RR 18, Temp 98.8° F HEENT: PERRLA, EOMI Discs: Normal on undilated examination Central visual acuity is preserved Anterior chamber appears clear Heart: RRR without murmurs, rubs, or gallops Lungs: CTA Abdomen: Bowel sounds present; no hepatosplenomegaly LABORATORY: CD4: 30/cu mm Viral load 50,000 copies/mL A photo taken by the ophthalmologist is shown in this figure: Which of the following is the most likely diagnosis? A) HIV retinopathy B) Cytomegalovirus retinitis C) Pneumocystis jiroveci retinitis D) Neurosyphilis E) Toxoplasma chorioretinitis

Answer B. Cytomegalovirus retinitis Explanation CMV retinitis is the most common eye disorder to occur in patients with CD4 counts below 50. Patients will frequently initially complain of floaters, and they will gradually lose vision. This patient has classic findings on ophthalmologic examination: attenuation of vessels in the area affected by retinitis. Retinal lesions will occur as 2 possible presentations. In the posterior type, large areas of thick white infiltrate are accompanied by retinal hemorrhage, with a distribution along retinal vessels. The peripheral type demonstrates granular retinitis with satellite lesions and less hemorrhage. Behind the advancing border is necrotic retina. Treatment is with oral valganciclovir unless she has sight-threatening lesions; then intravitreal injection/implant ganciclovir or IV ganciclovir is generally used. Also, it is important to get her HIV under better control and, with better anti-HIV therapy, raise her CD4. This would be very helpful in controlling her disease.

79-year-old man has a right hip prosthesis. Ten months after his prosthesis is placed, he complains of persistent pain that has not improved since his surgery. He has tried ibuprofen without relief. For the last few weeks, the pain has worsened to the point that he will not get out of bed unless his grandchild helps him. He has not had any fevers during this time. PAST MEDICAL HISTORY: Lung cancer diagnosed 20 years ago; had resection and cure Hypothyroidism for 30 years; on replacement therapy Hypertension for 50 years; on ACE inhibitor for 10 years with good control SOCIAL HISTORY: Lives with his granddaughter and her 3 kids Doesn't smoke or drink PHYSICAL EXAMINATION: BP 130/70, P 80, RR 18, Temp 99° F HEENT: PERRLA, EOMI Throat clear Neck: Supple Heart: RRR without murmurs, rubs, or gallops Lungs: CTA Abdomen: Benign Extremities: Normal muscle strength Range of motion in right hip is diminished compared to the left; severe pain produced with movement Neurovasculature is intact Plain films of hip: Lucency of the right hip located where the prosthesis and the femur interact Which of the following is the next step in workup? A) Blood culture. B) Fluoroscope-guided aspiration of the hip. C) Technetium99m bone scan. D) Begin oral dicloxacillin. E) Begin oral gatifloxacin.

Answer B. Fluoroscope-guided aspiration of the hip. Explanation You need to attempt to identify an organism before starting therapy. It is likely to be Staphylococcus, but it could be either sensitive or resistant to methicillin. Other organisms are possible because a previous surgery is involved. This is a chronic problem, and blood cultures are likely not to be helpful. The bone scan really is not of any help either—we know he has bone involvement by plain films. Starting therapy at this point is not prudent, and it is probable that he will need removal of the prosthesis and debridement of the area.

A 48-year-old woman living in Arkansas presents complaining of a "swollen, infected and tender neck gland" associated with fever, nausea, and vomiting. On physical examination, her temperature is 103° F. Positive findings include a tender, fluctuant, and warm 2 x 1.5-cm anterior cervical lymph node and an ulcerative lesion with associated eschar formation located several inches below the node. Upon further questioning, she reports that she has recently been caring for a rabbit after it was bitten by their family dog. Which of the following is the most appropriate choice of treatment for this patient? A) Cefotaxime B) Gentamicin C) Imipenem-cilastatin D) Famciclovir E) Itraconazole

Answer B. Gentamicin Explanation She has clinical signs and symptoms most consistent with tularemia, caused by the gram-negative bacteria Francisella tularensis . It is most common among residents of Arkansas, Oklahoma, and Missouri. The organism, which can penetrate intact skin and mucous membranes, is transmitted following contact with an infected animal or by a tick vector. Tularemia most often presents with ulceroglandular or isolated glandular disease. The ulcer is often associated with eschar formation followed by regional lymphadenopathy. Physical findings may also include hepatosplenomegaly and a variety of cutaneous lesions including a generalized maculopapular rash, erythema multiforme, or erythema nodosum. Pneumonia may follow inhalation of the organisms, and conjunctivitis may result from direct inoculation by contaminated fingers or debris. The diagnosis may be confirmed by serum agglutination tests with titers of ≥ 1:160. The treatment of choice is gentamicin in more severe cases or doxycycline in those that are less severe; streptomycin is an alternative to gentamicin, but most prefer gentamicin because of the ability to use IV instead of IM injections. The Boards will not make you choose between appropriate antibiotic choices because it is a clinical decision, and there won't be a "right answer" unless the patient is deathly ill. Relapses are more common in patients treated with tetracyclines, but it is reasonable to try if the patient is not that ill. The organism is not susceptible to third-generation cephalosporins. Prevention is best accomplished by taking appropriate precautions in tick-infested areas and in handling injured animals.

A 25-year-old man presents for a routine physical exam. He has no chronic medical problems, takes no medications, and does not smoke or drink alcohol. He is gay, has been sexually active with one partner for the past 6 months, and reports sporadic condom use. He works as an accountant and has not traveled outside the U.S. His exam is normal. You suggest HIV testing, sexual risk counseling, and routine blood work for cholesterol. He has not had a recent PPD or CXR, and he has not had hepatitis A, hepatitis B, or pneumococcal vaccines. Which of the following preventive health measures is also appropriate for him today? A) Hepatitis A, hepatitis B, and pneumococcal vaccine B) Hepatitis A and B vaccines C) Placing a PPD D) Screening chest radiograph E) Pneumococcal vaccine

Answer B. Hepatitis A and B vaccines Explanation He has unprotected sex and therefore is at increased risk for HIV, hepatitis A and B, and other STDs. Therefore, he should receive both hepatitis A and B vaccines today. He has no real risk factors that increase his risk for pneumococcal infection at this time. If he is HIV infected, then pneumococcal vaccine is appropriate as would placing a PPD, but otherwise he does not need either one of these at this time. A CXR is also unnecessary.

A previously healthy 45-year-old man comes to the emergency department with a history of headache and fever, which he has had for about a day and a half. His wife reports that he has been confused and, at times, she is not able to understand what he is saying. This has progressively gotten worse in the past few hours. They are "outdoorsy" people and camp quite a bit in the Ozark Mountains in Missouri and Arkansas. PAST MEDICAL HISTORY: Negative SOCIAL HISTORY: Airline pilot; no travel outside of U.S. FAMILY HISTORY: Father with Alzheimer's at age 60 Mother healthy PHYSICAL EXAMINATION: BP 110/70, P 100, RR 18, Temp 102° F Oriented only to person; does not know where he is or the current year HEENT: PERRLA, EOMI TMs clear Throat clear Neck: Stiff with meningismus Heart: RRR without murmur, rubs, or gallops Lungs: CTA Abdomen: Bowel sounds present; no hepatosplenomegaly Extremities: No cyanosis, clubbing, or edema Skin: No rash Neuro: Normal other than mental status testing LABORATORY: CT without contrast preliminary results: no infarct; ventricles normal EEG shows abnormalities in the right temporal lobe area Lumbar puncture results below: WBC: 220 WBC/cu mm (30% polys; 70% lymphs) RBC: 400 RBC in tube 1 RBC: 300 RBC in tube 4 Protein: 65 mg/dL (normal) Glucose: 80 mg/dL with serum of 120 mg/dL Gram stain of CSF: No organisms seen Acid-fast stain: No organisms seen Cryptococcal antigen assay on CSF: Negative Which of the following is the most likely cause of his illness? A) Streptococcus pneumoniae B) Herpes simplex virus C) Listeria monocytogenes D) Francisella tularensis E) Borrelia burgdorferi

Answer B. Herpes simplex virus Explanation Even though this man has been in an area with possible tularemia, it is very unlikely to present in this fashion. He has not been in an endemic area for Lyme disease (Borrelia burgdorferi). Listeria and Streptococcus are much less likely, based on his CSF findings. The protein and glucose are normal, which more likely indicates a viral etiology, although the protein is commonly elevated in herpes encephalitis. His confusion indicates encephalitis is also occurring, which more commonly occurs with herpes infection than with any of the other organisms listed. The EEG findings localized to the temporal lobe area are classic for herpes encephalitis. PCR of the CSF would be the best diagnostic test to detect herpes simplex virus. IV acyclovir is the treatment of choice.

A 35-year-old woman presents for evaluation. She has a 5-year-old son with an infection diagnosed 2 weeks ago; he presented then with "slapped cheeks" and rash on his extremities. Today, she complains of a mild rash and arthritis of her hands. The arthritis mainly involves the wrist and the proximal interphalangeal joints. She reports a good amount of pain associated with this arthritis. Which of the following is true about her infection? A) Her arthritis will likely become superinfected with Streptococcus pyogenes. B) If she has sickle cell disease, she could be at risk for aplastic crisis. C) The arthritis is due to a bacterial superantigen. D) The child is likely still contagious to other family members. E) Immunization could have prevented this from occurring.

Answer B. If she has sickle cell disease, she could be at risk for aplastic crisis. Explanation Parvovirus B19 is a common illness of childhood. In adults who are infected, rash and arthritis are common, especially in women. In those with hemolytic anemias and particularly sickle cell disease, parvovirus may cause aplastic anemia/crisis. It also can do this in AIDS patients. The arthritis is likely an immune reaction to the virus, not due to any bacteria, and complicated bacterial arthritis is extremely rare in these patients. Once the rash appears in the child, the child is considered no longer contagious. There is no human vaccine to prevent parvovirus B19.

A 30-year-old woman presents in her 32nd week of pregnancy. She initially sees her obstetrician with complaints of fever and myalgias without any localizing symptoms. She denies any other problems. Her OB sends her to you for evaluation. PAST MEDICAL HISTORY: 1st pregnancy; no problems until now SOCIAL HISTORY: Doesn't smoke or drink Works as a pharmaceutical rep FAMILY HISTORY: Non-contributory REVIEW OF SYSTEMS: Really quite uninteresting PHYSICAL EXAMINATION: Normal except for a temperature of 102.5° F LABORATORY: WBC is 10,000/cu mm with 70% neutrophils, 10% bands. Urinalysis is normal. Blood cultures are growing a gram-positive diphtheroid-like organism. Which of the following is the most appropriate antibiotic for this organism? A) Intravenous ceftriaxone. B) Intravenous ampicillin. C) No antibacterial therapy is indicated. D) Intravenous gentamicin. E) Intravenous clindamycin.

Answer B. Intravenous ampicillin. Explanation This poor woman has bacteremia with Listeria, a gram-positive diphtheroid-like organism. Think of this organism if pregnancy is involved (especially in the 3rd trimester after week 30 or so) or with someone who eats a lot of goat or imported cheeses, particularly from Mexico, or is an immigrant living in California. Penicillin or ampicillin is the only effective treatment; none of the other antibiotics is effective. Gentamicin can be used for synergy. She is bacteremic, so you are not going to use oral penicillin or ampicillin for this type of infection. If she is penicillin-allergic, you could use trimethoprim/sulfamethoxazole, or you could desensitize her if she was seriously ill. Normally, if you hear about a diphtheroid-like organism you suspect contamination—however, in someone immunocompromised or semi-immunocompromised (like pregnancy), don't ignore the blood culture result until you are sure it comes back as something unimportant.

A 50-year-old man fishes regularly on the Chesapeake Bay. Three weeks ago he was listening to the radio and was distracted. On that day, he caught some crabs, and one of the crabs scratched him on his arm with one of its claws. Over the next 2 weeks, he developed progressive painful swelling of the wound site. He took aspirin and ibuprofen without relief. His wife had some leftover dicloxacillin that he took, which also did not improve his condition. His physical examination is really unremarkable except for marked swelling and induration of the scar area on the side of his left arm. He does not drink alcohol or smoke. Which of the following organisms is most likely the cause of his illness? A) Clostridium septicum B) Mycobacterium marinum C) Staphylococcus aureus D) Streptococcus pyogenes E) Bartonella crustacea

Answer B. Mycobacterium marinum Explanation Think about this organism in someone who is around water and marine organisms. For humans, the most common method of getting this is minor trauma, particularly contact with fish spines or crustaceans. Fish tank and swimming pool outbreaks have also occurred. The infection takes a while to incubate, usually 2-3 weeks, and obviously will not respond to routine antibiotics. Sporotrichosis, chromomycosis, and blastomycosis can also all present like this—but nothing in the history helps us think about these. Vibrio vulnificus is also associated with water, but you would expect to see bullous skin lesions; and it is usually seen in someone with liver disease, particularly alcoholics. S. pyogenes, C. septicum, and S. aureus may all cause skin and soft tissue infections in other settings. Bartonella crustacea does not exist.

A 25-year-old man develops diarrhea after traveling to a water park in Georgia. He ate hamburgers and played in the "kiddie" pool while there. His diarrhea became bloody this morning, and he presents with fever, cramps, and bloody stool. His physical examination is unremarkable except for positive occult blood on his rectal examination. Cultures are sent and reveal enterohemorrhagic E. coli. Based on this finding, which of the following antibiotics should you start? A) Metronidazole B) No antibiotics C) Trimethoprim-sulfamethoxazole D) Ciprofloxacin E) Amoxicillin

Answer B. No antibiotics Explanation He has diarrhea due to enterohemorrhagic E. coli (O157:H7). Studies have shown that using antibiotics will increases the risk of the hemolytic uremic syndrome. Therefore, antibiotics are contraindicated for this type of diarrhea.

You are about to discharge a 62-year-old man from the hospital after a GI bleed, which was determined to be caused by an AV malformation in the colon. He feels well and has no symptoms. His past medical history consists of mild, diet-controlled hyperlipidemia and cataracts. He is a life-long nonsmoker. He was initially admitted to the MICU with anemia, but was stepped down after 24 hours and has had an uncomplicated hospital course since then. It is now hospital day #3, and his hemoglobin remains stable at 10. He has had an indwelling Foley catheter and a peripheral central venous catheter since admission to the MICU. On exam, he is afebrile, and the exam is normal. WBC: 7,600 cells/mm 3 Urinalysis performed on day #2: 3+ bacteria, 10 WBC/hpf, 0 RBC/hpf, nitrite-negative Urine culture: E. coli > 10 5 cfu/mL; sensitive to ampicillin Which of the following is the most appropriate management? A) Oral antibiotics, remove Foley and peripheral central venous catheters, discharge to home. B) No antibiotics, remove Foley and peripheral central venous catheters, discharge to home. C) Oral antibiotics, remove Foley and peripheral central venous catheters, delay discharge, repeat U/A in 24 hours, discharge without antibiotics if U/A is negative. D) IV antibiotics, remove Foley catheter, delay discharge 24 hours, repeat U/A and culture.

Answer B. No antibiotics, remove Foley and peripheral central venous catheters, discharge to home. Explanation Asymptomatic bacteriuria does not require treatment in patients with an indwelling catheter, even in the presence of WBCs in the urine. Patients in whom asymptomatic bacteriuria should be treated include pregnant women, men about to undergo TURP, and patients about to undergo urologic procedures in which mucosal bleeding is anticipated; it also may be considered in asymptomatic women with catheter-acquired bacteriuria that persists 48 hours after indwelling catheter removal. Patients in whom screening for bacteriuria is not recommended include the following: premenopausal, nonpregnant women; diabetic women; older persons living in the community; elderly, institutionalized subjects; persons with spinal cord injury; and catheterized patients while the catheter remains in situ . The catheter probably should have come out a lot sooner, and the U/A and urine culture never should have been sent in the first place without signs or symptoms.

While taking out her garbage to the curb, a 35-year-old woman steps on a rusty old can and punctures her foot. As she hobbled back to her house, she stepped in dog feces. She reports having had all her childhood immunizations and a Tdap booster 4 years ago. Which of the following immunizations should be administered at this time? A) DTaP and Tetanus IG B) No further immunization at this time C) DTaP only D) Td and Tetanus IG E) Td only

Answer B. No further immunization at this time Explanation Here are the guidelines to remember: Wound is dirty and either patient has 1. < 3 immunizations, or 2. Immunization history unknown = Give TIG + vaccine Wound is clean and immunizations are up-to-date (< 10 years since last) = No treatment Wound is dirty and immunizations are up-to-date, with most recent < 5 years = No treatment So this woman has a dirty wound and had a booster Td < 5 years ago; therefore, she is protected and does not need further immunizations at this time.

A 33-year-old woman returns from a visit to central Africa. She reports good adherence to her malarial prophylaxis during her trip. She continues to take her mefloquine as directed on returning home. You receive a notification from the CDC that travelers to the country she visited are at risk for a type of malaria that has hypnozoite forms, and that primaquine must be taken as well on return to the United States. Which species of malaria require the use of adjunctive primaquine to prevent the relapsing form of malaria? A) P. vivax and P. malariae B) P. ovale and P. vivax C) P. ovale , P. vivax , and P. malariae D) P. ovale and P. falciparum E) P. falciparum only

Answer B. P. ovale and P. vivax Explanation These 2 species of malaria form hypnozoites in the liver and require primaquine for effective eradication. Primaquine should be given the last 2 weeks of a prophylaxis period after travel to areas where there is P. vivax or P. ovale to eradicate the liver stage. Know that the main causes of malaria in the U.S. are either not taking prophylaxis or stopping prophylaxis too soon after returning from travel to endemic areas.

A 22-year-old female comes to you because of a persistent vaginal discharge. She has had several episodes in the past 2 years that have been self-diagnosed as yeast infections. They have always responded to over-the-counter topical clotrimazole. She has been sexually active since age 16 and currently has 2 male partners during the last 6 months. None of her past or current partners have told her they have any sexually transmitted diseases. Recently, she completed another course of clotrimazole with no improvement in her discharge. She denies a history of vaginal lesions or vesicles. PAST MEDICAL HISTORY: Currently on oral contraceptives SOCIAL HISTORY: Doesn't drink or smoke PHYSICAL EXAMINATION: Completely normal except for slight yellow discharge from the cervical os In addition to counseling her on safe sexual practices you would also do which of the following? A) Pap smear, RPR, HIV antibody, and viral culture. B) Pap smear, RPR, HIV antibody, and Neisseria gonorrheae and Chlamydia trachomatis nucleic acid amplification testing. C) Pap smear, RPR, HIV antibody, and culture for Candida. D) Pap smear, RPR, HIV antibody, no culture needed, and treat empirically for Candida. E) Pap smear, RPR, HIV antibody, and culture for Gardnerella vaginalis.

Answer B. Pap smear, RPR, HIV antibody, and Neisseria gonorrheae and Chlamydia trachomatis nucleic acid amplification testing. Explanation This sexually active young woman has cervicitis without evidence of vaginal inflammation on pelvic exam. This, and her failure to recently respond to topical clotrimazole make Candidiasis unlikely. Because of her cervicitis, she should be tested for gonorrhea and chlamydia infection, because the primary site for these infections in women is the cervix. She should also be tested for concommitant HIV infection with an antibody assay. Culturing for HSV is not indicated without the presence of herpetic ulcerations, and the virus may be shed episodically without the need for antiviral therapy. The absence of vaginal discharge on exam speaks against bacterial vaginosis and culturing for Gardneralla would not be the test of choice (a vaginal slide looking for clue cells and pH testing would be). The 2012 American College of Obstetrics and Gynecology guidelines for the prevention of cervical cancer recommend Pap smears to begin at age 21 and repeated every 3 years. If HPV DNA testing is also used, testing can begin at age 30 and be every 5 years.

A man from Arizona presents with an inguinal lymphadenopathy and fever. He is a hunter and lives in the desert. Which of the following diseases is most likely in this man? A) Tularemia. B) Plague. C) Pastuerella infection. D) Ehrlichiosis. E) Either tularemia or plague is likely.

Answer B. Plague. Explanation Note that for these questions where they give you geography, use it to your advantage! They are trying to tell you something and see if you are familiar with common findings such as this. So Arizona would have plague. Tularemia and ehrlichia (which doesn't have an isolated lymph node anyway) are found in Arkansas, Missouri, and Oklahoma. Pasteurella is seen with cat and dog bites (nothing here indicates that as a possibility). If you were thinking cat scratch, that is due to Bartonella henselae, which wasn't mentioned here.

A 35-year-old woman with a history of a bicuspid aortic valve presents with fever, extreme fatigue, anorexia, and malaise. Her temperature is 101.5° F. On physical examination, a previously noted systolic ejection click is associated with a new harsh ejection murmur. Additional clinical findings include mucous membrane and extremity petechiae and blanching, painless erythematous lesions on the palms and soles. Which one of the following is the most likely cause of this patient's clinical signs and symptoms? A) Neisseria meningitides B) Streptococcus pneumoniae C) Viridans streptococci D) Escherichia coli O157:H7 E) Haemophilus influenzae type b

Answer C. Viridans streptococci Explanation She has clinical signs and symptoms consistent with subacute bacterial endocarditis (SBE). This disorder is caused most often by Staphylococcus aureus (~ 30%), viridans streptococci (~ 20%), enterococci (~ 10%), coagulase-negative staphylococci (~ 10%), Streptococcus bovis (~ 5%), and other streptococci (~ 5%). In suspected cases of SBE, the blood should be cultured for at least 7 days in order to detect fastidious bacteria or fungi. At least 3 cultures should be obtained in any patient undergoing evaluation for SBE. The yield on blood culture is relatively consistent and not related to the intensity of the patient's symptoms or presence or absence of fever. Pretreatment will decrease the yield of a positive blood culture to 50-60%. As with the case described, the majority with SBE has a preexisting structural cardiac abnormality (e.g., bicuspid aortic valves, septal defects, coarctation of the aorta, tetralogy of Fallot, a prosthetic valve) and present with a new or worsening murmur. Petechiae are common in SBE and are most often located on the mucous membranes and extremities. Other cutaneous findings may include splinter hemorrhages: nonblanching, linear reddish-brown lesions beneath the nail beds; Janeway lesions: macular, blanching, painless, erythematous lesions on the palms and soles; and Osler nodes: painful, violaceous nodules located on the pulp of the fingers and toes. Some patients will also have evidence of Roth spots, characteristic findings on eye exam that appear as exudative, edematous hemorrhagic lesions within the retina.

A 65-year-old man with non-insulin dependent diabetes mellitus is seen in your office because of severe pain and tenderness of his right ear. He had been doing well before this. PAST MEDICAL HISTORY: NIDDM for 30 years Currently takes metformin 1,000 mg daily SOCIAL HISTORY: Lives alone Doesn't smoke or drink Retired college math professor PHYSICAL EXAMINATION: BP 130/70, P 100, RR 20, Temp 103° F HEENT: PERRLA, EOMI TMs: Examination extremely painful and shows marked edema, erythema, and purulent material in the external auditory canal External ear is markedly swollen Throat clear Neck: Supple; no meningismus Heart: RRR without murmurs, rubs, or gallops Lungs: CTA Abdomen: Benign Extremities: No cyanosis, clubbing, or edema Which of the following is the likely etiology for his infection? A) Streptococcus pneumoniae B) Pseudomonas aeruginosa C) Streptococcus diabeticus D) Candida albicans E) Staphylococcus aureus

Answer B. Pseudomonas aeruginosa Explanation This is the most common cause of acute otitis externa in patients with diabetes. Note: This patient needs admission and to be treated with intravenous antibiotics that cover Pseudomonas well to avoid the consequences of malignant otitis externa—osteomyelitis, meningitis, and brain abscess. Choices would include piperacillin/tazobactam or cefepime. The other organisms can cause otitis externa but are less common, especially in diabetics.

Maria Fitz-Hugh-Curtis, a dancer by night, a part-time petting zoo employee by day, presents with a 2-day history of diarrhea. She reports some blood in the stool and has had a low-grade fever. She lives alone and says that coworkers at the petting zoo have also had diarrhea; no one at the dancing club has had diarrhea that she knows of. When asked about the animals at the petting zoo, she rattles off a long list including goats, pigs, caterpillars, iguanas, mice, gerbils, rabbits, ducks, chickens, swans, donkeys, llamas, camels, horses, cows, and snakes. WHEW! You then ask her which animals in particular does she have contact with. She reports that she likes to deal only with the iguanas, snakes, and goats. Assuming that her diarrhea is infectious and due to bacteria, what organism is most likely, based on her animal exposure? A) Klebsiella B) Salmonella C) Shigella D) Francisella E) Neisseria

Answer B. Salmonella Explanation Because she reports iguana exposure, you should have immediately gone for that choice. If a question is going to be odd enough to talk about an iguana, then you need to know they must be talking about Salmonella. Snakes carry the organism as well. Shigella would not be associated with these animals, and Klebsiella does not usually cause a diarrheal syndrome. Francisella could be in the petting zoo but does not cause a diarrhea-exclusive illness. Neisseria would also not cause diarrhea and would not be likely in the petting zoo.

An 18-year-old woman is transported to the emergency room after being found lying on the street in a confused and disoriented state. Two friends accompany her and report that they and the patient have been "living on the streets." Upon arrival, she vomits and is incontinent of stool. On physical examination, she is lethargic but able to recognize her friends. Her temperature is 103.7° F and blood pressure is 88/40. A diffuse, generalized and intensely erythematous rash, associated with hyperemia of the conjunctiva and a "strawberry tongue," is noted in addition to 2 inflamed 4 x 6-cm superficial abrasions over her lower extremity. Laboratory results include elevated serum transaminases, a BUN of 55 mg/dL, and a serum creatinine of 2.7 mg/dL. Which of the following is the most likely cause of this patient's symptoms? A) Viridans streptococci B) Staphylococcus aureus C) Pseudomonas aeruginosa D) Shigella flexneri E) Clostridium difficile

Answer B. Staphylococcus aureus Explanation She has clinical evidence of toxic shock syndrome (TSS), often caused by extracellular toxins produced by Staphylococcus aureus , which cause intravascular fluid loss leading to multi-system disease. TSS may also be caused by certain types of group A β-hemolytic streptococci containing specific M proteins. Clinical signs and symptoms follow systemic absorption of toxin and include fever, hypotension, and an intensely erythematous generalized rash (erythroderma) often involving the mucous membranes. The rash may be preceded by a 2-3 day prodrome of nausea, vomiting, diarrhea, malaise, and myalgia. In addition, documentation of dysfunction in at least 2 other organ systems is required for the diagnosis: elevated serum transaminases, coagulopathy, elevated renal functions, elevated serum creatinine kinase, or altered mental status. Potential sources of toxin-producing Staphylococcus aureus include burns, abrasions, surgical wounds, tampons, indwelling catheters, and nasal packings. Recovery may be complicated by adult respiratory distress syndrome, disseminated intravascular coagulation, renal failure, and cardiogenic shock. Aggressive fluid and hemodynamic resuscitation to reverse hypotension is required. Antimicrobial therapy is recommended to eradicate the offending organisms from the site of infection. Empiric treatment with clindamycin, to suppress toxin synthesis, and nafcillin/vancomycin is recommended prior to positive identification of the offending organism. During recovery, desquamation, especially of the palms and soles, is common. IVIG has also been used in some studies as an adjunctive therapy.

A 35-year-old man comes to the emergency department because of malaise, fever, chills, and a diffuse rash that he has had since awakening this morning. He is a mailman and was bitten by a dog 1 week ago. Subsequently, he noted some pain, redness, and discharge from the bite wound. PHYSICAL EXAMINATION: BP 80/60, P 120, RR 40, Temp 104° F HEENT: PERRLA, EOMI Conjunctivitis is noted Throat: Tongue is beefy red and he has large papillae Neck: Supple Heart: RRR without murmurs, rubs, or gallops Lungs: CTA Abdomen: Benign Skin: Diffuse, pink rash with blotches is noted over his entire body A yellowish fluid is draining from his bite wound LABORATORY: WBC: 20,000/cu mm; 50% polys, 29% bands Hemoglobin: 12.5 mg/dL Platelets: 55,000/cu mm BUN: 70 mg/dL Creatinine: 5.0 mg/dL AST: 500 U/L Serum albumin: 2.2 g/dL Urinalysis: 3+ proteinuria Which of the following is the most likely organism responsible? A) Pasteurella multocida B) Staphylococcus aureus C) Eikenella corrodens D) Bartonella henselae E) Neisseria meningitidis

Answer B. Staphylococcus aureus Explanation What gives you hypotension with a rash and multi-organ involvement with a history of trauma? Toxic shock syndrome! And that is what this poor man has. The dog bite may have tricked you, but remember he is not immunocompromised. Disseminated Pasteurella and Eikenella are really unusual and unlikely to give you this constellation of findings—endocarditis, yes, but multi-organ involvement, no. Bartonella henselae (cat scratch fever) would not do this either. Then we have Neisseria, and it is possible, but the trauma history is what should have led you to Staphylococcus.

An 18-year-old male patient presents to your office complaining of rash and watery diarrhea for the last 10 days. He has no significant past medical history and no history of allergies. He denies fevers or muscle ache, but does note some occasional, diffuse abdominal pain and "achy joints" that have been bothering him more for the last 2 days. He recently returned from a visit to North Carolina with his grandfather. There they hiked several days in the Great Smoky Mountains. On exam, the patient is febrile with temperature of 100° F (37.7° C). He denies pruritus. Which of the following is the most appropriate next step in patient care? A) Send blood for serology and start erythromycin. B) Start doxycycline and send blood for serology. C) Start ceftriaxone 2 grams IV q 24 hours and send blood for serology. D) Perform immunofluorescent staining on a biopsy of the petechial lesions and then have the patient return in three days for follow-up of the results.

Answer B. Start doxycycline and send blood for serology. Explanation This patient has Rocky Mountain spotted fever (RMSF). Fevers, arthralgia, and a maculopapular rash that progresses to a petechial rash on the extremities spreading to the trunk are classic signs and symptoms. Patients may also present with diarrhea and abdominal pain. Diagnosis is based on a high index of suspicion and clinical findings, and it is imperative to begin treatment immediately because this disease does have 3% mortality. Serological testing can be done for confirmation but should not delay treatment. Doxycycline is the treatment of choice for RMSF regardless of age. Definite diagnosis can be obtained via immunofluorescent staining on a biopsy of a petechial lesion, but this, too, should not delay treatment. Ceftriaxone 2 grams IV q 24 h is the treatment for patients with Lyme disease who are manifesting cardiac and neurologic sequelae and for presumed pneumococcal or meningococcal sepsis. This patient's history of hiking in the mountains, however, makes the possibility of RMSF more likely than either S. pneumoniae or N. meningitidis . He has no symptoms of Lyme carditis or meningitis. The rash is not consistent with Lyme disease either.

A 27-year-old Med-Peds resident travels frequently and asks about going to Southeast Asia for a mission trip and wonders about prophylaxis for various diseases. Which of the following should he take for malaria prophylaxis? A) Primaquine B) Quinine and clindamycin C) Atovaquone/proguanil followed by primaquine D) Chloroquine E) Chloroquine followed by primaquine

Answer C. Atovaquone/proguanil followed by primaquine Explanation Atovaquone/proguanil (Malarone®) is effective against chloroquine-resistant Plasmodium falciparae malaria, which he is likely to encounter in Southeastern Asia. Plasmodium ovale/vivax are also present, and primaquine is required to eliminate the liver stage of infection with this organism, which can occur even if acute infection is prevented. Quinine and clindamycin are used for treatment, not prophylaxis.

You are asked to evaluate a 27-year-old otherwise healthy man whose dog bit him yesterday. The animal has not been ill and receives regular veterinary check-ups and vaccinations. The patient has no symptoms, no fevers, and is otherwise healthy. His last tetanus booster was 7 years ago. On exam, he appears well and is afebrile. His left second finger is swollen and erythematous, and he has pain with grasp. He reports numbness over the distal and mid-finger. There is no lymphadenopathy, and no erythema is noted on the arm. Which of the following is the most appropriate intervention? A) Immediate incision and drainage, parenteral antibiotics, no Tdap booster, no rabies vaccine B) Tdap booster, IV ampicillin-sulbactam, no rabies vaccine, consultation with hand surgeon C) Tdap booster, oral amoxicillin-clavulanate, observation, no rabies vaccine D) No Tdap booster, oral amoxicillin-clavulanate, observation, no rabies vaccine E) Tdap booster, IV ampicillin-sulbactam, begin rabies vaccine, consultation with hand surgeon

Answer B. Tdap booster, IV ampicillin-sulbactam, no rabies vaccine, consultation with hand surgeon Explanation Animal bites are often polymicrobial infections ( Staphylococcus sp., Streptococcus sp., gram negatives, and anaerobes) that may include Pasteurella multocida (30-50%). Dog bites are usually not as deep as cat bites. Cat teeth are sharper and more slender and may be more likely to extend down to bone. Amoxicillin-clavulanate is the drug of choice for all bites due to a beta-lactamase produced by the Pasteurella organism. In this case, with the severity of the bite wound and possible compartment syndrome, it is prudent to give a parenteral antibiotic (such as ampicillin-sulbactam). Bites on the hand require consultation with a hand surgeon because of the chance of rapid spread of infection through fascial planes. All animal bites are considered high risk for tetanus; therefore, anyone without a booster within 5 years should get a Tdap booster. Rabies shots (postexposure vaccination) are necessary only if the animal shows signs of illness or if the animal cannot be quarantined.

A 20-year-old woman is evaluated in the emergency department for severe headache and altered consciousness. She undergoes a lumbar puncture, which reveals purulent CSF with a Gram stain showing intracellular gram-negative diplococci. Of the following, who should receive meningococcal prophylaxis? A) All of the above B) The anesthesiologist who intubated the patient C) The resident who did the lumbar puncture D) The third-year medical student who started the peripheral IV

Answer B. The anesthesiologist who intubated the patient Explanation The medical student and resident did not have close exposure to upper respiratory secretions where meningococci colonize the nasopharynx. Since there is a possibility that the anesthesiologist did get exposed to secretions while intubating the patient, the anesthesiologist is the only one who should be offered prophylaxis.

A 34-year-old patient with HIV presents with cough, dyspnea, and bilateral infiltrates. A presumptive diagnosis of PCP is made, and he is treated with trimethoprim/sulfa and prednisone. In addition, he is noted to have oral candidiasis, and fluconazole is given. Labs obtained 48 hours after admission: Na 133, K 7.0, Cl 100, HCO3 23, BUN 20, Cr 1.2 Of the following, what is the most likely cause for the hyperkalemia? A) Fluconazole B) Trimethoprim/sulfamethoxazole C) Fluconazole and prednisone D) Adrenal insufficiency E) Prednisone

Answer B. Trimethoprim/sulfamethoxazole Explanation This patient with advanced HIV disease develops hyperkalemia while being treated for PCP. TMP/SMX can block tubular potassium secretion, and severe hyperkalemia has been reported in AIDS patients receiving IV TMP/SMX for the treatment of PCP. Hyperkalemia has also been reported in elderly patients receiving oral TMP/SMX. Although adrenal insufficiency can cause hyperkalemia and does occur in patients with advanced HIV disease, it would be unlikely in this patient, because he received steroids.

A 60-year-old man is evaluated because of a 1-week history of lower extremity weakness, new onset of difficulty speaking, and decreased attention span. He has had occasional diarrhea and abdominal pain in the last year. Of significance is that he has lost about 25 lbs during the past year. He complains of joint pains, particularly in his knees. He has had low-grade fever but no chills during the last year. He reports occasional night sweats. He has noted no other neurologic findings like seizures. His wife reports that areas of his skin are becoming darker—particularly those exposed to light. PAST MEDICAL HISTORY: Healthy before this episode SOCIAL HISTORY: Lives in Michigan Works in the auto industry FAMILY HISTORY: Negative REVIEW OF SYSTEMS: Pretty much covered in the HPI PHYSICAL EXAMINATION: BP 130/80, T 99.9° F, P 84, RR 18 General: Alert, but oriented only to person and place HEENT: PERRLA, EOMI Mild right-sided facial droop Throat clear Neck: Scattered lymphadenopathy in the anterior and posterior cervical chains; most nodes are 1 x 1 cm, but a few are 2 x 1 cm Heart: RRR without murmurs, rubs, or gallops Lungs: CTA Abdomen: Spleen tip palpated; no hepatomegaly A questionable abdominal mass discerned with deep palpation Extremities: No cyanosis, clubbing, or edema Neuro: Right lower extremity with increased tone and 4/5 muscle strength Sensation is normal Deep tendon reflexes are symmetrical LABORATORY: Hemoglobin: 15.2 gm/dL Hematocrit: 50% WBC: 30,000/cu mm; 65% neutrophils, 28% lymphs ESR: 13 mm/hr Glucose: 200 mg/dL Albumin: 3.5 g/dL ALT: 30 U/L AST: 25 U/L CT of the head shows a hypodense left frontal lobe lesion. A stereotactic brain biopsy is taken and shows acute inflammation and necrosis with no malignant cells. Gram stain shows 1+ WBCs but no organisms. However, a specimen stained with periodic acid-Schiff (PAS) shows multiple PAS-positive foamy macrophages. Which of the following organisms is likely responsible for his condition? A) Coxiella burnetii B) Trophermyma whipplei C) Mycobacterium tuberculosis D) Nocardia asteroides E) Actinomyces israelii

Answer B. Trophermyma whipplei Explanation Patients will present after a long time with nonspecific complaints. The main symptoms are weight loss, diarrhea, and arthropathies—and in nearly 75% of the cases these 3 occur together. The GI symptoms usually begin later but frequently lead to the diagnosis. Usually, they will present with episodic and watery diarrhea or steatorrhea accompanied by abdominal pain and, in about 20-30%, they will have occult blood in their stool. Skin hyperpigmentation is frequently common to the sun-exposed areas. Just about every organ in the body will be infected with this bacterium. The diagnosis is usually made by upper endoscopy, but biopsy of lymph nodes or masses will also yield the diagnosis. Upper endoscopy will show whitish or yellowish plaques distributed on a friable mucosa. The key here is the PAS staining. PCR is also now available and would be useful for confirming this diagnosis. With PAS staining, be careful, because a few other conditions in the colon or rectum would show you this—for example, melanosis coli. Trophermyma whipplei is best treated with trimethoprim/sulfamethoxazole, but severely ill patients like this guy should receive parenteral ceftriaxone initially for at least 2 weeks. Amazingly, chloramphenicol (that old antibiotic) would be a good consideration for CNS disease too because of its great penetration.

A 40-year-old woman lives in a town of 150,000 people in Wisconsin. She reports a 3-day history of diarrhea that is non-bloody in character. This is the 10th person you have seen in the office today with similar history, and your nurse has received at least 10 additional phone calls about similar episodes. Your colleague says that she has seen a bunch of diarrhea cases in the past day. You call the local health department and they report that the diarrhea is citywide, and they are working on figuring out the etiology. They do report that routine stool cultures are negative so far. Stool ova and parasites are pending. What test is most likely to be positive and give a clue as to the etiology? A) Salmonella specific antigen B) Stool for Giardia antigen C) Acid-fast stain of the stool D) Fungal stain of the stool E) Urine antigen test

Answer C. Acid-fast stain of the stool Explanation The community likely has Cryptosporidium, which was involved in a widespread outbreak in Milwaukee in the 1990s. This organism won't grow in regular stool cultures since it is a parasite but will be seen on acid-fast staining of the stool. The other tests listed won't help you with this organism.

A 52-year-old male presents to the local urgent care following an encounter with a neighborhood cat. The patient attempted to catch the cat that was frequently drinking from the family's fish pond and frightening the fish. He was scratched multiple times, and two or three bite marks seem to be present as well. The wounds are washed and irrigated extensively. In considering prophylactic antibiotics for this person, which of the following would be the most appropriate choice? A) Erythromycin B) Clindamycin C) Amoxicillin-clavulanate D) Cephalexin E) Dicloxacillin

Answer C. Amoxicillin-clavulanate Explanation Cat bites have a high probability of being infected with species of Pasteurella organisms and anaerobes. Medications that are effective against Pasteurella include penicillin-VK, amoxicillin-clavulanate, cefuroxime, doxycycline, and fluoroquinolones, and for anaerobes, medications include amoxicillin-clavulanate, metronidazole, and clindamycin. Thus, the only oral antibiotic that covers both Pasteurella and anaerobes is amoxicillin-clavulanate. In penicillin-allergic patients, use doxycycline, TMP/SMX, or quinolones plus either clindamycin or metronidazole.

The Office of Homeland Security notifies all physicians in your area that a possible bioterrorist event has occurred at a local high school football game. You are told to be on the lookout for people with pneumonia and a widened mediastinum on CXR. They tell you that these individuals will not be contagious via the respiratory route and will not require respiratory isolation. Which of the following pathogens is most likely? A) Human herpesvirus 6 B) Smallpox virus C) Bacillus anthracis D) Yersinia pestis

Answer C. Bacillus anthracis Explanation Of the agents listed, anthrax ( B. anthracis ) will cause pneumonia with a widened mediastinum and, importantly, is not spread person-to-person via the respiratory route, as are plague (Y. pestis ) and smallpox. Smallpox pneumonia would be rare and would not present with a widened mediastinum. Plague would present as a hemorrhagic pneumonia. Human herpesvirus 6 causes roseola, a common childhood illness that would not cause pneumonia.

A 20-year-old man came to you with a 1-week history of fever, chills, and left eye conjunctivitis with an associated pre-auricular lymph node. He reported that he was well until this episode. He lives at home with a dog and 3 cats. None of the pets have been ill. PAST MEDICAL HISTORY: Negative SOCIAL HISTORY: Works as a veterinarian's assistant; recently was bitten by a turtle and a rabbit PHYSICAL EXAMINATION: Besides the lymph node and the non-purulent conjunctivitis, everything else is normal He was started on oral cephalexin. He returns 3 days later with no improvement. A surgery colleague sees him and performs a biopsy, which shows necrotizing granuloma without organism. Acid-fast stains are negative. The most likely organism causing this picture is which of the following? A) Staphylococcus aureus, methicillin-resistant B) Herpes simplex 1 virus C) Bartonella henselae D) Aeromonas hydrophilia E) Borrelia burgdorferi

Answer C. Bartonella henselae Explanation This patient has cat scratch fever or disease. Note: Commonly you'll see patients present with axillary nodes and scratches on the hand. But know that a classic conjunctivitis pre-auricular node syndrome is cat scratch disease. Bartonella is one of the few organisms other than TB that causes necrotizing granulomas. Lyme disease does not cause conjunctivitis. Herpes causes keratitis, not conjunctivitis. MRSA would cause a purulent conjunctivitis. The poor turtle here is innocent, so Aeromonas is not a problem either. Aeromonas typically would cause an exudative diarrhea. What causes lymph node swelling in a person who works with animals that does not respond to anti-staph drugs? Most common thing is going to be cat scratch.

A 70-year-old man presents with a painless lesion on his penis. He says that he had his sildenafil citrate prescription refilled 2 months ago and had unprotected sex with a prostitute 4 weeks ago while visiting Orlando. His rapid plasma reagin (RPR) and MHA-TP are positive. A picture of the penile lesion is shown here: Which of the following is the most appropriate pharmacotherapy? A) Penicillin G 12 million units IV once daily for 10 days B) Benzathine penicillin G 2.4 million units IM q week x 3 C) Benzathine penicillin G 2.4 million units IM D) Ceftriaxone 1 gram IM E) Procaine penicillin 4 million units IM x 1

Answer C. Benzathine penicillin G 2.4 million units IM Explanation For primary syphilis and secondary syphilis (< 1 year since acquiring the infection), one dose of long-acting penicillin is appropriate. Procaine penicillin should not be prescribed unless it is combined with probenecid to prolong the half-life of penicillin. Ceftriaxone is not recommended by the CDC for primary syphilis. If he was penicillin-allergic, you would prescribe doxycycline 100 mg bid x 14 days.

A 55-year-old man comes to the night clinic with a history of acute onset of nausea, severe headache, and facial flushing. He says that the symptoms began while he was sitting at home drinking a beer. On further questioning, you discover that he is receiving an outpatient intravenous antibiotic for a wound infection. Which of the following antibiotics is most likely responsible? A) Ceftriaxone B) Clindamycin C) Cefotetan D) Imipenem E) Piperacillin/tazobactam

Answer C. Cefotetan Explanation Ingestion of alcohol by patients receiving cephalosporins containing the MTT (methylthiotetrazole) moiety at position 3 has been associated with disulfiram-like reactions. The MTT group's configuration is similar to disulfiram, and appears to block alcohol metabolism at the acetaldehyde step. The accumulation of acetaldehyde is associated with these symptoms. The antibiotics with the MTT side group are cefotetan, cefamandole, cefoperazone, moxalactam, and cefmetazole (the latter 2 are no longer available in the U.S.). Note that these agents also inhibit vitamin K 2,3—epoxide reductase, which converts inactive vitamin K to its active form. Therefore, in a patient with already depleted vitamin K (like an alcoholic or someone with liver disease), use of these antibiotics could increase the PT and cause bleeding problems.

An 84-year-old man comes to the emergency department because of jaw tightness and neck stiffness. About a week ago, he had a minor injury to his left thigh—his grandson jabbed a pitchfork into his thigh while cleaning out the barn. He applied local care and pretty much ignored the wound as it healed. He takes no medications. PAST MEDICAL HISTORY: Pneumonia at age 36 Healthy since the pneumonia SOCIAL HISTORY: Lives alone on his farm FAMILY HISTORY: Negative PHYSICAL EXAMINATION: Significant aspects: He is alert and oriented to person, place, and time All vital signs are normal On mouth exam: He cannot open it completely Pupils react normally and extraocular movements are intact Neck is stiff Abdomen is tense; you note that all of the muscles of his abdominal wall are rigid His extremities exam is remarkable for all of the muscles of his extremities being rigid During the exam, especially with his reflexes, you note tonic-clonic motor activity While examining him, a loud, boisterous nurse walks in and startles you both; he develops opisthotonic posturing with flexion of the arms and extension of his legs. He does not lose consciousness but experiences severe pain with this episode. Of the following, what is the likely etiology for this man's condition? A) Clostridium botulinum B) Clostridium septicum C) Clostridium tetani D) Fusobacterium necrophorum E) Corynebacterium diphtheriae

Answer C. Clostridium tetani Explanation This man has tetanus! Look for it in elderly people who have likely not been immunized in years. Classically, they will present with "lockjaw" and then progress to have the other symptoms. The weird opisthotonic posturing described is classic for tetanus. Note: The only other thing that will do this is strychnine poisoning. Dystonic reactions to drugs can also look like this, but remember that usually this involves lateral head turning, which is very rare in tetanus. Dental infections may produce trismus but do not cause the other manifestations of tetanus. Treatment is well laid out in many texts: always protect the airway; administer diphenhydramine to be sure you are not dealing with a dystonic reaction; then proceed to giving a benzodiazepine to control spasms and decrease rigidity. Treatment consists of administering human tetanus immunoglobulin and immunization with Td (tetanus toxoid). Metronidazole is also given for 7-10 days. The course can be quite prolonged and require 6-8 weeks of intense rehabilitative therapy. Patients will receive 3 Td doses over the period of 8 weeks.

A 21-year-old man is evaluated for a rash and fever. He states he has felt sick for the past 3 weeks with "flu-like" symptoms that progressed to fever, fatigue, and muscle aches. He took some left-over amoxicillin from an episode of pharyngitis 6 months ago, and subsequently developed a reddish rash over his trunk and extremities. He takes no other medications and has no risks for HIV infection. He is up-to-date with immunizations. He does not smoke cigarettes, uses alcohol only on weekends, and does not use "recreational" drugs. Physical exam shows an ill-appearing but well-nourished young man. Temperature is 38.8° C (101.8° F); otherwise vital signs are normal. There are multiple tender anterior and posterior cervical lymph nodes. There is a pharyngeal exudate, and you note a few petechiae on the palate. A "fullness" is palpable in the left upper quadrant. He has an erythematous, morbilliform rash on his chest, back, arms, and legs. Which of the following is the most appropriate next step in management to prevent morbidity? A) Counsel the patient to abstain from sulfa-based antibiotics. B) Counsel the patient to abstain from alcohol. C) Counsel the patient to abstain from contact sports. D) Counsel the patient to abstain from sun exposure. E) Counsel the patient to abstain from estuarine water exposure

Answer C. Counsel the patient to abstain from contact sports. Explanation This patient has mononucleosis, most likely EBV-associated. The presence of palatal petechiae, exudative pharyngitis, and a morbilliform rash that worsens after exposure to amoxicillin is highly suggestive of EBV. In addition, he has an enlarged spleen on physical exam, which makes him susceptible to splenic rupture. All significantly ill patients with EBV should be advised to avoid activities that could lead to splenic rupture. A white blood cell count with differential may show atypical lymphocytes, and a heterophile antibody test confirms the diagnosis. There is no reason for this patient to specifically avoid alcohol, sulfa-based drugs, estuarine water exposure, or sun exposure, given this history.

A 25-year-old woman is newly diagnosed with HIV. Her CD4+ count returns and is 165. Her HIV viral load is 100,000 copies/mL. However, she is asymptomatic. Which of the following treatments do you recommend at this point? A) Offer her ZDV/d4T and efavirenz with TMP/SMX. Do resistance testing if poor response to this regimen. B) Do resistance testing, then, if okay per resistance testing, treat her with ZDV only since she might get pregnant in the next few months. C) Do resistance testing, then, if okay per resistance testing, treat her with tenofovir, emtricitabine, and efavirenz. Also give her TMP/SMX. D) Offer her ZDV/d4T/ddC with TMP/SMX. Do resistance testing if poor response to this regimen. E) Return in 6 months and, if CD4 is below 150, start anti-retroviral therapy.

Answer C. Do resistance testing, then, if okay per resistance testing, treat her with tenofovir, emtricitabine, and efavirenz. Also give her TMP/SMX. Explanation Current guidelines now recommend: 1) Resistance testing for all treatment-naïve patients immediately before initiating treatment. 2) Start combination anti-retroviral therapy (ART) on all patients with HIV infection to reduce disease progression and help prevent transmission of HIV. Preferred initial regimens for antiretroviral-naïve patients (after resistance testing): *Tenofovir/emtricitabine/efavirenz (TDF / FTC / EFV) [Remember: EFV is absolutely contraindicated in pregnancy.] *Tenofovir/emtricitabine/ritonavir-boosted atazanavir (TDF / FTC / ATV/r) *Tenofovir/emtricitabine/ritonavir-boosted darunavir (TDF / FTC / DRV/r) *Tenofovir/emtricitabine/raltegravir (TDF / FTC / RAL) Note that TDF and FTC are included in all of the above combinations! The TMP/SMX is for primary PJP prophylaxis based on her CD4 count < 200. The other question answer choices are wrong—in some cases for multiple reasons. If resistance testing before treatment is not done, or if combination ART is not used, that answer choice is automatically wrong. Other points: It is inappropriate to send her away without offering therapy and not scheduling her follow-up for 6 months. The ZDV/d4T and efavirenz regimen is wrong because the ZDV/d4T combo is antagonistic. Using ZDV/d4T and ddC would also be wrong because of the ZDV/d4T antagonism as well as the inferiority of a 3-nucleoside regimen with the inferior ddC (never pick ddC). Again, never choose single-drug therapy such as ZDV alone as therapy for anyone.

A 24-year-old otherwise healthy day care worker presents with signs and symptoms of meningitis that developed 10 days after a cold. Lumbar puncture (LP) is performed. LP results: Opening pressure: 210 cm H2O WBC: 4,000/µL with 85% granulocytes RBC: 20/µL Total protein: 300 mg/dL Glucose: 10 mg/dL (serum is 98 mg/dL) It is 3 a.m. and the Gram stain cannot be completed until the morning laboratory crew arrives. Which of the following is the most appropriate empiric pharmacotherapy? A) IV ceftriaxone B) IV vancomycin C) IV vancomycin and IV ceftriaxone D) IV ampicillin and IV gentamicin E) IV vancomycin and IV gentamicin

Answer C. IV vancomycin and IV ceftriaxone Explanation The scenario doesn't give us much information, but you must assume that the two most likely organisms are Streptococcus pneumoniae and Neisseria meningitidis . For Neisseria , ampicillin or ceftriaxone is likely adequate; but for S. pneumoniae , vancomycin should be added initially until you know the sensitivities for the organism. The clues that she works in a day care center and had a recent cold should raise your suspicions that she has been exposed to resistant pneumococci, because infants and children in day care settings are more likely to be colonized or infected with penicillin-resistant pneumonococcus. Be aware also that in some parts of the U.S., 8-10% of S. pneumoniae are resistant to ceftriaxone

Following clinical evaluation for a febrile illness, a 25-year-old woman is thought to likely have an infection caused by adenovirus. Which of the following findings is most likely to be associated with her illness? A) Petechiae covering the extensor surfaces of the hands and feet B) Ulcerative lesions covering the soft palate C) Inflammation of the bulbar and palpebral conjunctiva D) Swelling of one or more large joints E) Enlargement of the spleen and liver

Answer C. Inflammation of the bulbar and palpebral conjunctiva Explanation Especially during the summer months, adenovirus is a common cause of fever associated with exudative pharyngitis, nonpurulent conjunctivitis, and a macular rash most prominent over the trunk. Additional clinical findings often include preauricular and/or anterior cervical lymphadenopathy, headache, malaise, and rhinitis. Patients may initially undergo evaluation for group A β-hemolytic streptococcal (GAS) infection. However, it is important to note that GAS infection characteristically presents with signs and symptoms of exudative pharyngitis in the absence of associated upper respiratory symptoms. In fact, patients with an exudative pharyngitis associated with additional upper respiratory symptoms such as cough and rhinitis should not routinely be tested for GAS disease, as positive results in this setting will likely be the result of a carrier state rather than active infection. Adenovirus may also cause epidemics of keratoconjunctivitis, as well as myocarditis, acute diarrhea, pneumonia, and hemorrhagic cystitis. Enlarged spleen or liver is often associated with Epstein-Barr virus infection; petechiae of the extensor surfaces is consistent with Rocky Mountain spotted fever; swelling of one or more joints is consistent with parvovirus; and ulcerative lesions of the soft palate with herpangina caused by coxsackievirus.

A 28-year-old female in her third month of pregnancy is referred to you by her obstetrician for recommendations regarding malaria prophylaxis. The patient is anticipating a 6-week church mission trip to a chloroquine-resistant malarial area. Despite recommendations to defer the trip, the patient is adamant on completing her travel plans. Which of the following is the most appropriate anti-malarial pharmacotherapy? A)) Atovaquone-proguanil B) Doxycycline C) Mefloquine D) Frequent applications of topical 70% DEET

Answer C. Mefloquine Explanation DEET is an effective mosquito repellant with limited safety studies in pregnancy. High concentrations of DEET have been linked with rashes, seizures, and encephalopathy in children. Concentrations over 50% have not been shown to provide additional protection. Frequent, direct application of high-concentration DEET to skin is not recommended. Doxycycline is contraindicated in pregnancy, and atovaquone-proguanil does not have enough safety data in pregnancy to recommend its use. Mefloquine is the drug of choice for pregnant patients in high-risk, chloroquine-resistant malaria situations and can be used in all trimesters.

An 18-year-old high school student presents to the emergency department with a severe swollen left ankle. She notes that she had fever today and then had some "bumps" come up on her leg. The ankle became swollen this morning and has progressively gotten worse. She is sexually active but reports that her "boyfriend was a virgin like me" and denies any knowledge of STD exposure. She started her menses yesterday, and it has been normal. She denies any trauma. Physical examination is significant for a markedly swollen left ankle with frank arthritis and palpable edema. Two small pustules are located above the ankle area. What is the most likely etiology for her arthritis? A) Streptococcus pyogenes B) Chlamydia C) Neisseria gonorrhoeae D) Staphylococcus aureus E) Parvovirus B19

Answer C. Neisseria gonorrhoeae Explanation She has classic skin lesions, exposure history, and the disease process started with menses. This scenario in a young person with arthritis should lead you to this diagnosis on the Board exam. It is very unusual for a sexually active person to have arthritis due to Staphylococcus aureus or other bacteria. Parvovirus B19 usually involves smaller joints of the hands.

You are called to evaluate a 74-year-old man with Parkinson disease. He has been hospitalized twice in the last year for urosepsis. Because of incontinence, he has required a chronic indwelling urinary catheter. He is currently afebrile and has no abdominal pain. He has had no mental status changes or recent falls. Lab work from 48 hours ago includes: WBC 8,000, Hct 42%, Na 137, K 3.6, BUN 20, Cr 1.2, and urine culture > 100,000 colonies of Enterococcus. Of the following, what therapy do you recommend? A) Amoxicillin B) Ceftriaxone + gentamicin C) No therapy at this time D) Ampicillin + gentamicin E) Ciprofloxacin

Answer C. No therapy at this time Explanation This patient is asymptomatic but has a positive urine culture for a frequently drug-resistant organism. Most chronically catheterized patients will have positive urine cultures. Treatment should occur only if the patient has a symptomatic infection. This patient is asymptomatic, including a normal WBC. Attempted treatment would likely lead to a more drug-resistant organism without offering benefit to the patient.

A 30-year-old man with AIDS presents for worsening fatigue. He was found to be HIV-infected about 12 years ago. His CD4 count at that time was 180/cu mm, and he began taking trimethoprim-sulfamethoxazole and zidovudine (AZT). During the last several years, he has not wished to take any antiretroviral therapy and takes only monthly aerosolized pentamidine for Pneumocystis prophylaxis. Five months ago, his hemoglobin level started to decrease. A workup for gastrointestinal bleeding revealed nothing. Laboratory for iron, B12, and folate were all normal. A serum erythropoietin level was 700 mU/mL (normal is 4-26). He received a red cell transfusion about 3 months ago. His physical examination shows pallor. LABORATORY STUDIES: Hemoglobin: 6.0 g/dL Hematocrit: 19% WBC: 5,000/cu mm CD4 count: 62/cu mm Reticulocyte count: 0% A bone marrow aspirate and biopsy reveal red blood cell aplasia with giant pronormoblasts. Which of the following is the likely etiology for his anemia? A) Prior zidovudine use B) HTLV-I from his transfusion 3 months ago C) Parvovirus B19 D) Campylobacter jejuni E) Pentamidine-induced anemia

Answer C. Parvovirus B19 Explanation Parvovirus B19 infects red cell precursors in the marrow and causes an arrest in red cell maturation (at the pronormoblast stage) with a resultant hypoproliferative anemia and high erythropoeitin levels. Immunocompetent patients will generate an immune response that clears the virus in about a week, and thus hemoglobin levels drop only 1-2 grams, which often goes unnoticed. Patients with late-stage HIV infection are incapable of mounting an immune response to the virus and develop a chronic, often severe anemia. Treatment is with IVIG, which contains sufficient levels of antibody to the virus to erradicate the infection. Zidovudine could cause anemia, but the anemia resolves after stopping the agent. HTLV-1's hematologic effect is to cause a T-cell lymphoma/leukemia and not pure red cell aplasia. Inhaled pentamidine is poorly absorbed and rarely causes any systemic toxicity. Campylobacter causes severe diarrhea in HIV-infected persons, but not red cell aplasia. Anemia is a well known dose-dependent side effect of zidovudine, but it resolves after discontinuation.

A 26-year-old man presents to the emergency department with fever and new-onset seizures. He has been acting erratically; and this morning, he thought he was the King of Nigeria. Prior to arrival, he had 2 episodes of tonic-clonic seizures, and upon arrival to the emergency department, he seized a third time. He was given lorazepam and a loading dose of phenytoin. On physical examination, he appears agitated and confused. Temperature 100.3° F (37.9° C), BP 120/65 mmHg, P 120 bpm, and oxygen saturation 97% on room air. Blood glucose level at bedside = 98 mg/dL. HEENT: PERRLA, EOMI No papilledema No lacerations or bruising Neck: No meningismus Heart: Regular rate without murmurs, rubs, or gallops Lungs: Symmetric excursions, resonant to percussion, vesicular breath sounds throughout Abdomen: Normal bowel sounds, no hepatosplenomegaly Extremities: No cyanosis, clubbing, or edema Neuro: Sedated and unable to follow commands No gross facial asymmetry noted Gag reflex intact Appropriate responses to noxious stimuli Well-developed musculature No rigidity noted Reflexes were equal and symmetrical bilaterally Initial Laboratory: WBC: 12,000/µL Hgb: 13.0 g/dL Plt: 404,000/µL Sodium: 134 mEq/L Potassium: 3.7 mEq/L Chloride: 92 mEq/L Carbon dioxide: 26 mmol/L BUN: 10 mg/dL Creatinine: 1.1 mg/dL Drug screen: Negative CXR: Normal CT of Brain: No mass lesions; no hemorrhages LP: Opening pressure: 18 cm H2O 176 WBC/µL (90% lymphocytes) 760 RBC/µL 70 mg/dL glucose 95 mg/dL protein Gram stain, acid-fast smear, and cryptococcal antigen negative Serum RPR: Negative Which of the following tests is the most sensitive for discerning HSV as a cause of meningoencephalitis? A) CT of the head B) EEG C) Polymerase chain reaction (PCR) testing of CSF D) Virus culture of cerebrospinal fluid (CSF) E) MRI of the head

Answer C. Polymerase chain reaction (PCR) testing of CSF Explanation PCR testing is most likely to be positive 3 days after infection, so an initial negative PCR should be repeated at 3 days if HSV is strongly suspected. The sensitivity is 98%, with a specificity of 94-100% during active infection. MRI has less sensitivity at 89% and, when abnormal, will show lesions in the temporal lobes. EEG has a sensitivity of approximately 80%, and CT scan has a sensitivity of about 55%. A viral culture of CSF is rarely positive. It used to be that the "gold standard" was brain biopsy; this has been replaced by PCR.

An 18-year-old boy from Michigan presents for evaluation. Four days ago, he stepped on a nail while wearing his tennis shoes. When this occurred, he was running in a cross country race in pastureland. The nail penetrated his tennis shoe and punctured his right foot. After the initial pain, he was fine. Last night, however, he developed severe pain and swelling. Since last evening, he has been unable to bear weight on his foot. He is admitted to the hospital and a culture is obtained in the operating room. The Gram stain shows gram-negative rods. Based on this, which of the following is the most likely organism? A) Streptococcus pyogenes (Group A strep) B) Staphylococcus aureus C) Pseudomonas aeruginosa D) Leptospirosa interrogans E) Eschericia coli

Answer C. Pseudomonas aeruginosa Explanation This is a classic presentation for Pseudomonas osteomyelitis. A history of "nail through a tennis shoe" should automatically cause you to pursue a Pseudomonas infection, especially osteomyelitis. The organism grows in the water absorbent layer of the shoe and is inoculated at the time of injury.

An 80-year-old woman comes to your office with a 5-day history of fever, severe diarrhea, and increased urinary frequency. She develops hypotension, end-organ failure, and requires admission to the intensive care unit. The woman lives with her daughter and her family. They own a dog, cat, ferret, and turtle, but the patient has not had direct contact with the ferret or turtle. The daughter mentions that the turtle bowl is cleaned in the family kitchen sink. Cultures of her urine, stool, and blood all yield the same organism. The patient's cultures will most likely grow which of the following organisms? A) Shigella B) E. coli O157:H7 C) Salmonella D) Campylobacter E) Pasteurella

Answer C. Salmonella Explanation Turtle importation ceased in the 1970s; and with that, we observed a marked decrease in cases of Salmonella infection. However, in the 1990s and 2000s, iguanas became very popular, and there was again a big increase in the number of cases of this infection. The CDC recently reported several cases of Salmonella due to turtles—so be suspect of turtles on the Board examination; they are likely culprits for Salmonella just like our friend, the iguana. The organism is easily transmitted on fomites or from contaminated communal living areas such as the kitchen sink. Direct contact with the reservoir reptile is not required for infection. Shigella is not associated with reptiles. Pasteurella is associated with cat and dog bites, but this was not described and would be very unlikely in this clinical picture. E. coli O157:H7 is associated with contaminated meat products or point-source outbreaks. Campylobacter is not common from undercooked fowl but is not associated with reptiles.

A 32-year-old woman is brought to the clinic by her boyfriend. She is complaining of a terrible headache and neck stiffness. Her boyfriend says that she has not been herself since about 3 hours ago. She has had fever to 102° F and says that she had a "drooping of her left face." She now complains of double vision. Physical Exam: Temperature is 102.5° F. Patient has paresis of the left lower face with forehead spared and neck stiffness. No papilledema. No other focal neurologic signs. You immediately have her transferred to the local hospital where you recommend which of the following to the nurse at the front desk? A) Stat blood cultures, then give IV antibiotics, and then get a head CT followed by a lumbar puncture, followed by IV dexamethazone. B) Stat blood cultures, then give IV antibiotics, then do a lumbar puncture, and then do a head CT. C) Stat blood cultures, then give IV antibiotics and IV dexamethazone, then do a head CT, and then do a lumbar puncture if safe. D) Stat blood cultures, then get a head CT, then do a lumbar puncture, then give antibiotics, and then give dexamethasone. E) Give dexamethasone, then intravenous antibiotics, followed by a lumbar puncture.

Answer C. Stat blood cultures, then give IV antibiotics and IV dexamethazone, then do a head CT, and then do a lumbar puncture if safe. Explanation This woman has bacterial meningitis with focal neurologic signs consistent with a central CN 7 palsy. The focal findings indicate that she may have increased intracranial pressure and might herniate with an LP. She needs antibiotics immediately but you must do a CT scan before you can do the LP. Dexamethasone is recommended (2004 IDSA guidelines) for suspected bacterial meningitis. It appears to be effective mainly in those with pneumococcal meningitis. Additionally, the dexamethasone is given before or at the same time as the antibiotics but NEVER after the antibiotics (worsens outcomes).

You are consulted to evaluate a 28-year-old Hispanic patient in the ICU after a motor vehicle accident. He was thrown from a car and sustained extensive trauma to the skin of his back, legs, and right upper extremity. His family recently immigrated to the area from Central America. Local family members were involved in the accident and are unable to provide medical information. His previous immunization status is unknown. The trauma surgeons have asked your advice regarding recommendations for tetanus prophylaxis. Which of the following immunizations should be administered at this time? A) Tdap vaccine alone B) TIG with follow-up Tdap tetanus vaccine in six weeks C) Tdap in combination with tetanus immune globulin (TIG) D) Diphtheria, tetanus toxoid, and acellular pertussis (DTaP) vaccine and tetanus immune globulin (TIG)

Answer C. Tdap in combination with tetanus immune globulin (TIG) Explanation Tetanus (lockjaw) is a neurologic disease mediated by a neurotoxin produced by the anaerobe, Clostridium tetani . This organism is ubiquitous, but is more common in wounds contaminated with dirt, feces, or saliva. Patients with burns and wounds that compromise dermal defenses (penetrating or desquamating processes) are at an increased risk of tetanus. Patients who have less than 3 prior tetanus immunizations—or patients with serious wounds and an unknown immunization status—should receive both the appropriate tetanus vaccine and TIG. Use of DTaP is restricted to children younger than 7 years of age, while individuals older than 7 years of age should receive Tdap. There are no contraindications to the coadministration of tetanus vaccine and tetanus immune globulin, except that they must be given in different body sites.

You are called to evaluate an 83-year-old female resident of a long-term care facility with long-standing dementia. Nursing staff has noted increasing restlessness, generalized scratching, and a progressive rash. These symptoms have been progressing over the last 36-48 hours. The patient is unable to describe her complaints, but is persistently scratching and seems distressed. Exam reveals significant excoriations diffusely over the entire body, with greatest involvement at the wrists and waistbands of her underclothing. No significant crusting or cellulitis is noted. Her roommate, who was not under your care, was transferred five days ago to an inpatient setting after she developed extreme skin involvement with an extensive impetiginous rash. On further examination of her hands, you notice what appear to be burrow-like structures: You do a scraping of one of the lesions and find this: Which of the following is the most appropriate therapy? A) Oral ampicillin-sulbactam B) Oral diphenhydramine C) Topical permethrin D) Topical steroids E) Topical calamine

Answer C. Topical permethrin Explanation Scabies is a parasitic infestation from the mite Sarcoptes scabiei , which causes an extremely pruritic rash. Burrows (as seen in the hand picture) are commonly noted. Transmission can readily occur from person-to-person, and outbreaks are possible in long-term care facilities, hospitals, and mental institutions. Patients with decreased mental status, compromised immune function, and glucocorticoid usage are at risk for hyper-infestation, resulting in a variant referred to as Norwegian scabies. This form is extensive and associated with diffuse crusting, as described in this patient's roommate. The responsible mite can be effectively eliminated by the use of permethrin. Itching often continues after the removal of the offending mites due to persistence of fecal pellets, which continues until the skin containing these pellets is replaced during normal dermal turnover. Oral antibiotics may be necessary for secondary bacterial involvement, but that is not described in this scenario. Topical steroids, oral diphenhydramine, and calamine lotion can provide symptomatic relief but have no effect on eradicating the offending mite.

A 25-year-old woman presents in shock. She is noted to have fever, diarrhea, and is incoherent. On examination, she has a flushed red appearance to her skin and her BP is 70/20. She is menstruating; her boyfriend denies knowledge of sexually transmitted disease. A few petechiae are noted on her extremities. Laboratory shows elevated transaminases, hypocalcemia, and an elevated PT. What is the most likely cause of her findings? A) Rocky mountain spotted fever B) E. coli sepsis C) Toxic shock syndrome D) Urinary tract infection E) Disseminated Neisseria gonorrhoeae

Answer C. Toxic shock syndrome Explanation She has all of the findings of this syndrome. Hypotension with red skin and multi-organ involvement should make you think of this—here we have GI, liver, neuro, hematologic, and electrolyte abnormalities. This form associated with tampon use (rare today) is due to Staphylococcus aureus and may also be seen associated with S. aureus soft tissue infections and pneumonia. Streptococcus pyogenes (group A Strep) also produces a similar syndrome.

An 80-year-old man lives in a nursing home. He has been hospitalized three times in the last 3 years for urosepsis. He has been forced to use a chronic indwelling catheter for incontinence after prostate surgery. He presents today with fever to 103° F and flank pain. BP is 120/78, PR 106 and RR 22. In the past he has had MRSA, Enterococcus faecalis, and Pseudomonas aeruginosa as a cause of his urosepsis. What therapy do you recommend at this time? A) Ceftriaxone and gentamicin. B) None, as this is likely just colonization. C) Vancomycin and ceftazidime. D) Vancomycin and ceftriaxone. E) Ciprofloxacin alone.

Answer C. Vancomycin and ceftazidime. Explanation He is septic and must receive antibiotics immediately. They tell you that he has had MRSA, Enterococcus, and Pseudomonas in the past and, with his indwelling catheter, you have to suspect that he is colonized with any or all of these. Therefore, you need to treat each of these in your empiric coverage. Vancomycin, remember, is the only thing listed that gets MRSA, so that is required; and is the best choice listed to also cover the Enterococcus. So, that just leaves us with vancomycin + ceftazidime or vancomycin + ceftriaxone. Ceftriaxone does not cover Pseudomonas reliably; therefore, the only choice here is vancomycin and ceftazidime.

A 65-year-old salt water fisherman in Galveston, Texas, is brought to the emergency department with fever and lethargy. He was well until 3 days ago, when he started having fever and shaking chills. He took acetaminophen and ibuprofen but did not get any better. Today, he became confused, and so he was brought in by his wife. He has chronic hepatitis C but has refused therapy to date. His diagnosis was made when he had an acute attack of jaundice 3 years ago. PAST MEDICAL HISTORY: Vietnam veteran; had malaria in 1961 SOCIAL HISTORY: Lives with wife in a small apartment Drinks a fifth of vodka a week Smokes 2 packs of cigarettes daily PHYSICAL EXAMINATION: BP 100/60, Temp 104° F, P 120, RR 25 General: He is lethargic and confused HEENT: PERRLA, EOMI TMs clear Throat: Clear Supple; no meningismus Supple; no meningismus Heart: RRR with I/VI systolic ejection murmur Lungs: CTA Abdomen: Liver down 4 cm Extremities: Scratches noted on legs and arms Skin: Numerous spider angiomas noted on trunk Several 2-cm bullous lesions noted on the trunk; appear to contain fluid Which of the following is the most likely diagnosis? A) Leptospirosis B) Staphylococcus aureus C) Vibrio vulnificus D) Invasive Streptococcus pyogenes infection E) Malaria

Answer C. Vibrio vulnificus Explanation This is a classic case: an alcoholic man with liver disease who has exposure to salt water or fish and comes in with sepsis, confusion, and bullous skin lesions. Another case is a person with liver disease who eats oysters or goes to the beach. The other choices are less likely, particularly malaria and leptospirosis. Group A Streptococcus or a Staphylococcus could do this, but his history is too classic to pick anything else but V. vulninficus for this unfortunate man.

A 35-year-old woman with AIDS (CD4 count 5) presents for evaluation. She has had HIV infection for 13 years and has done well on HAART therapy. She takes trimethoprim/sulfamethoxazole for PCP prophylaxis as well as azithromycin for MAC prophylaxis. She presents today with worsening headache, vomiting, and low-grade fever for one week. She denies vision changes. Physical Examination: HEENT: PERRLA, EOMI TMs: Clear Throat: Clear Neck: No meningismus Heart: RRR with no murmur Lungs: Few scattered crackles; cleared with coughing Abdomen: +BS, soft, no hepatosplenomegaly Extremities: No rashes noted Neurologic: She is alert and oriented x3. There are no abnormal neurologic findings. CT scan of the head with and without contrast: Normal; no evidence of increased intracranial pressure or lesions. CBC: WBC 2,500 with 50% lymphocytes Hemoglobin: 11 mg/dL Platelets: 225,000 Serum VDRL: Negative Lumbar Puncture: WBC 2 (100% lymphocytes) CSF Protein 100 mg/dL (normal 30-80 mg/dL) CSF Glucose 70 mg/dL (normal 60-90 mg/dL) Gram stain: Negative CSF VDRL: Negative Based on the above results, which of the following do you recommend at this point? A) Begin IV penicillin. B) Administer vancomycin and ceftriaxone. C) Administer ceftriaxone. D) Administer amphotericin B and flucytosine. E) Administer INH, rifampin, ethambutol, and pyrazinamide.

Answer D. Administer amphotericin B and flucytosine. Explanation The most common cause of meningitis in an AIDS patient with a low CD4 count is Cryptococcus neoformans. This would be best diagnosed in this case by sending the CSF for cryptococcal antigen. However, the findings of subacute presentations, lack of meningeal signs or altered mental status, lack of CSF pleocytosis, and a slightly elevated CSF protein are sufficient to start empiric treatment while awaiting this test. Typical bacterial meningitis (e.g., from Streptococcus pneumoniae) would have a more acute onset and is ruled out by the absent CSF pleocytosis. Tuberculous meningitis should also have a pleocytosis and almost always has a low CSF glucose. Syphilis should be tested for in the CSF by sending it for a VDRL, but therapy is not warranted until this result returns.

A 30-year-old woman with negative past medical history works in a daycare and provides care for 20 children a day. Most of the children are aged 1-3 years. Her husband brings her to the emergency room with fever and severe headache. She notes her neck is stiff and that while driving over to the hospital the sunlight "hurt her eyes." She has no known sick contacts except for the numerous runny noses and colds that the daycare children have on a daily basis. Physical examination is significant for meningismus and a positive Kernig sign. She has marked photophobia. Her optic discs are well demarcated, and no signs of intracranial pressure are noted. She has no focal neurologic signs. Which of the following treatments do you recommend? A) If lumbar puncture cannot be done immediately, you should withhold IV antibiotics until you can get the CSF to ensure a proper culture. B) A CT scan should be performed before lumbar puncture is attempted. C) Attempt a quick lumbar puncture followed by immediate administration of IV ceftriaxone. D) Attempt a quick lumbar puncture followed by immediate administration of IV vancomycin and IV ceftriaxone. E) IV acyclovir should be administered immediately.

Answer D. Attempt a quick lumbar puncture followed by immediate administration of IV vancomycin and IV ceftriaxone. Explanation She has bacterial meningitis until proven otherwise. The most common cause of bacterial meningitis today is Streptococcus pneumoniae. Unfortunately, an increasing percentage of these bacteria are becoming resistant to penicillin and cephalosporins. Therefore, vancomycin should be used in addition to ceftriaxone in any case where pneumococcal meningitis is suspected. CT scan is not indicated in this case initially since there are no focal findings and she clinically is alert and oriented. Herpes simplex is a possibility, but bacterial meningitis is much more likely. The daycare history should make you think of resistant pneumococcus also—another clue to the etiology here. Never withhold IV antibiotics in a patient with suspected bacterial meningitis. If you can't do the lumbar puncture quickly, give life-saving antibiotics as soon as possible.

A 30-year-old woman is found to be HIV-seropositive. She has no medical complaints. She was diagnosed on routine screening while donating blood. Her physical examination is normal, except she has noticed that she bruises more easily than normal. LABORATORY: WBC: 3,500/cu mm; 60% polys, 30% lymphocytes Hemoglobin: 13 mg/dL Platelet count: 40,000/cu mm CD4 count: 250 cells/mm3 HIV viral load: 150,000 copies Bone marrow examination is requested. Which of the following should you do now? A) Begin trimethoprim/sulfamethoxazole. B) Begin corticosteroid therapy. C) Begin intravenous immunoglobulin therapy (IVIG). D) Begin combination antiretroviral therapy. E) Infuse platelets.

Answer D. Begin combination antiretroviral therapy. Explanation She most likely has HIV-related immune thrombocytopenia which is very common in HIV-infected individuals. Treatment with antiretroviral therapy is often associated with a rise in platelet count. The 2014 DHHS guidelines recommend initiating antiretroviral therapy for all HIV-infected individuals regardlesss of their CD4 count. Steroids would be contraindicated in this immunocompromised patient, and the relatively normal Hb speaks agains Parvovirus B19 causing the thromboytopenia so use of IVIG is not warranted. She is not actively bleeding and her platelet count is above 30,000, so there is no need to transfuse her with platelets at this time. Her CD4 count is too high (>200) to begin trimethoprim/sulfamethoxazole.

A 35-year-old woman works as a forester in Southern Arkansas. She comes to you with a 3-month history of cough and recent development of hemoptysis. While talking to her, you discover that she has lost 10 pounds in the last 3 months. She has been smoking cigarettes for 20 years and smokes up to 2 packs a day. PAST MEDICAL HISTORY: Negative SOCIAL HISTORY: Lives in Camden, Arkansas, with her husband of 3 years 4 children ages 20, 18, 16, 1 Drinks 6-pack of beer on the weekend No illicit drug use FAMILY HISTORY: Father died at age 67 of lung cancer Mother died at age 64 of lung cancer PHYSICAL EXAMINATION: BP 110/70, P 88, Temp 100° F, RR 25, Weight 130 lbs HEENT: PERRLA, EOMI TMs clear Throat clear Neck: Supple Heart: RRR without murmurs, rubs, or gallops Lungs: Scattered crackles in the bases Abdomen: No hepatosplenomegaly Extremities: Draining lesion on her left leg Has been there for 3 months; puts "poultice" on it, and it gets better but then gets worse again LABORATORY: CXR: Pulmonary mass noted at right base Biopsy of cutaneous lesion: Broad-based budding yeast Based on your findings, which of the following is the most likely diagnosis? A) Lung carcinoma B) Tuberculosis C) Coccidioidomycosis D) Blastomycosis E) Histoplasmosis

Answer D. Blastomycosis Explanation The combination of lung mass, draining cutaneous lesion, and the finding of budding yeast should point you toward either blastomycosis or histoplasmosis, both of which are present in Arkansas. Blastomycosis has broad-based budding (4 Bs: blasto, broad, based, bud). Coccidioides is seen in the Southwest, but its tissue form is a spherule, not a budding yeast. Tuberculosis and lung cancer would not explain the yeast on skin biopsy. Treatment would be with oral itraconazole for 6-12 months. If she were severely ill, you would use amphotericin B.

A 40-year-old woman is receiving chemotherapy with cytarabine and daunorubicin for acute myelogenous leukemia. She presents to the emergency department with a one-day history of chills and fever. She is noted to have a temperature of 103° F in the emergency department, and you are called immediately. Her physical examination is completely normal except for mucositis, some pallor, and tachycardia. She appears well otherwise. Laboratory studies reveal a WBC of 0.6 without neutrophils or bands. CXR and urinalysis are normal. Blood and urine cultures are taken. Which of the following antibiotic choices is appropriate at this point? A) Vancomycin and cefepime B) Cefepime and metronidazole C) Ciprofloxacin and piperacillin/tazobactam D) Cefepime E) Cefepime and gentamicin

Answer D. Cefepime Explanation The current IDSA guidelines recommend that empiric therapy of febrile neutropenia should consist of a single beta-lactam antibiotic that is highly likely to cover aerobic, gram-negative bacilli, including Pseudomonas aeuruginosa. Dual therapy is no longer necessary and neither is anaerobic coverage. Adding empiric vancomycin has not been shown to reduce morbidity or mortality and thus should only be used when there is a high-risk of gram-positive infection specifically in patients with suspected catheter-related infection, known colonization with MRSA or resistant pneumococc, or patients with skin and soft tissue infection as the likely source. This patient has none of these. Because the current broad-spectrum beta-lactams adequately cover viridans streptococci, mucositis is no longer an indication for vancomycin.

A 45-year-old male is admitted through the emergency department with the diagnosis of community-acquired pneumonia. He presented with a three-day history of fever and chills associated with night sweats and generalized myalgias. He has developed productive cough and shortness of breath. On physical examination, the patient is noted to have moderate tachypnea and scattered wheezes and rales on chest auscultation. His pulse oximeter on room air is 85%. The chest radiograph is abnormal and reveals alveolar infiltrates with a reticulonodular pattern. Blood and sputum cultures were obtained, and the patient was started on IV azithromycin with admission for observation. As you are beginning rounds, you receive a call from pathology informing you that the sputum is most remarkable for a characteristic thick-walled yeast cell (8- to 15-mcg diameter) with broad-based daughter cells consistent with Blastomyces dermatitidis . The most appropriate action at this time would be: A) Await culture confirmation of blastomycosis. B) Change the antibiotic choice to ketoconazole. C) Add ceftriaxone to the azithromycin coverage. D) Change the antibiotic choice to itraconazole. E) Continue current antibiotic regimen.

Answer D. Change the antibiotic choice to itraconazole. Explanation Blastomycosis is a fungal pathogen most commonly found in the areas of the Mississippi and Ohio rivers as well as the Great Lakes region. The disease severity ranges from asymptomatic to life-threatening and is worsened in immunocompromised hosts. Diagnosis can be made by direct observation of the typical forms on sputum or tissue samples or through culture. Colonization by blastomycosis is not a recognized occurrence, making identification very sensitive for diagnostic identification. Itraconazole is the treatment of choice for mild to moderate infections and is superior to ketoconazole. Because of the variability in absorption, itraconazole levels should be ordered 2 weeks into therapy to make sure they are therapeutic. Amphotericin B may be required in severe cases or with immune compromise. Azithromycin, aztreonam, and ceftriaxone have no efficacy against this pathogen. Treatment for blastomycosis typically lasts 6-12 months.

You see a 46-year-old female in the emergency department with initial complaints of blurred vision, followed by difficulty with swallowing, and increasing difficulty breathing. The symptoms began several hours after a family gathering where traditional foods were consumed. Communication with family indicates that several other members are also experiencing symptoms but with fewer difficulties. The patient deteriorates and requires intubation and ventilatory support. Over the next several hours, other family members also present with worsening and similar symptoms. A child, who was at the family gathering, presents with the following flaccid paralysis: Which of the following is the most likely offending agent? A) Bacillus cereus B) Enterotoxigenic E. coli C) Shigella D) Clostridium botulinum

Answer D. Clostridium botulinum Explanation Botulism is mediated by a toxin generated by C. botulinum under anaerobic conditions. The toxin is heat labile, but may not be inactivated under standard cooking conditions. The classic presentation of botulism is a descending paralysis with involvement of extraocular and pharyngeal muscles progressing to respiratory compromise. Sensory function is unaffected, but the patient may experience confusion. Risk for botulism includes foods that are prepared in home bottling and preservation techniques that allow propagation of the botulism organism or fail to destroy formed toxin. Some traditional foods of various ethnic groups meet these criteria. Bacillus cereus usually presents with abdominal pain and diarrhea, but uncommonly produces a neurotoxin that results in a significant element of vomiting. Enterotoxigenic E. coli results in watery diarrhea without a direct neurological involvement. Shigella species may produce a variety of symptoms based on the various organism types. Patients may have diarrhea and increased fever, which in younger and susceptible individuals may result in seizure activity. Neurologic manifestations also include seizures associated with hyponatremia and hypoglycemia. Less common is encephalopathy, which presents with unusual posturing and cerebral edema. Descending paralysis is not part of the Shigella constellation of symptoms.

A 28-year-old woman, who is a health care worker in Memphis, Tennessee, is being evaluated for a 2-week history of progressive shortness of breath, dry cough, fever, and weight loss. She lives with her boyfriend who has a history of IV-drug abuse. She denies use of IV drugs and says that her boyfriend has been "clean" since last year. She does not know his HIV status, however. PAST MEDICAL HISTORY: Negative SOCIAL HISTORY: Works as a nurse's aide in local hospital Doesn't smoke or drink Lives with current boyfriend for the past 3 years, monogamous FAMILY HISTORY: Negative PHYSICAL EXAMINATION: BP 120/70, P 80, RR 28, Temp 101° F HEENT: PERRLA, EOMI TMs clear Throat: Oral thrush Neck: Mobile, nontender lymph nodes noted in the posterior cervical chain Heart: RRR with I/VI systolic flow murmur Lungs: Scattered crackles, especially prominent in left base Abdomen: Liver is 4 cm below right costal margin; mildly tender Spleen tip palpable Extremities: No cyanosis, clubbing, or edema Neuro: No deficits noted LABORATORY: WBC: 3,200/cu mm; 80% lymphocytes Hemoglobin: 9.8 mg/dL Platelets: 110,000/cu mm AST: 100 U/L ALT: 89 U/L HIV ELISA: Positive Western blot: Pending CD4 lymphocytes: 37/cu mm Which of the following is the most likely diagnosis? A) Disseminated Pneumocystis infection B) Disseminated Mycobacterium tuberculosis C) Disseminated coccidioidomycosis D) Disseminated Histoplasma infection E) Lymphoma

Answer D. Disseminated Histoplasma infection Explanation Note that the patient has AIDS. She lives in Memphis, which is endemic for Histoplasma. We can rule out coccidioidomycosis based on her geography. Disseminated Pneumocystis is rare outside of the lung. Lymphoma is possible but less likely, given her other findings. Tuberculosis also would be less likely to cause this picture. If she had a higher CD4 count—of, say, 300—we would be more concerned about tuberculosis than an opportunistic infection like Histoplasma. Back to the Histoplasma. She has disseminated disease. It is likely that if we did a bone marrow aspiration, we would find the organism there. Also, today we can do blood cultures and effectively grow the organism quite easily. The other simple test to do would be a serum or urine Histoplasma antigen. This is highly sensitive and specific for disseminated Histoplasma in HIV-infected patients.

A 28-year-old healthy man who lives in southern Missouri presents with 3 days of fever. He lives on a sheep, goat, and dairy farm. He has no lymphadenopathy, but has a slightly erythematous rash on his trunk. He likes to hunt with his brothers. He has removed numerous ticks from his body. His CBC shows a pancytopenia. A bone marrow is performed and shows the following: morula Which of the following is the most likely pathogen? A) Klebsiella B) Yersinia C) Francisella D) Ehrlichia E) Legionella

Answer D. Ehrlichia Explanation This patient has typical ehrlichiosis. If you see the words "Missouri," "tick bite," and "pancytopenia," think ehrlichiosis due to Ehrlichia chaffeensis. The bone marrow is showing the classic "morula" of Ehrlichia, which sometimes can also be seen in a peripheral smear. Of course, RMSF is endemic to the same area, but the patients in the exam will have petechial rashes around their wrists and ankles, and they will be much more ill. Other potential clinical manifestations often included in patient scenarios include hyponatremia and elevated LFTs. Both RMSF and Ehrlichia present with the pancytopenia (and possibly the sodium and LFT derangements, as well.) While Ehrlichia doesn't classically have a rash, and is called "Rocky Mountain spotless fever," it is not uncommon to see a mild diffuse erythematous rash on the trunk. But, you do not see the RMSF petechial rash. Various forms of Ehrlichia are seen also in Missouri, Wisconsin, and the Northeast. He does not live in a Lyme-endemic area or a plague area (Yersinia). Francisella would cause an enlarged lymph node with or without an ulcerated lesion and does not cause pancytopenia. Klebsiella does not cause this sort of clinical presentation.

A 72-year-old Caucasian male presents with a 3-day history of fever, malaise, and myalgias. He recently returned from a trip to Missouri, where he reports being bitten by a tick approximately 10 days ago. He was there on a fishing trip. He is healthy otherwise and reports that he was doing well. PAST MEDICAL HISTORY: No medications SOCIAL HISTORY: Retired from a medical publishing firm Never smoked Drinks 2 glasses of wine on the weekends FAMILY HISTORY: Negative REVIEW OF SYSTEMS: No rash No joint manifestations No conjunctivitis No lymphadenopathy PHYSICAL EXAMINATION: BP 120/70, P 100, RR 18, Temp 103° F HEENT: PERRLA, EOMI TMs clear Throat clear Neck: Supple Heart: RRR without murmurs, rubs, or gallops Lungs: CTA Abdomen: Bowel sounds present; no hepatomegaly; spleen tip 2 cm below left costal margin Extremities: No cyanosis, clubbing, or edema LABORATORY: WBC: 2,200 with 60% polys, 20% bands Hemoglobin: 12.5 mg/dL Platelets: 140,000/cu mm AST: 150 IU/L ALT: 140 IU/L Based on your history, physical, and laboratory values, which of the following is the most likely etiology? A) Lyme disease B) Histoplasmosis C) Tularemia D) Ehrlichiosis E) Blastomycosis

Answer D. Ehrlichiosis Explanation This patient has fever and nonspecific symptoms. He has splenomegaly. He has pancytopenia with elevated liver transaminases and has visited an endemic area, Missouri. This clinical picture goes with ehrlichiosis. For tularemia, you would expect lymphadenopathy and not expect to find a pancytopenia. Histoplasma and Blastomyces would usually present with pneumonia and/or skin lesions. Missouri is not endemic for Lyme disease, and he lacks the most common presenting symptom of Lyme disease—erythema migrans.

A 35-year-old man underwent a heart transplant 5 days ago. He is receiving immunosuppressive therapy with methylprednisolone, cyclosporine, and azathioprine. Today, he develops a temperature of 102° F. PHYSICAL EXAMINATION: Ill-appearing man on the ventilator since surgery BP 130/50, Temp 102° F, RR 30, P 100 SIGNIFICANT FINDINGS: Chest: Crackles and rhonchi heard over the right lung fields LABORATORY: Tracheal secretions are now yellow FiO2 requirements have increased from 35% to 50% Pulmonary artery wedge pressure is 15 mmHg (normal 6-12) WBC: 17,000/cu mm with 90% neutrophils CXR: Dense consolidation in right middle and lower lobes Which of the following is most likely the etiology for his pneumonia? A) CMV B) Legionella pneumoniae C) Cryptococcus neoformans D) Pseudomonas aeruginosa E) Pneumocystis jiroveci

Answer D. Pseudomonas aeruginosa Explanation Note: He is only 5 days out from his transplant. The most common organisms to cause problems this early are hospital-acquired infections, particularly with gram-negatives like Pseudomonas or gram-positives like Staphylococcus aureus. CMV and Pneumocystis are likely 1-4 months out. Cryptococcus is a problem more often 4 or more months out. Legionella doesn't have much more increased incidence, unless there were something wrong with the processing of water in the hospital.

A 40-year-old woman is vacationing in Acapulco, Mexico. She eats salad and tacos at a local café and drinks multiple margaritas during her stay. While on the trip, she develops watery, nonbloody diarrhea. Additionally, she has cramping abdominal pain that is relieved with a bowel movement. She has low-grade temperatures between 99° and 100° F for a day or two. Her symptoms resolve without any treatment after 4 days of illness. You see her in follow-up after the trip. Her physical examination is normal. She has no symptoms now. Which of the following organisms was most likely responsible for her illness? A) Campylobacter jejuni B) Shigella sonnei C) Escherichia coli O157:H7 D) Enterotoxigenic Escherichia coli E) Rotavirus

Answer D. Enterotoxigenic Escherichia coli Explanation She had traveler's diarrhea. This is usually a nonbloody diarrhea that resolves without treatment. Eating leafy vegetables or uncooked foods will expose travelers to this organism. Additionally, ice and water are common sources, especially since travelers mistakenly think that ice is protected. Also, a few people feel like the alcohol in a mixed drink will kill the bacteria—but it doesn't work that way. The other bacteria listed would generally cause much longer symptoms and more extensive disease. Remember: With E. coli O157:H7, do not give antibiotics because of the perceived increased risk of hemolytic-uremic syndrome. Rotavirus is unlikely, and you would expect it more often in a daycare setting or with exposure to children, and it would not be likely in a "travel" question.

A 17-year-old girl presents with a 5-6-day history of malaise, headache, sore throat, and increased temperature. Findings on physical examination include periorbital and eyelid edema unassociated with changes in the conjunctiva, petechial lesions at the junction of the hard and soft palate, and an erythematous maculopapular morbilliform-like eruption on the trunk, upper arms, and thighs. Which of the following is the most likely cause of her clinical signs and symptoms? A) Human herpesvirus 6 B) Parvovirus B19 C) Coxsackievirus A16 D) Epstein-Barr virus E) Adenovirus

Answer D. Epstein-Barr virus Explanation She has clinical signs and symptoms often associated with infectious mononucleosis, caused by Epstein-Barr virus. Periorbital and eyelid edema is an early clinical finding in up to 50% of patients with this disorder. Although a rash develops in a large proportion of patients with mononucleosis treated with ampicillin or amoxicillin (usually for presumed streptococcal infection), up to 10-15% of patients will develop a morbilliform-like rash as described in the vignette in the absence of any antimicrobial treatment. An enanthem, characterized by petechial lesions at the junction of the soft and hard palates, may also be seen in up to 25-30% of patients. Additional clinical findings may include severe membranous tonsillitis, anterior and posterior cervical lymphadenopathy, splenomegaly, hepatomegaly and jaundice secondary to icteric hepatitis. Large basophilic-staining lymphocytes containing a "foamy-like" cytoplasm (atypical lymphocytes) are often identified on peripheral smear. Rapid diagnosis is made by the rapid Monospot test which identifies heterophil antibodies produced in infectious mononucleosis. However, the rapid test may be negative early in the course of the disease. In general, treatment is symptomatic only, with avoidance of contact sports and other activities which could lead to increased risk of splenic rupture, following abdominal trauma, for 4-6 weeks. Corticosteroids should be avoided unless there is concern about significant upper airway obstruction.

Which of these antibiotics is effective for gram-negative killing even after the drug has fallen below the MIC? A) Aztreonam B) Ceftazidime C) Erythromycin D) Gentamicin E) Vancomycin

Answer D. Gentamicin Explanation What is described here is the "post-antibiotic effect." The aminoglycosides will continue to be bactericidal even when their levels fall below the MIC for the particular gram-negative bacteria. This is why once-daily administration is so effective even with low levels near trough. Azthreonam and ceftazidime do not demonstrate this effect. Note that vancomycin does not get any gram negatives, and erythromycin is a poor gram-negative drug also.

A 19-year-old college student is brought into the emergency department by his roommates because they were unable to awaken him this morning. They report that he has not had any alcohol for the past 3 months and is a model student. During the past 2 to 3 days, however, they say that he has exhibited bizarre behavior and has been intermittently confused. He takes no medications, and his friends adamantly deny that he has ever used any type of illicit drug. PAST MEDICAL HISTORY: Several visits to the emergency department for "the drip" SOCIAL HISTORY: Majoring in Interior Design PHYSICAL EXAMINATION: Temp 102° F, P 100, RR 22, BP 120/70, Ht: 5'3", Wt: 260 lbs General: Responds only to deep pain HEENT: PERRLA, EOMI TMs clear Throat clear Neck: Supple Heart: RRR without murmurs, rubs, or gallops Lungs: CTA Abdomen: Bowel sounds present, no hepatosplenomegaly Extremities: No cyanosis, clubbing, or edema; no rash Neuro: No focal neurologic signs LABORATORY: CBC: Normal Electrolytes: Normal MRI of head: Focal lesion at the base of the left temporal lobe with mild edema CXR: Normal Lumbar puncture below: WBC: 80 WBC/cu mm (50% neutrophils, 50% lymphocytes) RBC: 10 RBC/cu mm Protein: 43 mg/dL (normal) Glucose: 60 mg/dL (plasma glucose 80 mg/dL) Gram stain: Negative Based on your findings, which of the following is the likely diagnosis? A) Neurosyphilis B) Varicella meningoencephalitis C) Bartonella henselae infection D) Herpes simplex meningoencephalitis E) Streptococcus pneumoniae meningitis

Answer D. Herpes simplex meningoencephalitis Explanation The key here is the "bizarre behavior" and the findings on MRI of temporal lobe involvement. To help you diagnose this, a PCR for herpes simplex virus DNA could be helpful on the CSF. CSF cultures for herpes are rarely positive, except in the severely immunocompromised or in neonates. He certainly is at risk for syphilis, but neurosyphilis takes decades to occur and is rare in teenagers unless they are HIV-infected. You would, though, order a CSF VDRL on this patient just because he is at high risk based on his past history. Bartonella henselae is the etiology for cat-scratch disease (for which he has no exposure history) and does cause significant CNS disease on occasion; usually it will present with seizures, which is likely with herpes encephalitis too. He has no rash consistent with varicella, and this does not look like bacterial meningitis with the normal protein and glucose.

You are asked to consult on a 26-year-old female who is 4 months pregnant and reported to the emergency department with concerns regarding meningitis exposure. The patient moved this week to your area to stay with her parents for the duration of her pregnancy. She was notified that her boyfriend, with whom she was in intimate contact with up to seven days ago, was hospitalized for meningococcal meningitis. Her family was contacted by the hospital recommending evaluation for his close contacts. The information conveyed indicated his cultures were (+) for Neisseria meningitides and was identified as serotype B. Due to her contact history, which of the following is the most appropriate recommendation? A) Administer tetravalent polysaccharide meningococcal vaccine. B) Advise no treatment and close observation only. C) Prescribe prophylactic rifampin therapy. D) Institute prophylactic ceftriaxone therapy. E) Recommend prophylactic ciprofloxacin therapy.

Answer D. Institute prophylactic ceftriaxone therapy. Explanation Prophylactic antibiotics are recommended for close contacts of index cases of meningococcal disease. Effective agents include rifampin, ceftriaxone, and ciprofloxacin. Rifampin is the drug of choice for children, but is not recommended in pregnant females. Ciprofloxacin is also contraindicated in pregnancy. The most appropriate recommendation would be for prophylactic ceftriaxone. To institute no therapy in this situation would not be appropriate. Vaccination may be helpful as an adjunct and can reduce meningococcal carriage, but is not adequate alone in an acute exposure due to the delay in effectiveness. Of note as well is the fact that tetravalent meningococcal vaccine covers serogroups A, C, Y, and W-135, but does not cover type B. Also the conjugate vaccine is preferred over the polysaccharide for patients older than 11 years of age.

A 35-year-old IV drug abuser presents with headache and fever. He is incoherent and you have difficulty getting a good history from him. He was in the ED 2 weeks ago for an alcohol binge; HIV testing at that time was negative. A CT scan is negative, and IV ceftriaxone and IV vancomycin are given in the ED before the CT scan. A lumbar puncture is performed and shows the following results: CSF WBC 100 cells: 60% polys and 40% lymphs CSF Protein: 170 mg/dL CSF Glucose: 70 mg/dL with serum glucose of 100 mg/dL CSF VDRL is negative. CSF cryptococcal antigen is negative. Which of the following is the most likely of all those listed? A) Bacterial meningitis B) Carcinomatous meningitis C) Fungal meningitis D) Menigovascular syphilis

Answer D. Menigovascular syphilis Explanation Bacterial meningitis rarely has a CSF WBC of only 100 on presentation. Fungal meningitis is uncommon in HIV - patients, and the most common fungal meningitis, cryptococcal, almost always has a (+) cryptococcal antigen. Carcinomatous meningitis usually occurs in older individuals with a known primacry malignancy. IVDU is a risk factor for STDs given the common practice of sex for drugs transactions. Meningovascualr syphilis presents as an aseptc meningitis and the CSF VDRL is only about 50% sensitive.

A 50-year-old woman from Texas reports having experienced several tick bites in the last 3-4 years while camping in the Texas Panhandle. She presents with complaints of "chronic fatigue" and believes she has Lyme disease. She denies a rash or arthritis, and she has never been hospitalized for meningitis or carditis. Her only complaint is the fatigue, and she requests that you perform testing for Lyme disease. Which of the following is the most appropriate next step in diagnosis? A)B. burgdorfer i IgM, IgG, and PCR. B) B. burgdorferi IgG. C) B. burgdorferi IgM and IgG. D) No additional testing is recommended. E) PCR for B. burgdorferi .

Answer D. No additional testing is recommended. Explanation This is very important! Physicians are commonly ordering tests for Lyme disease inappropriately, so this has become a very, very common subject for Medicine Boards. If a patient presents with only "fatigue," never order Lyme testing of any kind. The Infectious Diseases Society of America's clinical guidelines for diagnosis of Lyme disease requires 1) a disease consistent with Lyme disease (not just fatigue!) and 2) exposure to a Lyme-endemic area. At this time, Texas is not a Lyme-endemic area. And to warrant further testing, she should have something else to make you think of Lyme—arthritis, Bell's palsy, or carditis. A nonspecific symptom like "fatigue" does not warrant an investigation for Lyme. To clarify, the classic erythema migrans rash of Lyme is so characteristic that if you see it, and the history is appropriate for exposure to possible Lyme, go ahead and treat as though the rash is due to Lyme. Serology is frequently not positive at the time of the rash. Additionally, serology is not very good, and false positives are common. Polymerase chain reaction has only been useful in cases of Lyme arthritis, and positive results are not consistent. In cases of known Lyme meningitis, PCR testing has been negative. Hence, PCR testing is not recommended. Check Lyme serology only in a patient who has clinical criteria for Lyme and has been in a Lyme-prone area!

A 30-year-old man works as a nurse on the wards in your hospital. He spilled a urine specimen from an HIV-infected patient on his hands 30 minutes ago. The patient has end-stage AIDS with a CD4 count of 1/cu mm and a viral load of 1 million copies/cc. The patient has been on zidovudine/lamivudine/indinavir for the past year with failure noted because of poor adherence to the regimen, although a recent genotype showed resistance patterns developing to zidovudine and indinavir! The patient is hepatitis B-surface-antigen-negative, and the nurse has received a series of 3 doses of hepatitis B vaccine, which was completed last year when he finished nursing school. The nurse notes that he has a healing laceration from a cat scratch on his left hand, and that the urine splashed onto this lesion. The lesion is well scabbed over. As soon as he spilled the urine on his hands, he scrubbed them meticulously. Which of the following do you recommend for this nurse? A) Zidovudine, lamivudine, and indinavir for 4 weeks. Lamivudine alone is sufficient. B) A regimen of using agents other than zidovudine and indinavir is indicated because of the known resistance patterns. C) No prophylaxis is indicated. D) Test the remaining urine for HIV RNA and start treatment if the viral load is detectable

Answer D. No prophylaxis is indicated. Explanation The source patient's HIV viral load is really high, so we would offer him the 3-drug therapy list of zidovudine, lamivudine, and indinavir for 4 weeks. No data exists (at least as of February 2009) on whether you should change therapy based on genotypes. Some authorities currently recommend considering it. Remember: For large amounts of infectious fluid exposure with a high viral load, use 3-drug prophylaxis! For noninfectious fluid or fluid that is in contact with "intact skin only," no prophylaxis is indicated. Note that urine is not on the list of body fluids with which to be worried about transmission. Blood, semen, vaginal secretions, CSF, synovial, pleural, peritoneal, pericardial, and amniotic fluids are considered "potentially infectious." Let's say this was blood in a cup. He would have exposure to a large volume of blood to what would be considered "skin with integrity potentially compromised."

A 65-year-old woman has a 30-year history of insulin-dependent diabetes mellitus. She is currently in the hospital for pneumonia due to Streptococcus pneumoniae. A colleague admitted her, and you are covering the weekend. You notice that she still has an indwelling Foley catheter that has been in since her admission 5 days ago through the emergency department. You remove the catheter and send a urinalysis. She complains of dysuria but is afebrile. PAST MEDICAL HISTORY: Negative except for above SOCIAL HISTORY: Works as a librarian in the medical school library Lives with her partner, Louise Drinks 1 beer a day for 40 years Doesn't smoke PHYSICAL EXAMINATION: Improving pneumonia with decreased crackles in right base; otherwise unremarkable LABORATORY: Urinalysis: 100 WBCs/hpf Gram stain shows numerous neutrophils with budding yeast; no bacteria Serum creatinine is 1.5 mg/dL Which of the following is the most appropriate management? A) Intravenous amphotericin B B) Oral amphotericin B C) Re-insertion of the indwelling urethral catheter for bladder irrigation of amphotericin B D) Oral fluconazole E) Intravenous fluconazole

Answer D. Oral fluconazole Explanation She has cystitis with yeast likely to be sensitive to fluconazole and more than likely due to the urethral catheter that should have been removed days ago. Fluconazole has > 90% oral bioavailability and is excreted in the urine, so it would not need to be given IV to treat cystitis. You would not subject this woman to IV amphotericin B, and oral administration works only for oral and esophageal candidiasis because it is not absorbed. Re-insertion of the indwelling urethral catheter is not a good idea, because you will likely just perpetuate her symptoms with continued colonization of her bladder.

A 40-year-old man presents with stiff neck and fever of 1 hour duration. PMH: Negative Family History: Noncontributory Physical Examination: BP 110/70, P 100, RR 30, Temp 103° F He is in moderate distress from his headache. Exam is significant for: Meningismus, Brudzinski, and Kernig signs are present. His optic discs are sharp and the rest of the neurologic examination is normal. After ordering blood cultures, which of the following sequences would you do? A) Order IV ceftriaxone and IV vancomycin, then head CT scan, then lumbar puncture B) Head CT scan, then lumbar puncture, then order IV dexamethasone and ceftriaxone C) Lumbar puncture, then order IV dexamethasone and ceftriaxone D) Order IV dexamethasone, ceftriaxone, and vancomycin; then lumbar puncture E) Order IV vancomycin, then lumbar puncture

Answer D. Order IV dexamethasone, ceftriaxone, and vancomycin; then lumbar puncture Explanation The headache, fever, and meningeal signs are indicative of meningitis and ordering of empiric antibiotics should be done immediately. Dexamethasone should also be administered to decrease the inflammatory response to killed bacteria. This has been shown to decrease morbidity and mortality in patients with Streptococcus pneumoniae meningitis—the most common bacterial pathogen causing meningitis in adults. CT scans are only indicated in those patients who have findings predictive of a possible space occupying lesion. Such predictors are: focal neurologic deficits, altered mental status, seizures in last week, age ≥ 60, immunocompromised state, or known prior central nervous system disease. None of these are present in this patient, so a CT scan is not indicated and could potentially delay the administration of antibiotics. In most settings, the prevalence of resistance of S. pneumoniae to cetriaxone is high enough to warrant the addition of vancomycin until the organism's sensitivity to antibiotics is known. Vancomycin alone would not treat other potential pathogens such as Neisseria meningitidis.

A 30-year-old man comes to the emergency department with increasing inability to walk without holding on to something. He denies any other complaints. PAST MEDICAL HISTORY: Negative SOCIAL HISTORY: Works as a bellhop in a local hotel Admits to having multiple sexual partners (both male and female) Smokes 2 packs/day of cigarettes Drinks a 6-pack of beer daily FAMILY HISTORY: Noncontributory PHYSICAL EXAMINATION: BP 130/70, P 90, Temp 99° F, RR 18 HEENT: PERRLA, EOMI TMs clear Throat clear Neck: Supple; no meningismus Heart: RRR without murmurs, rubs, or gallops Lungs: CTA Abdomen: Bowel sounds present, no hepatosplenomegaly Extremities: Benign GU: No lesions Neurologic: Romberg sign is present; possible decreased position sense in lower extremities LABORATORY: MRI of head: Normal WBC: 2500/cu mm; 60% polys, 30% lymphs CD4: 160/cu mm HIV ELISA: Pending Electrolytes: Normal Renal panel including creatinine and BUN: Normal Serum VDRL: Positive at 1:32 Serum fluorescent treponemal antibody test (FTA-ABS): Positive LP Results are below: CSF WBC: 50 WBCs/cu mm; 65% lymphocytes CSF Protein: 150 mg/dL CSF Glucose: 60 mg/dL (plasma glucose 90 mg/dL) CSF VDRL: Negative Based on your findings, which of the following is the appropriate treatment? A) Benzathine penicillin G, 2.4 million units IM, single dose B) Penicillin G, 1 million units intravenously q 6 hours C) Vancomycin, 1 gram intravenously q 12 hours and ceftriaxone, 2 grams IV q 24 hours D) Penicillin G, 3 million units intravenously q 4 hours E) Benzathine penicillin G, 2.4 million units IM, q week x 3 weeks

Answer D. Penicillin G, 3 million units intravenously q 4 hours Explanation This patient has neurosyphilis. But, the CSF VDRL was negative. How can he have neurosyphilis? Remember that the CSF VDRL is very specific (meaning that if it is positive, it means they truly have neurosyphilis), but it is not very sensitive (only about 1/2 of patients with documented neurosyphilis will have a positive CSF VDRL!). The serum VDRL and FTA-ABS were both positive. If you do an LP on someone with neurologic disease and they have a positive RPR or VDRL on their serum that is relatively high, then you are going to have to consider that they might have neurosyphilis. He has meningitis by definition with 50 WBCs in his CSF, and he has an elevated abnormal protein in the CSF. Both go along with neurosyphilis. And, he also has AIDS, which he didn't know he had before coming to see you. Note: Always look for syphilis in a young person with hearing loss or in an adolescent with alopecia!

A 40-year-old man is followed for tuberculosis. He began therapy about 1 month ago and is taking the standard 4-drug regimen of INH, rifampin, pyrazinamide (PZA), and ethambutol. He is in for routine screening lab, because he had some abnormal liver function tests at the time of his initial diagnosis. Everything is normal except for a uric acid of 11 mg/dL. Which of the following drugs is most likely responsible for this finding? A) Rifampin B) Ethambutol C) INH D) Pyrazinamide

Answer D. Pyrazinamide Explanation Note that hyperuricemia is common but gout is rare. PZA along with INH and rifampin may cause hepatitis also. INH can cause peripheral neuropathy. Rash is seen with PZA and ethambutol; and with ethambutol, the thing to look out for is optic neuritis, particularly manifested by changes in vision, especially with colors. This is rare if the dose of 15 mg/dk/d is used.

A 30-year-old woman lives in Houston, TX. She frequently buys fresh produce at a local stand set up by immigrant workers from the area south of Houston. She likes the selection, and the prices are very good compared to supermarket prices. Unfortunately, after eating one of these selections, she develops severe diarrhea as does her family of 7. Cyclospora is found in her stool. Based on this history, which of the following is the likely source for the Cyclospora? A) Her cat B) Milk C) Green beans D) Raspberries E) Fresh chicken

Answer D. Raspberries Explanation It is associated with eating berries—especially raspberries that come from Central America, and most outbreaks have occurred in southern Texas.

A nurse on the HIV unit of the local hospital presents to your employee health department. Today he is drawing blood on a patient with HIV and accidentally stuck himself with the needle he used to draw the blood. He quickly washed the hand thoroughly and reports to employee health. Which of the following should happen next? A) No therapy is needed, test nurse for HIV antibody in 4-6 weeks. B) Begin interferon alpha-ribavirin. C) Start zidovudine only. D) Start ZDV, 3TC, lopinavir/ritonavir. E) Draw viral loads on the patient and, if undetectable, start Combivir® only.

Answer D. Start ZDV, 3TC, lopinavir/ritonavir. Explanation A needle that has been in contact with the blood of an HIV infected person is considered a high-risk needle and requires at least 3-drug treatment as soon as possible. Combination drugs as listed here are used frequently and recommended by the CDC.

Physicians are required to report certain communicable diseases to appropriate public health agencies. Which of the following conditions is designated as a notifiable disease on the national level in the United States? A) Genital herpes B) Human papilloma virus C) Trichomonas D) Syphilis E) Gardnerella

Answer D. Syphilis Explanation Reporting of communicable disease is an important public health function. It allows intervention to the spread of serious diseases and allows the monitoring of rates and locations of specific diseases. The national list is created in consultation with the CDC and is updated on a regular basis. In addition, individual states and local health departments occasionally impose additional reporting requirements for their specific jurisdiction. Although all of the pathogens listed may require treatment and several involve treatment of contacts, only syphilis (in all of its forms) is required at the national level for reporting. Gonorrhea and Chlamydia trachomatis are also included on the list of reportable diseases.

A 52-year-old man is being evaluated because of a one-week history of lower extremity weakness, new onset of difficulty speaking, and decreased attention span (no jokes from the women out there about, "How would a man know if he had a decreased attention span?"). He has had occasional diarrhea and abdominal pain in the last year. Of significance is that he has lost about 25 lbs during the past year. He complains of joint pains, particularly in his knees. He has had low-grade fever but no chills during the last year. He reports occasional night sweats. He has noted no other neurologic findings like seizures. His wife reports that areas of his skin are becoming darker—particularly those exposed to light. Past Medical History: Healthy before this Social History: Lives in Florida Works as a bouncer for a teen-club Family History: Negative Physical Examination: BP 130/80; T 99.9; P 84; RR 18 General: Alert, but oriented only to person and place HEENT: PERRLA, EOMI Mild facial droop Throat: Clear Neck: Scattered lymphadenopathy in the anterior and posterior cervical chains; most nodes are 1x1 cm but a few are 2x1 cm Heart: RRR without murmurs, rubs, and gallops Lungs: CTA Abdomen: Spleen tip palpated; no hepatomegaly; a questionable abdominal mass discerned with deep palpation Extremities: No cyanosis, clubbing, or edema Neuro: Right lower extremity with increased tone and 4/5 muscle strength Sensation is normal Deep tendon reflexes are symmetrical Laboratory: Hemoglobin: 16.0 g/dL; Hematocrit: 52% WBC: 29,000/mm3; 65% neutrophils, 28% lymphs ESR: 13 mm/hr Glucose 200 mg/dL Albumin 3.7 g/dL ALT 32 U/L; AST 23 U/L CT of the head shows a hypodense left frontal lobe lesion. A stereotactic brain biopsy is taken and shows acute inflammation and necrosis with no malignant cells. Gram stain shows 1+ WBCs and no organisms. Periodic acid-Schiff (PAS) staining shows multiple PAS-positive foamy macrophages. Which of the following organisms is likely responsible for his condition? A) Coxiella burnetii B) Mycobacterium tuberculosis C) Nocardia asteroides D) Trophermyma whippleii E) Actinomyces israelii

Answer D. Trophermyma whippleii Explanation Trophermyma whippleii is the causative agent of Whipple's disease. Patients present after a long time with the main symptoms of weight loss, diarrhea, and arthropathies. In nearly 75% of the cases, these 3 occur together. The GI symptoms usually begin later but frequently lead to the diagnosis. Usually they will present with episodic and watery diarrhea or steatorrhea and are accompanied by abdominal pain. In about 20-30% they will have occult blood in their stool. Skin hyperpigmentation is frequently common to the sun-exposed areas. Just about every organ in the body may be infected with this bacteria. The diagnosis is usually made by upper endoscopy, but biopsy of lymph nodes or masses will also yield the diagnosis. Upper endoscopy will show whitish or yellowish plaques distributed on a friable mucosa. The key here is the PAS staining. PCR is also now available and would be useful for confirming this. With PAS staining, be careful because a few other conditions in the colon or rectum would show you this—for example, melanosis coli. Treatment of this is with trimethoprim/sulfamethoxazole, but severely ill patients like this guy should receive parenteral ceftriaxone for a prolonged period of time. Amazingly, chloramphenicol (that old antibiotic) would also be a good consideration for CNS disease because of its great penetration.

36-year-old woman lives in Bogue Holler, Arkansas, on the Buffalo River. She works as a schoolteacher in the local elementary school that has an enrollment of 15. Last week while "gettin" the firewood for the classroom stove, she noticed that a tick was embedded in her right leg. She removed the tick and didn't think anything about it until 3 days later, when she noticed that she had a marked irritation and "swellin" in her groin. That evening she developed fever and severe malaise. She applied a poultice of sprouts, cayenne pepper, and Tabasco sauce, but it didn't get any better. She presents to you today for evaluation. Physical exam reveals an ulcerated area at the tick bite with a 3x5 cm right inguinal lymph node. She has a temperature of 103.2° F. Besides that, she has poor dentition. Which of the following is the most likely diagnosis? A) Ehrlichia B) Plague C) Bartonella henselae infection D) Tularemia E) Lyme disease

Answer D. Tularemia Explanation Tularemia is endemic to Arkansas, Missouri, and Oklahoma. It has 6 distinct forms, including ulceroglandular (as in this patient), glandular (no ulcer at tick bite), oropharyngeal (from ingestion of organisms through meats, etc.), oculoglandular (usually a hunter who has been cleaning game and self-inoculates him/herself), typhoidal (just a bacteremia without glandular involvement), and pneumonic (pneumonia with the organism). Treatment is with doxycycline or gentamicin/streptomycin. It is an organism to consider with biological warfare also.

A 40-year-old woman from Connecticut presents with a history of subjective fever, myalgia, headache, and a rash, which she first noticed the day prior to presentation. On physical examination, her temperature is 100.6° F. A 3 x 5-cm circular erythematous lesion with central clearing is present on the left upper thigh. A similar but smaller lesion is located just below her left knee. Which of the following complications often occurs later in the course of this illness in patients who do not initially receive appropriate treatment? A) Symmetric proximal lower extremity weakness B) Lower back pain associated with paresthesias in the lower extremity and urinary retention C) Temporary loss of vision associated with hemiparesis D) Unilateral facial nerve palsy E) Miosis, ptosis, and anhidrosis

Answer D. Unilateral facial nerve palsy Explanation She has clinical signs and symptoms consistent with Lyme disease. The causative agent, Borrelia burgdorferi , is transmitted to humans by the bite of infected ticks belonging to certain species of the genus Ixodes . The characteristic skin rash, erythema migrans, consists of one or more erythematous, circular, outwardly expanding painless lesions often associated with central clearing. Most individuals have associated flu-like symptoms including malaise, generalized myalgia, increased temperature, photophobia, and headache. If untreated, the rash lasts 2-3 weeks followed by an array of discrete symptoms including recurrence of the rash, unilateral or bilateral facial nerve palsy (Bell's palsy, which usually resolves in 1-2 months), nighttime radicular pain unassociated with sensory or motor defects, carditis and arrhythmias associated with AV block, meningitis, and meningoencephalitis. Many individuals go on to develop an inflammatory arthritis involving one or more joints, sometimes leading to chronic arthritis, commonly in the knee(s), unassociated with destructive changes. Arthritic symptoms usually begin some 6 months after the initial rash. Treatment recommendations for patients with erythema migrans include oral therapy with doxycycline.

A 60-year-old alcoholic male visits the gulf coast of Texas on a frequent basis and enjoys eating raw oysters and drinking large quantities of beer. This year, he rented a fishing house on the beach and wades out daily to fish in the Gulf of Mexico. He presents via ambulance with a 30-minute history of "feeling weak" and passing out. He is found by a friend in his home. He has skin lesions on his leg. His blood pressure is 76/20 with a pulse of 120. He is in presumed septic shock. Which of the following is the likely organism causing his sepsis? A) Vibrio cholera B) Clostridium difficile C) Yersinia enterocolitica D) Vibrio vulnificus E) Pseudomonas aeruginosa

Answer D. Vibrio vulnificus Explanation Look for a septic alcoholic male who visits the gulf coast and has exposure to oysters, gulf water, or both. Skin lesions are common and look like boils or bullous lesions. Hypotension is quite common.

A healthy 25-year-old woman works as a lifeguard at the local swimming pool. She comes to your office because of redness and tearing of her right eye. She denies pain, saying she is experiencing only slight discomfort. She says that there are several kids and adults at the pool with obvious red eyes. She also says it is hard for her to differentiate whether their eyes are red "from chlorine or from diseases." Review of systems is negative for diarrhea, abdominal pain, skin rashes, and fever blisters. On exam, her right eye demonstrates conjunctival injection without obvious ulceration. There is increased lacrimation. Pupillary response and extraocular movements are normal. Fluorescein stain of her eye reveals no abnormalities. Which of the following is the most likely pathogen? A) Mycobacterium marinum B) Vibrio vulnificus C) Mycobacterium tuberculosis D) Histoplasma capsulatum E) Adenovirus

Answer E. Adenovirus Explanation If you selected Mycobacterium marinum , you could be forgiven—"swimming pool" was in the question. But there is no mention of a skin lesion. Be careful when the Board question gives you an epidemiologic clue such as geography or a specific location (like "swimming pool"), because several organisms might be associated with that clue. In this case, conjunctivitis with an epidemic-like outbreak associated with swimming pools is due to adenovirus. These outbreaks are most common in the summer, hence, the association with swimming pools. Other examples of important epidemiologic information: Arkansas + lymph node = tularemia; Arkansas + pancytopenia = ehrlichiosis; Arkansas + pneumonia + skin lesions = blastomycosis. So read the whole question and pay attention to the presentation of the infection before you jump to the most likely infection associated with the epidemiologic clue! And unfortunately, sometimes the epidemiologic clue is just a red herring to steer you in the wrong direction—make sure the clinical history matches the suspected pathogen!

You are seeing a 30-year-old man with AIDS whom you have been following for years and recently have noticed that his CD4 count has continued to fall. He has not had any opportunistic infections and has done well with his antiretroviral therapies. You feel that he has most likely developed a more resistant HIV infection. He is taking trimethoprim/sulfamethoxazole for Pneumocystis prophylaxis. His physical examination is unremarkable. LABORATORY: CD4 lymphocyte count is 28/cu mm Viral load is 350,000 copies/mL Because of his worsening immunosuppression, which of the following bacterial infection prophylaxis regimens do you recommend he start? A) Rifampin daily B) Nothing C) Clarithromycin weekly D) Rifabutin and clarithromycin daily E) Azithromycin weekly

Answer E. Azithromycin weekly Explanation His CD4 count has fallen below 50, putting him at risk for Mycobacterium avium complex (MAC) infection. Of the choices listed, only the azithromycin weekly is currently approved for prophylaxis. Clarithromycin could be used, but he would have to take it daily. Rifampin is not approved for MAC prophylaxis. A rifabutin and clarithromycin combo would be used for treatment, not prophylaxis. The choice of "nothing" is incorrect, because he really is at high risk for MAC infection.

A 60-year-old woman with AML is undergoing chemotherapy. She finished her current round of chemotherapy 2 weeks ago and has been neutropenic for about a week. She was admitted last week with fever and has been on piperacillin/tazobactam and ciprofloxacin for this time period. Today, a blood culture from 3 days ago is growing a yeast subsequently determined to be Candida krusei. CT of the abdomen shows no lesions. She had a central venous catheter placed 2 weeks ago with her chemotherapy. PHYSICAL EXAMINATION: BP 130/70, P 80, RR 18, Temp 102° F General: Well appearing; mild distress with fevers and chills HEENT: PERRLA, EOMI Discs examined by ophthalmologist and are normal Throat clear; no thrush Heart: RRR without murmurs, rubs, or gallops Lungs: CTA Abdomen: Bowel sounds present; no hepatosplenomegaly Extremities: No rashes; no cyanosis, clubbing, or edema GU: Normal; no rectal abscesses Skin: Insertion site for central venous catheter looks clean and nontender; no evidence of infection Which of the following do you recommend? A) Do not change the catheter site; add intravenous fluconazole. B) Change the catheter site; add intravenous fluconazole. C) Do not change the catheter site; add caspofungin. D) Continue current therapy; Candida krusei is a contaminant. E) Change the catheter site; add caspofungin.

Answer E. Change the catheter site; add caspofungin. Explanation She has infection with Candida krusei, which is almost always resistant to fluconazole and has reduced susceptibility to amphotericin B. Caspofungin and the other echinocandins (micafungin, anidulafungin) have been shown to be as effective and better tolerated than amphotericin B for Candida krusei infections. Literature supports removal and replacement of a central venous catheter if catheter tip cultures are positive for a Candida species or other fungi. Never, never, never consider a Candida blood culture result as a contaminant. Remember: With disseminated Candida, you do want to look for endophthalmitis and liver and splenic lesions, which she did not have at this time. Note that sometimes the liver/spleen lesions may not occur until after the neutropenia has resolved!

A 30-year-old man was running through the cornfields near his home when he sustained a puncture wound from a nail that pierced his tennis shoe. Six days later, his foot is swollen, and he develops fever. A bone scan confirms osteomyelitis. Which of the following is the most appropriate empiric pharmacotherapy? A) Nafcillin B) Trimethoprim/sulfamethoxazole C) Tetracycline D) Ceftriaxone E) Ciprofloxacin

Answer E. Ciprofloxacin Explanation The most likely etiologic agent for the infection is Pseudomonas . You are given the classic history of a "nail through the tennis shoe." Pseudomonas is known to colonize the glue used to adhere leather in tennis shoes. Ciprofloxacin would have very good coverage and has good bone penetration. Trimethoprim/sulfamethoxazole, ceftriaxone, nafcillin, and tetracycline do not cover Pseudomonas .

A 70-year-old man presents to your office with symptoms of epididymitis. He denies sexual activity, especially since his sildenafil citrate prescription ran out 6 months ago. He is monogamous and has never had a sexually transmitted disease. Which of the following is the most likely pathogen? A) Neisseria gonorrhoeae. B) Chlamydia trachomatis. C) Pseudomonas aeruginosa. D) Chlamydia trachomatis or Neisseria gonorrhoeae is equally likely. E) E. coli.

Answer E. E. coli. Explanation In men over the age of 35, E. coli is the most common cause of epididymitis. Even if you suspect that he isn't monogamous, E. coli is still #1 in his age group. If he were in his 20s, then Chlamydia trachomatis would be the most likely.

A 23-year-old woman comes to your office after being told by a local physician that she is HIV-infected. She had been tested as part of a routine physical examination. She has never injected intravenous drugs and has had sexual intercourse in her lifetime with only 4 men, each of whom reportedly has never used drugs and is healthy. At the time of physical examination, she is very anxious and tearful. Her complete physical examination is normal except for 2 lymph nodes in her posterior cervical chain that measure 0.3 x 0.5 cm. Her previous test results from her doctor show an HIV ELISA that is positive and a Western blot assay that is "indeterminate." You repeat her Western blot assay 3 months later, and it shows no bands (negative). A repeat Western blot assay at 6 months shows no bands (negative). Which of the following is the best interpretation of her initial lab result that prompted her to see you? A) HIV-2 infection. B) She has HIV; the Western blots are now false negatives because of her advanced disease. C) Resolved HIV-1 infection. D) Early HIV-1 infection. E) False-positive HIV ELISA for HIV-1.

Answer E. False-positive HIV ELISA for HIV-1. Explanation She never should have been told she had HIV infection. False-positive HIV ELISAs are quite common; that is why we must wait for the Western blot results. Unfortunately, indeterminate Western blots are also seen occasionally. Indeterminate Western blots generally are false-positive results and are positive in only 1 band that is tested. The other thing to note is that this patient is really at low risk. When you test a low-risk population with a highly sensitive test, you are likely to see a large number of false-positive test results. It was possible early on that this was early HIV-1 infection; however, by 3 months and 6 months, she should have developed more bands. Resolved HIV infection in adults has not been clinically described. HIV-2 infection would show up on the Western blot. The other choice that this is a false-negative Western blot is very unlikely at this point. Additionally, the best test to help discern this woman's status would be an HIV viral load or an HIV PCR DNA test (qualitative). These would be useful early in determining if her indeterminate Western blot was a false-positive.

A 25-year-old man with a history of IV drug abuse is admitted with fever, malaise, headache, and weakness. He denies other symptoms at this time. He has not been in the hospital before. PAST MEDICAL HISTORY: Negative; admits to using IV drugs for 5 years and currently still using SOCIAL HISTORY: Works at a local restaurant as a cook Drinks two 6-packs of beer on the weekends Smokes 1 pack of cigarettes daily FAMILY HISTORY: Father 50 and healthy Mother 50 and healthy REVIEW OF SYSTEMS: Weight loss of 10 lbs noted for the past 2 months Chronic "smoker" morning cough Occasional night sweats PHYSICAL EXAMINATION: BP 98/70, RR 30, Temp 102° F, P 110 HEENT: PERRLA, EOMI TMs clear Throat slightly erythematous Neck: Supple; no lymphadenopathy noted Heart: RRR without murmurs, rubs, or gallops Lungs: Coarse breath sounds; clear with cough Abdomen: Bowel sounds present; no hepatosplenomegaly Extremities: No cyanosis, clubbing, or edema LABORATORY: Blood cultures grow Salmonella enterica, serotype typhimurium You start him on intravenous ceftriaxone initially and then change him to oral ciprofloxacin 750 mg twice daily to complete a 2-week course. He follows up as an outpatient at the end of therapy and is doing well. However, 4 weeks later, he returns with the same symptoms, and blood cultures grow Salmonella enterica, serotype typhimurium again. Which of the following diagnostic tests should you order at this time? A) Bone marrow biopsy B) Intravenous urography C) Quantitative serum immunoglobulins D) CT scan of the head E) HIV ELISA

Answer E. HIV ELISA Explanation Patients with HIV infection have an increased risk of having recurrent infections with Salmonella. All Salmonella are now considered to be in a single species, S. enterica, with various serotypes that were previously considered to be seperate species, such as typhimurium. Know this: If someone has recurrent bacteremias with Salmonella enterica or Streptococcus pneumoniae, think about HIV! If you have a patient from Southeast Asia who is growing Penicillium marneffei, then also think about HIV. The other choices are much less likely to be helpful in this patient. Be aware also that complement deficiency can predispose to these infections too, as well as sickle cell patients and others without functional spleens. I'd also be worried about endocarditis and consider an echocardiogram as well.

A 27-year-old man from North Carolina presents with a 48-hour history of fever, fatigue, anorexia, diffuse myalgias, and headache. Upon awakening on the day of presentation, he noticed a rash that continues to worsen. On physical examination, his temperature is 102.6° F. He appears ill and has an erythematous maculopapular and petechial rash that includes the trunk and, most prominently, the wrists, ankles, palms, and soles. Which of the following laboratory findings is most likely to be identified in this patient? A) Elevated serum levels of amylase and lipase B) Hyperkalemia C) Thrombocytosis D) Elevated WBC count associated with prominent lymphocytosis E) Hyponatremia

Answer E. Hyponatremia Explanation The patient has clinical signs and symptoms consistent with Rocky Mountain spotted fever (RMSF), caused by Rickettsia rickettsii . Patients complain of a variety of systemic signs and symptoms which include increased temperature, severe myalgias, headache and/or photophobia, anorexia often associated with abdominal pain, nausea, vomiting, diarrhea, and rash. The rash usually appears within the first several days of onset of the disease, beginning as an erythematous and macular rash, which progresses to maculopapular and petechial lesions. It begins in the distal portions of the extremities, including the palms, soles, wrists, and ankles, often spreading to the trunk within 12-24 hours. Although characteristic of RMSF, the rash may be absent in up to 1/5 of all patients, which makes the clinical diagnosis more difficult. Certain laboratory findings are helpful in diagnosing the infection, including hyponatremia, thrombocytopenia, elevated serum transaminases, leukopenia, and anemia. Damage to the endothelial cells lining the small blood vessels leads to a systemic vasculitis, vascular leakage, edema, and hyponatremia. Many tick species (American and brown dog ticks, the wood tick) are capable of transmitting Rickettsia rickettsii to humans. RMSF is most common from May to October and occurs primarily in the eastern coastal, southeastern, and far western states. The treatment of choice is doxycycline.

A 40-year-old woman presents with genital lesions that started yesterday. They are on both labia majora, are itchy and painful, and she has never had them before. In the past few months, she has been dating a new boyfriend. They have had sexual intercourse on numerous occasions, and she reports the use of a diaphragm for birth control. On examination, you find clustered vesicles consistent with the diagnosis of herpes simplex (HSV) and begin her on a course of valacyclovir. She is concerned about her diagnosis of genital herpes and has several questions for you. Which of the following is true regarding her genital herpes infection? A) A Tzanck smear would be negative at this point. B) Serum serology will be predictive of liklihood or recurrence. C) Use of anti-HSV drugs will not be beneficial in preventing future outbreaks. D) If she were pregnant with active lesions, a C-section would not be helpful. E) It is possible for her to spread the virus to others even if lesions are not present.

Answer E. It is possible for her to spread the virus to others even if lesions are not present. Explanation Patients with HSV infection may spread by asymptomatic shedding of the virus (i.e., without evidence of active lesions). If she were pregnant with active lesions, a C-section would be indicated because it would markedly reduce transmission to the newborn. Use of acyclovir, famciclovir, or valacyclovir would be very beneficial if she has recurrent outbreaks but is not given prophylactically until the frequency and severity of recurrences are known. A Tzanck smear of active lesions is highly likely to be positive. Serology can help distinguish HSV-1 from HSV-2 infection, and the latter would have a higher likelihood of recurrence. However, this is better determined by a culture (which should be done in this case) and, given she has only 1 day of symptoms, her antibodies to either virus would not be present from the current infection yet.

A 55-year-old man has fever and cough. He has AIDS and a history of disseminated histoplasmosis 2 years ago. He was treated initially with amphotericin B and had been maintained on itraconazole since. Last month, he started having fever and chills. Histoplasmosis was suspected again, and amphotericin B was restarted. Blood cultures, however, grew Mycobacterium intracellulare, and he was started on appropriate medications for that. He was restarted on his itraconazole maintenance therapy last month. Today, he comes in with recurrence of his fever and chills. MEDICATIONS: Combivir and efavirenz for 6 months Rifabutin 600 mg daily for 1 month Azithromycin 500 mg daily for 1 month Ethambutol 1200 mg daily for 1 month Itraconazole 200 mg daily for 2 weeks Ranitidine 150 mg bid for 2 weeks for gastric reflux symptoms Trimethoprim/sulfamethoxazole DS 1 daily PHYSICAL EXAMINATION: Temp 103° F, BP 120/70, RR 24, P 100 HEENT: PERRLA, EOMI TMs clear Throat: clear Neck: Supple Heart: RRR without murmurs, rubs, or gallops Lungs: Diminished breath sounds at left base Abdomen: Liver span 14 cm Spleen tip palpable Extremities: No lesions noted LABORATORY: WBC: 1,200 cells/cu mm Hgb: 10 g/dL Platelets: 100,000 Peripheral smear: Yeast forms seen Which of the following is most likely correct? A) The yeast forms are likely contaminants. B) The patient has itraconazole-resistant Histoplasma. C) A serum cryptococcal antigen will be positive. D) Blood cultures will grow Candida krusei. E) Itraconazole and its metabolites are below therapeutic levels.

Answer E. Itraconazole and its metabolites are below therapeutic levels. Explanation Itraconazole is the drug of choice for histoplasmosis but has reduced absorption with reduced stomach acid from H2 blockers or proton pump inhibitors. When this patient was placed on ranitidine with his itraconazole, the itraconazole was likely not absorbed as well, which allowed his histoplasmosis to relapse, as evidenced by yeast in the blood. It is more likely that he is reactivating a prior yeast infection rather than acquiring a new one (e.g., Cryptococcus or Candida krusei). Histoplasma capsulatum is rarely resistant to the azoles. Yeast in the blood should never be considered a contaminant.

A 34-year-old man is brought to the emergency department in November with a painful rash on his hands and feet that began today. He has had fever and severe myalgias for one day. Three days ago, he had traveled to visit relatives in Texas. They had dogs that were known to have ticks, but he does not recall a tick bite. He is sexually active with the same partner for 14 years and denies any sexually transmitted diseases. He has never been tested for human immunodeficiency virus (HIV) infection. Past Medical History: Asthma Social History: Does not smoke. Drinks 3 beers on weekends. Family History: Both parents alive and well. No siblings or children. Review of Symptoms: He denies headache, shortness of breath, or adbominal pain Physical Exam: BP 78/40 mmHg, PR 128, RR 24, Temp 101° F (38.3° C) HEENT: Bilateral large conjunctival hemorrhages Neck: Supple Nodes: None Heart: No murmurs or gallops Lungs: Clear to auscultation and percussion Abdomen: Bowel sounds normal, no organomegaly Extremities: Petechiae, purpura, and ecchymoses on the dorsal aspects of hands, wrists, feet, and ankles CBC: WBC 18,000, 54% polymorphonculear, 18% bands, 8% monocytes, 20% lymphocytes. Hb 13.1 g/dL, platelets 53,000 mm 3 PT/INR: 17/2.26 PTT: 57s D-dimer: 8.0 Creatinine: 2.8 mg/dL Which of the following is the most likely diagnosis? A) Gonococcemia B) Rocky Mountain spotted fever C) Toxic shock syndrome D) Secondary syphilis E) Meningococcemia

Answer E. Meningococcemia Explanation This patient is presenting with fever and a rapidly progressive painful acral rash in association with hypotension and disseminated intravascular coagulopathy (DIC), which is consistent with purpura fulminans. Meningococcemia is the most common cause of purpura fulminans and commonly leads to early septic shock and organ dysfunction. Although secondary syphilis causes an acral rash, it is not associated with coagulopathy; and the lesions are maculopapular, not petechial/purpuric. Toxic shock syndrome certainly causes shock, multiple organ injury, and DIC—but the rash is a diffuse blanching erythema. Rocky Mountain spotted fever does cause an acral petechial rash, and the patient has had exposure to dog ticks, but DIC is uncommon (< 10%), and the infection is rare after September in the calendar year. Also, the incubation period is usually > 3 days, and the rash typically occurs > 5 days after the onset of fever. Gonococcemia rarely causes organ failure or DIC, and the rash is usually a smattering of pustules.

A 70-year-old man with chronic renal insufficiency presents for evaluation. He is on chronic hemodialysis, which he receives 3 days a week at his local dialysis center. Yesterday morning, he developed a fever and comes in for his usual dialysis. He is not that ill but has a temperature of 101.5° F. His graft site looks fine and he is without other complaints. Physical examination is unremarkable. Blood cultures are taken and within 24 hours are growing an organism. Which of the following organisms is most likely, given his current examination and findings? A) Methicillin-resistant Streptococcus pneumoniae. B) Methicillin-sensitive Staphylococcus epidermidis. C) Methicillin-resistant Streptococcus aureus. D) A gram-negative organism is most likely. E) Methicillin-resistant Staphylococcus epidermidis.

Answer E. Methicillin-resistant Staphylococcus epidermidis. Explanation Note that he is not that ill, so a gram negative is much less likely. MRSE is the most likely, followed by MRSA. Note that methicillin-resistant Streptococcus aureus does not exist. There is no genus/species of Streptococcus aureus. Also, Streptococcus pneumoniae is a respiratory pathogen and the patient has no signs or symptoms of a respiratory illness. Vancomycin would be the drug of choice for MRSE and would also be useful against MRSA as well.

A 60-year-old retiree named Ken is enjoying his first cruise in the Caribbean. The ship has been at sea for 2 days, and many passengers have complained of diarrhea, which the patient attributes to rough seas. He got a scopolamine patch from his doctor and has not felt the least bit nauseous. However, suddenly at 2 a.m., he awakens with the urge to defecate and has explosive diarrhea. This continues throughout the remaining night. He calls the ship's doctor but cannot get through because the line is busy. He manages to make it to the infirmary and notes 30-40 passengers in the hallway, all with similar symptoms. What is the most likely etiology for this unfortunate diarrheal illness? A) Rotavirus B) Campylobacter jejuni C) Cyclospora D) Clostridium difficile E) Norovirus (formerly known as Norwalk virus)

Answer E. Norovirus (formerly known as Norwalk virus) Explanation This has been the subject of much public scrutiny as well as bad press for the major cruise lines. Usually an infected employee "infects" other crew members and passengers in this fast spreading, oral-fecal route. It is self-limited but in the immunocompromised or debilitated can cause severe dehydration. Most resolve without specific therapy. Rotavirus is associated with children and daycare outbreaks. Campylobacter is commonly associated with pet exposure, particularly puppies. Clostridium is usually antibiotic associated. Cyclospora occurs with ingestion of infected raspberries. If the outbreak was in a small number of people and occurred after eating at a "buffet," then think about Staphylococcus or Bacillus toxin from food sitting out too long.

A previously healthy 24-year-old Caucasian woman comes to your office for evaluation of joint pain and fever. She had been well until 3 days ago, when pain and swelling developed in the left knee and right ankle. Skin lesions also appeared, primarily on the extremities. The patient has a dozen sexual partners, who are reported to be healthy. On physical examination, temperature is 38.2° C (100.8° F). Approximately 5 papulopustular skin lesions are present on the extremities; a few are also noted on the lower torso. There is tenderness with mild swelling in several joints, including the right knee, and the left wrist and the base of the right thumb are tender. Based on your findings, which of the following is the most likely organism responsible? A) Borrelia burgdorferi B) Streptococcus pyogenes C) Chlamydia trachomatis D) HIV E) Neisseria gonorrhoeae

Answer E. Neisseria gonorrhoeae Explanation She has the classic findings of disseminated gonococcal infection (DGI). Two main syndromes of DGI are 1) polyarthralgia/dermatitis/tenosynovitis (as in this case), and 2) purulent arthritis without skin lesions. Overlap syndromes may also occur. Note: If she was having menses, or recently completed her menstrual period, that would have been even more indicative of this type of infection. Other risk factors include pregnancy and SLE. Remember that women are more likely than men to have this infection. It is unlikely that you will grow the organism from the blood, but possibly from the genitourinary tract. However, concurrent symptomatic genital infection is rare. The other thing to remember is that ceftriaxone is the treatment of choice because of the resistance problem with penicillin and the quinolones in the United States. None of the other choices would fit this picture except for possibly Streptococcus pyogenes—but endocarditis would have to be in the differential, and she has no evidence of this from the history.

A 20-year-old Mexican-American man is brought into the emergency department with a seizure. This is his first known seizure, and he has been in excellent health in the past—he runs marathons for the Mexican Olympic team. PAST MEDICAL HISTORY: Negative Immunizations up-to-date, including 2nd MMR and hepatitis B SOCIAL HISTORY: Lives with relatives in San Diego Works as a courier for an express mail company FAMILY HISTORY: Mother and father both healthy; live in small community outside of Mexico City REVIEW OF SYSTEMS: Completely negative PHYSICAL EXAMINATION: BP 120/70, RR 16, Temp 98.6° F, P 50 HEENT: PERRLA, EOMI TMs clear Throat clear Neck: Supple; no masses Heart: RRR without murmurs or rubs; healthy gallop Lungs: CTA Abdomen: Benign Extremities: Normal, no rashes LABORATORY: CT of head: 8 cystic lesions, 1 cm to 3 cm in diameter; 2 of the lesions enhancing slightly with contrast HIV ELISA: Negative CD4: 2,000/cu mm Which of the following is the most likely diagnosis? A) Herpes simplex virus B) Lymphoma C) Cryptococcal meningitis D) Toxoplasma gondii E) Neurocysticercosis

Answer E. Neurocysticercosis Explanation This term is used for human CNS infection with Taenia solium cysts. Most literature involves intracerebral cysts that cause seizures and mass effects. Intraventricular cysts, as well as subarachnoid cysts, can also occur. Drug therapy with albendazole is the treatment of choice. Sometimes, corticosteroids are also required if CNS inflammation is severe or causing symptoms. He does not have HIV, so that really throws Toxoplasma and Cryptococcus out the window. Cystic lesions would be very unusual with lymphoma. HSV should not do this normally. Also with this, think of someone with seizures who has this lesion but who hasn't traveled anywhere. The kicker is that they live with a housekeeper or someone who has the infection and is transmitting it fecally/orally in the house.

A 22-year-old pop singer has been ill with diarrhea for the past 2 weeks. She says that she noted this while she was performing in Latin America. You are the physician for the cruise line that she is performing for now. She tells you that she has had some low-grade temperatures since returning from her tour. She did not eat any fresh vegetables (unless you call French fries a fresh vegetable). She likes to eat beef jerky and prefers the "extra salty" version. She drank only bottled water and a soft drink for which she is a national spokesperson. She did drink these beverages poured over ice, but she thought that the "frozen stuff would kill the cooties." She has not noted any blood in her stool. She has lost about 2 pounds in the last week. Past Medical History: Cosmetic surgery at age 16 Depression since age 14, on no medications at the moment Social History: Sexually active with multiple partners Smokes 1 pack/day for the past 3 years Denies illicit drug use Denies use of alcohol Family History: She and her mother are estranged at the moment. Father left when she was 12 years of age Sister healthy Recently was married in Las Vegas; marriage was annulled after 24 hours—"I don't know what I was thinking." Review of Systems: Diarrhea is intermittent, and she has crampy abdominal pain on occasion. No rash No burning on urination No chills Diminished appetite Physical Examination: General: Pink hair with numerous piercings VS: Temp 100.0° F, BP 110/70, Pulse 95, RR 16 HEENT: PERRLA, EOMI Throat: Clear Heart: RRR with no murmurs, rubs, or gallops Lungs: CTA Abd: Hyperactive bowel sounds, non-tender examination; no hepatosplenomegaly GU: Normal female genitalia; no tenderness on bi manual palpation; no discharge noted Extremities: No cyanosis, clubbing, or edema Rectal: Heme positive (slight) Laboratory: Check for stool leukocytes: Positive Giardia specific antigen: Negative Stool culture: Salmonella enteritidis beta-lactamase producing Based on this information, which of the following is the best treatment? A) Ciprofloxacin 500 mg bid for 10 days B) Erythromycin 500 mg bid for 5 days C) Tetracycline 500 mg qid for 10 days D) Amoxicillin 500 mg tid for 10 days E) No antibiotic therapy

Answer E. No antibiotic therapy Explanation With uncomplicated Salmonella gastroenteritis in otherwise healthy young hosts, antibiotic therapy is not indicated. Treatment with antibiotics does not shorten the duration of illness, but it does prolong the carrier state and promote resistance. Treatment should be given to those over 50 and those who are immunocompromised. We treat them with antibiotics because the risk that the Salmonella may disseminate or cause more extensive problems is greater than the risk of prolonged shedding.

A 28-year-old man is referred to you because he is found to be seropositive for Epstein-Barr virus. For a year he has had difficulty falling asleep, as well as frequent awakening during the night. He has had increasing problems with daytime fatigue and reports that he can't concentrate as well. He has lost interest in his hobbies; he used to enjoy playing basketball avidly. He has gained 20 lbs in the last year and says that he "just doesn't feel like exercising." PAST MEDICAL HISTORY: Negative; started coming to the local physician about 1 year ago with sleep disturbances SOCIAL HISTORY: Divorced about 2 years ago Has 2 children; wife has custody "Recovered alcoholic"; hasn't had a drink in 5 years Smokes 1 pack of cigarettes daily No illicit drug use FAMILY HISTORY: Mother age 60; history of depression Father age 60; hypertension REVIEW OF SYSTEMS: No fever No chills No sore throat No lymph node enlargement noted No diaphoresis No cough No palpitations No constipation No diarrhea No risk factors for HIV PHYSICAL EXAMINATION: BP 120/70, P 90, RR 18, Temp: 99° F Ht. 5'10" Wt. 190 lbs (moderate truncal obesity) HEENT: PERRLA, EOMI TMs clear Throat clear; no erythema; no obvious dental caries noted Neck: Supple Heart: RRR without murmurs, rubs, or gallops Lungs: CTA Abdomen: Bowel sounds present; liver span 6 cm; no spleen palpated Extremities: No cyanosis, clubbing, or edema LABORATORY: CBC: Normal Electrolytes: Normal Liver enzymes and panel: Normal EBV titers: Only positive is EBV viral capsid (VCA) specific IgG antibody titer at 1:160 Based on your findings, which of the following do you think he needs? A) CMV testing. B) Referral to an infectious disease specialist for evaluation of chronic EBV syndrome. C) Check EBV DNA viral load. D) Measurement of T-lymphocytes. E) No further infectious workup is indicated.

Answer E. No further infectious workup is indicated. Explanation He has classic signs of depression, with changes in sleeping and eating patterns and loss of interest in usual entertaining activities. EBV serology has been blamed for many things, from chronic fatigue to a persistent EBV syndrome. He has a completely normal examination. You have no documentable symptoms except his fatigue, and he has had major life complications in the past year. These patients can be very difficult to convince that depression could be at the root of their problem. Frequently, it takes a combination of medication as well as an effective counselor to help these patients get back to a normal life. After starting on an antidepressant and receiving counseling, he returned to his youthful healthy self, lost 30 lbs in 6 months, and was remarried 3 years later. Not all patients are success stories like this, but it should be remembered that a nonspecific laboratory test, if misinterpreted, can frequently cause more harm than good.

A 25-year-old nursing student works on an AIDS ward in New Jersey. While emptying out the urine bag from an HIV-infected patient (viral load > 500,000 copies/cc), some of the urine splashed onto her ungloved hand. No lesions were noted on her hand, and she does not have any noticeable breaks in the skin. She immediately washed her hand with soap and water and poured bleach over the area. The urine was light yellow in color. She presents to the emergency room for follow-up care approximately 2 hours after the splash. Which of the following should be recommended to prevent morbidity? A) Start AZT + 3TC (Combivir®) only. B) Perform a genotype on the infected patient and initiate therapy based on results of the genotype (i.e., to determine if the patient has resistant virus). C) Start zidovudine only. D) Start 3-drug therapy. E) No treatment is recommended.

Answer E. No treatment is recommended. Explanation A urine exposure to intact skin requires no postexposure prophylaxis. If this had been a needle-stick, then starting antiretroviral therapy with 3-drug therapy would have been correct. Use of genotype testing has not been evaluated and currently is not recommended by the CDC. Using a single drug is never the correct answer. Using 2 drugs is appropriate in some circumstances, but the Boards will usually ask you a straightforward, "no therapy" or 3-drug therapy question for postexposure prophylaxis.

A 70-year-old woman is visiting the beautiful countryside of Colorado. She decides to take off her tennis shoes and goes walking in a field. While walking, she steps on a dirty nail. She cries out in pain and hobbles to a nearby meadow to sit down. While hobbling, she steps in a huge cow patty. After sitting down, her dog licks the injured foot. She comes into the Emergency Department wondering if she needs to have a tetanus immunization. On history, you learn she has had > 3 tetanus immunizations in the past, with the most recent 4 years ago. What do you recommend for her besides avoiding nails, cow poop, and dogs licking her feet? A) Culture the wound and give Td if it grows Clostridium tetani. B) She requires Td only. C) She requires tetanus immune globulin only. D) She requires Td and tetanus immune globulin. E) She requires no tetanus immunizations today.

Answer E. She requires no tetanus immunizations today. Explanation Here are the protocols to remember: Wound is dirty and < 3 tetanus immunizations or the immunization history is unknown—use TIG + Td. Wound is clean and immunizations are up-to-date (most recent < 10 years)—no treatment. Wound is dirty and immunizations are up-to-date, with most recent immunization < 5 years—no treatment. Note that if she needed a booster dose, she is too old to receive Tdap (the upper age limit is 64 years old); otherwise, that would be an appropriate choice for her.

A 45-year-old man underwent prosthetic mitral valve replacement 6 weeks ago. He comes in today with fever and chills and is found to have a vegetation on the annulus of the prosthetic valve. Which of the following organisms would be the most common cause of endocarditis in this patient? A) Candida species B) Enteric gram-negative rods C) Viridans streptococci D) Anaerobes E) Staphylococcal species

Answer E. Staphylococcal species Explanation The microbiology of prosthetic valve endocarditis is related to how soon after the surgery endocarditis occurs. Early endocarditis occurs within 2 months of valve implantation and late endocarditis is after 2 months. The pathogenesis of early prosthetic valve endocarditis is that organisms are implanted at the time of surgery and thus are usually skin flora, which is why staphylococcal species predominate. The prevalence of Staphylococcus aureus vs. coagulase-negative staph is approximately the same. Late endocarditis occurs due to bacteremias unrelated to the valve implantation surgery itself and thus resembles native valve endocarditis, with viridans streptococci being the most common infecting organism. In 2009, the International Collaborative Endocarditis published the largest review to date on prosthetic valve endocarditis. S. aureus was the cause in 23%, followed by coagulase-negative staphylococci.

A 28-year-old man with chicken pox presents in shock with a blood pressure of 70/50. He is noted to have a rash on his palms/soles and lower extremities. Laboratory returns and it appears that he has renal failure (creatinine 3.5 mg/dL); liver abnormalities (ALT 400, AST 450, PT 16); thrombocytopenia with platelets of 80,000; mucous membrane changes; and diarrhea. A blood culture grows an organism. What is the likely etiology of his condition? A) Francisella tularensis B) E. coli C) Rickettsia rickettsii D) Staphylococcus aureus E) Streptococcus pyogenes

Answer E. Streptococcus pyogenes Explanation This patient has toxic shock syndrome due to group A streptococcus. Remember that in toxic shock due to this organism, the blood culture is more likely positive; with Staphylococcus aureus, the blood culture is rarely positive. The multiple organ dysfunction listed should be the clue. Also, a history of chickenpox is classic for group A streptococcus superinfection. He does not have risk factors for E. coli, and nothing in the history makes you think of RMSF—no ticks, no North Carolina or similar state, no petechiae mentioned. Tularemia would be even rarer and again you need geography (Arkansas, Missouri, Oklahoma) and a tick bite or animal exposure.

A 20-year-old woman who underwent a splenectomy in 2000 after an MVA resulted in splenic rupture presents for evaluation. She received vaccination against pneumococcus and H. influenzae. She has been visiting the islands off Massachusetts and presents with high fever to 105° F, shaking chills, and rigors. She is quite pale. Laboratory returns and shows a severe hemolytic anemia. What is the best way to diagnosis her infection? A) Stool culture B) Blood culture C) Urine culture D) CT scan of the head to look for cysts E) Thick and thin blood smears

Answer E. Thick and thin blood smears Explanation She most likely has Babesia microti infection. This organism is seen in the Northeast U.S. and presents much like malaria. Hemolytic anemia is common and much more severe in splenectomized individuals. Treatment of severe infections is with clindamycin plus quinine or atovaquone and azithromycin. In severe cases, exchange transfusion may be warranted.

A 19-year-old female presents to the emergency department with a one-day history of septic symptoms. She reports a significant toothache over the last three days; and, on the morning of presentation, she developed vomiting, diffuse muscle pain, fever, some confusion, and a diffuse erythematous, macular rash with nondescript borders. She has no medical history and is taking no medications. Her last menstrual cycle was 10 days previously and was normal. On exam, she is in moderate distress. Her temperature is 38.9° C (102.0° F); heart rate is 136/min with weak peripheral pulses; blood pressure is 84/54 mmHg; and respiratory rate is 18/min. She has significant orthostatic changes. There is bilateral conjunctival erythema and significant swelling over the left maxillary area with an apparent complicated tooth abscess. There is significant hyperemia of the mucous membranes. Lungs demonstrate good air movement with clear auscultation. Cardiac exam reveals a rapid rate and no murmurs. Abdomen is unremarkable. Skin is erythematous and generally warm to the touch; no desquamation, blistering, or tenderness is noted. Patient is started on aggressive intravenous hydration; blood and urine cultures are obtained; and she undergoes drainage of the dental abscess. She is also started on a broad-spectrum antibiotic. Blood and urine cultures showed no growth, and cultures obtained during abscess drainage revealed Gram (+) cocci in clusters. Staphylococcus aureus grows in culture of the abscess. Which of the following is the most likely explanation for these findings? A) Staphylococcal scalded skin syndrome (SSSS) B) Stevens-Johnson syndrome C) Toxic epidermal necrolysis D) Erysipelas E) Toxic shock syndrome

Answer E. Toxic shock syndrome Explanation Toxic shock syndrome is a toxin-mediated process resulting from infection with either a toxin-producing Staphylococcus aureus or Streptococcus pyogenes . Although initially associated with tampon usage, current presentations can arise from any infection due to staph or strep organisms. Absence of host antibodies to the circulating toxin appears to be a primary risk for developing a toxic shock presentation. Treatment requires prompt intravenous fluid support, identification of the primary infection locus, and aggressive treatment of the underlying infection, including drainage of any localized purulence. Pending cultures, treatment is optimized with a β-lactamase-resistant, anti-staphylococcal antibiotic in combination with clindamycin. Clindamycin inhibits protein synthesis, which reduces the production of toxins and cytokines contributing to the systemic findings. Bacteria in deep-tissue abscesses are often in a stationary phase of growth, and very few organisms are replicating; hence, beta-lactam drugs that act on the cell wall during logarithmic growth are not the most effective drugs. Adding clindamycin helps to kill these organisms in stationary phase. Desquamation, which is common in this disease process, is often delayed by 10-14 days from the original presentation. Both Stevens-Johnson syndrome and toxic epidermal necrolysis are often caused by severe drug reactions that affect epidermal integrity resulting in varying degrees of epidermal detachment. Both can affect internal organs as well, resulting in a poor prognosis. This rash was present before the onset of medications and is not consistent with either of these diagnoses. Staphylococcal scalded skin syndrome (SSSS) is mediated by two types of toxin—exfoliative toxin A and exfoliative toxin B. It is more common in younger children and displays blister formation—and often severe exfoliation. The skin is often tender to palpation, and the mucous membranes are usually spared. Erysipelas is an infection typically caused by beta-hemolytic streptococci. The presence of the rash is typically concurrent with systemic symptoms, if these are present. Erysipelas is generally well-demarcated with raised edges, brightly erythematous, and tender to the touch. Rarely are systemic symptoms associated.

A 60-year-old woman with poorly controlled diabetes mellitus presents with a 2-day history of nasal stuffiness and frequent nose bleeds. She has some sinus tenderness on examination, and when you look into her nasal cavity, you see a small black necrotic area on the nasal turbinate. Which of the following organisms should you be most concerned about? A) Mycobacterium nasalitum B) Mucor species C) Haemophilus influenzae non-typeable D) Sporothrix schenckii E) Streptococcus pneumoniae

B. Mucor species Explanation Mucor is the organism to be most concerned about. She has poorly controlled DM with a black necrotic area in her nasal cavity. Mucor must be ruled out and, unfortunately, is the likely diagnosis. Morbidity (and mortality) are high because the organism likes to "grow back" into the brain. The bacterial organisms listed are not consistent with this clinical picture, and Sporothrix is associated with gardeners and rose bushes with finger/hand lesions. Mycobacterium in this location is very unlikely (the species name, "nasalitum," is not a real species).

A 30-year-old male immigrant, recently arriving from a remote area of the former Soviet Union, presents with a 2-day history of headache, generalized malaise, sore throat, and cough. His temperature is 101° F. Findings on physical examination include prominent suboccipital and postauricular lymphadenopathy and soft palate petechiae distributed among larger reddish spots. Which of the following is the most likely cause of this patient's clinical signs and symptoms? A) Coxsackievirus B) Epstein-Barr virus C) Rubella virus D) Group A beta-hemolytic streptococcus E) Rubeola virus

C. Rubella virus Explanation He has clinical signs and symptoms often observed during the prodromal period of rubella, or German measles. Unlike in most children, in adults the typical rash of rubella may be preceded by up to 5 days of increased temperature, generalized malaise, headache, sore throat, cough, and/or coryza. Associated clinical signs include posterior cervical, postauricular and suboccipital lymphadenopathy and an enanthem—termed Forchheimer's sign, characterized by soft palate petechiae and/or larger reddish spots. The exanthem of rubella, which usually lasts about 72 hours, is characterized by numerous, generalized, fine, discrete, pinkish maculopapules which begin on the face and then spread to and coalesce on the trunk during the 2 nd day, usually disappearing completely by the 3 rd day. Oftentimes the rash appears to fade as it spreads. In fact, the rapidly changing nature of the rash is characteristic of rubella and serves as a useful diagnostic tool. Adolescent and older females often have associated arthritis and arthralgias, beginning on the 2 nd and 3 rd day of illness and lasting up to 5-10 days. The disease is contagious 2-3 days prior to and after the appearance of the rash. The incubation period ranges from 14-21 days.

An 18-year-old freshman at Bowling Blue University in northeastern Lyme, Connecticut presents to you at the school infirmary with a 2-day history of fever, cough, runny nose, and conjunctivitis. He initially thought he had a cold since a friend of his had a similar illness 2 weeks ago. Last night, however, he developed a rash that began at the back of his neck and has started to spread downward. He denies tick bites and reports that he is not nor has he ever been sexually active. On physical examination, you note that his rash is maculopapular in appearance and indeed starts at his hairline and continues to his back and trunk. On examination of his oral mucosa, you note whitish spots on a red base. Which of the following is true? A) People with egg allergy should not receive the live virus vaccine. B) Immunize all immunocompetent people at the college born in or after 1957 with a live virus vaccine if you cannot document 2 vaccinations against this virus have been given previously. C) Asymptomatic HIV-infected students at the college should not receive the vaccine. D) Notify unimmunized pregnant women exposed to this illness that they are at high risk for congenital malformations in their infants. E) Immunize everyone regardless of age or immunization history with a killed virus vaccine

Infectious Disease Answer B. Immunize all immunocompetent people at the college born in or after 1957 with a live virus vaccine if you cannot document 2 vaccinations against this virus have been given previously. Explanation This patient has measles. Everyone born in or after 1957 should be immunized with MMR (or monovalent measles vaccine, if available) if 2 previous measles vaccinations cannot be documented. Those born before 1957 are considered immune and those with 2 documented vaccines are also considered immune. Rubella infection is much more likely than measles (rubeola) virus to cause a problem in fetuses. Previously egg allergy was a contraindication to MMR or measles vaccine, but the vaccine has been shown to be safe in patients with history of egg allergy. Asymptomatic HIV-infected patients should receive the vaccine if they are not immunized.

A 60-year-old RN has newly diagnosed herpes zoster infection. His infection is located on his right scapula and spreads laterally over 3-4 dermatomal areas. He is not immunocompromised and is otherwise healthy. He works in an outpatient clinic. What can you tell him about his zoster infection? A) If he had presented last week with a "zoster-prodrome" (e.g., pain before the rash appeared), you should have started him on anti-virals. B) A Tzanck smear would be negative. C) He may not return to work until all his lesions are crusted over. D) The duration of post-herpetic neuralgia may be decreased by using valacyclovir. E) This infection is due to reactivation of herpes simplex virus.

Infectious Disease Answer D. The duration of post-herpetic neuralgia may be decreased by using valacyclovir. Explanation Using famciclovir or valacyclovir may decrease the duration of post-herpetic neuralgia if begun soon after rash onset. No data supports the use of these agents in the "prodrome" period. Tzanck smear would be positive and would confirm the diagnosis if the diagnosis was otherwise unclear. Zoster is due to reactivation of varicella virus infection, not herpes simplex. He may return to work immediately as long as he can keep the rash covered with clothes (but saying that, if he works with immunocompromised patients—say in a bone marrow transplant unit—then you would transfer him to a less immunocompromised patient group). He is of little risk to patients unless they somehow could have contact with his lesions, which is very unlikely given their location. There is no respiratory spread of varicella in this form of the infection (unlike with the clinical syndrome, chickenpox).


Set pelajaran terkait

FINC Sirmans Final Quizzes and Menti Questions

View Set

Principles of Personal Health Unit 1

View Set

Chapter 5 TCI World History 9/29/19

View Set

BIO 110 EXAM 4 CLICKER QUESTIONS

View Set

M9 - Chapter 6 (Participating Life Insurance Policies)

View Set